Exam 3 Old Content

¡Supera tus tareas y exámenes ahora con Quizwiz!

A client has a low serum potassium level and is ordered a dose of parenteral potassium chloride (KCl). How does a nurse safely administer KCl to the client? a. Administers 5 mEq intramuscularly (IM) b. Dilutes 200 mEq in 1 liter of normal saline and infuses at 100 mL/hr c. Infuses 10 mEq over a 1-hour period d. Pushes 5 mEq through a central access line

C. Infuses 10 mEq over a 1-hour period A dose of KCl 10 mEq given over 1 hour is appropriate for this client. A dose of KCl 200 mEq in 1 liter of normal saline infused at 100 mL/hr is too concentrated and can cause injury. Potassium is a severe tissue irritant and is never given by the intramuscular or subcutaneous route. Because rapid infusion of potassium can cause cardiac arrest, potassium is not administered through central lines.

The nurse is providing care to a client who has a serum potassium level of 5.2 mEq/L. Which finding would the nurse expect to assess? Select all that apply. a. Polydipsia b. Cardiac dysrhythmia c. Polyuria d. Muscle weakness e. Diarrhea

B,E The client's potassium level suggests hyperkalemia, which is manifested by anxiety; irritability; gastrointestinal hyperactivity (diarrhea and intestinal cramping); tall, peaked T waves on electrocardiogram; and cardiac dysrhythmias. Muscle weakness, polyuria and polydipsia would be noted with hypokalemia.

When administering 20 mEq potassium chloride intravenously (IV), which is the priority intervention? a. Monitor for pain or burning at the IV infusion site. b. Administer at a rate of 10 mEq/hr. c. Monitor respiratory rate and depth. d. Place the client on a heart monitor during administration.

B. Administer at a rate of 10 mEq/hr. The maximum recommended infusion rate is 5-10 mEq/hr to avoid potentially lethal cardiac dysrhythmias. Monitoring for pain at the IV infusion site, assessing respiratory rate and depth, and placing the client on a heart monitor are all appropriate options, but because a rapid rate of administration could have lethal effects, it has the greatest priority.

What history and assessment findings may be associated with hypocalcemia in a 22-year-old man? Select all that apply. a. Decreased deep tendon reflexes without paresthesia b. Awakening at night with muscle spasms in the calf c. Recent blunt trauma to the throat during a football game d. Absent bowel sounds e. Tingling around the mouth

B. Awakening at night with muscle spasms in the calf C. Recent blunt trauma to the throat during a football game E. Tingling around the mouth A history of anterior neck injury may be associated with hypocalcemia. Symptoms of hyporcalcemia include "charley horses" in the calf during rest or sleep, and tingling in the lips. Hypocalcemia does not affect bowel sounds. Decreased deep tendon reflexes without paresthesia is a neuromuscular change in hypercalcemia.

Laboratory results report a client's serum potassium at 5.6 mEq/L. What does the nurse immediately assess in the client? a. Level of consciousness b. Heart rate c. Bowel sounds d. Feet for paresthesias

B. Heart rate Cardiovascular changes, specifically bradycardia; tall, peaked T waves; rhythm changes to complete heart block; asystole; and ventricular fibrillation are life-threatening consequences of elevated potassium. The provider or Rapid Response Team may need to be notified if changes in heart rate and rhythm are assessed. Paresthesias in the arms and feet and increased intestinal motility are lower-priority signs of elevated potassium. Level of consciousness would not be affected.

When assessing the laboratory results of a client who has hypomagnesemia, for which additional electrolyte imbalance should the nurse monitor? a. Hyperkalemia b. Hypocalcemia c. Hypernatremia d. Hypophosphatemia

B. Hypocalcemia Hypocalcemia often occurs with hypomagnesemia, so the nurse would monitor for signs and symptoms of low calcium levels. Hypomagnesemia may increase potassium secretion in certain circumstances, leading the health care provider to be aware that replacement of magnesium is crucial before attempting to replace potassium if the client is deficient in both. Hypernatremia and hypophosphatemia are not related to hypomagnesemia.

Which component of a client history would most likely be associated with hyperphosphatemia? a. Chronic bronchitis b. Kidney failure c. Hyperparathyroidism d. Colon cancer

B. Kidney failure Hyperphosphatemia in acute or chronic kidney disease results from decreased loss of phosphorus due to the poor function of the kidneys. Chronic bronchitis does not affect phosphorus levels. Hypoparathyroidism (not hyperparathyroidism) may contribute to hyperphosphatemia. Certain cancer treatments, not the cancer itself, may cause hyperphosphatemia.

A nurse is assessing a client and suspects an ECF volume excess. Which finding would the nurse identify as being most significant? a. bounding pulse b. weight gain of 0.75 kg in a day c. increased blood pressure d. slightly distended neck veins

B. weight gain of 0.75 kg in a day Although increased blood pressure, bounding pulse, and distended neck veins are signs of ECF volume excess, rapid weight gain (more than 0.5 kg per day) is the most significant symptom indicating ECF volume excess. A weight gain of 1 kg reflects retention of 1 L of ECF. Additionally, because the veins are very distensible, large volumes of fluid can be retained without any increase in blood pressure or changes in pulse or neck veins.

ANS: A The infant's heart rate is above the lower limit for which the medication is held. A dose of Lanoxin is withheld for a heart rate less than 100 bpm in an infant. The infant's heart rate is acceptable for administering Lanoxin. It is unnecessary to recheck the heart rate at a later time.

Before giving a dose of digoxin (Lanoxin), the nurse checked an infant's apical heart rate and it was 114 bpm. What should the nurse do next? a. Administer the dose as ordered. b. Hold the medication until the next dose. c. Wait and recheck the apical heart rate in 30 minutes. d. Notify the physician about the infant's heart rate.

Which person attending an all-day outdoor festival on a hot August day is at greatest risk for dehydration? a. 32-year-old man consuming alcoholic beverages b. 28-year-old woman with type 1 diabetes mellitus c. 72-year-old woman appearing to be at least 20 pounds overweight d. 68-year-old man who frequently wipes sweat from his forehead

C. 72-year-old woman appearing to be at least 20 pounds overweight The thirst mechanism is less sensitive in older adults, making them more at risk for dehydration. Women of any age have less total body water than men of similar sizes and ages, because men have more muscle mass than women and women have more body fat. (Muscle cells contain mostly water and fat cells have little water.) In addition, the 72-year-old woman is overweight, with an increased percentage of body fat compared to lean body mass, especially skeletal muscle. An obese person has less total water than a lean person of the same weight because fat cells contain almost no water. Although a 32-year-old man consuming alcohol, a 28-year-old with type 1 diabetes, and a 68-year-old who frequently sweats are at risk for dehydration, they are not as high-risk as the obese older adult.

Which acid-base imbalance would the nurse suspect after assessing the following arterial blood gas values: pH, 7.30; PaCO2, 36 mm Hg; HCO3−, 14 mEq/L? a. Respiratory acidosis b. Respiratory alkalosis c. Metabolic acidosis d. Metabolic alkalosis

C. Metabolic acidosis A low pH indicates acidosis. This, coupled with a low bicarbonate, indicates metabolic acidosis. The pH and bicarbonate would be elevated with metabolic alkalosis. Decreased PaCO2 in conjunction with a low pH indicates respiratory acidosis; increased PaCO2 in conjunction with an elevated pH indicates respiratory alkalosis.

The nurse is assessing a client with hyponatremia. Which finding requires immediate action? a. Diminished bowel sounds b. Heightened acuity c. Muscular weakness d. Urine output of 35 mL/hr

C. Muscular weakness Muscle weakness in clients with hyponatremia requires immediate action. If muscle weakness is present, immediately check respiratory effectiveness because ventilation is dependent on adequate strength of the respiratory muscles. Excessive bowel sounds, not diminished bowel sounds, are expected in the client with hyponatremia, as well as mild confusion, not heightened acuity. A urine output of 35 mL/hr is normal (minimally) and does not require immediate action.

Which medication may affect the assessment of a client's fluid balance after a surgical intervention? a. Penicillin twice a day b. Digoxin once a day c. Angiotensin-converting enzyme inhibitor twice a day d. Aspirin once a day

C. Angiotensin-converting enzyme inhibitor twice a day Urine output is used as an indicator of perfusion adequacy after surgery or other procedures. Medications used to manage hypertension such angiotensin-converting enzyme (ACE) inhibitors disrupt the renin-angiotensin II pathway, resulting in greater excretion of water and sodium in the urine. This may make it harder to use urine output as a primary measure of perfusion. Aspirin may prolong clotting. Penicillin and digoxin will not affect monitoring fluid balance postprocedure.

ANS: A Because cow's milk contains very little iron, infants should drink iron-fortified formula for the first year of life.

2. The statement by a mother that may indicate a cause for her 9-month-old having iron deficiency anemia is: a. "Formula is so expensive. We switched to regular milk right away." b. "She almost never drinks water." c. "She doesn't really like peaches or pears, so we stick to bananas for fruit." d. "I give her a piece of bread now and then. She likes to chew on it."

ANS: D The patient's clinical manifestations indicate moderate anemia, which is consistent with a Hgb of 6 to 10 g/dL. The other values are all within the range of normal.

A 62-year old man with chronic anemia is experiencing increased fatigue and occasional palpitations at rest. The nurse would expect the patient's laboratory findings to include a. a hematocrit (Hct) of 38%. b. an RBC count of 4,500,000/μL. c. normal red blood cell (RBC) indices. d. a hemoglobin (Hgb) of 8.6 g/dL (86 g/L).

ANS: A Digoxin has a rapid onset and is useful increasing cardiac output, decreasing venous pressure, and as a result, decreasing edema. Cardiac output is increased by digoxin. Heart size is decreased by digoxin. Digoxin decreases venous pressure.

A beneficial effect of administering digoxin (Lanoxin) is that it a. Decreases edema b. Decreases cardiac output c. Increases heart size d. Increases venous pressure

ANS: C The first manifestations of hypovolemic shock result from compensatory mechanisms. Signs of shock are first evident as changes in cardiovascular function. As shock progresses, changes in skin, respiration, and kidney function progress. The other questions would not identify early stages of shock.

A client brought to the emergency department after a motor vehicle accident is suspected of having internal bleeding. Which question does the nurse ask to determine whether the client is in the early stages of hypovolemic shock? a. "Are you more thirsty than normal?" b. "When was the last time you urinated?" c. "What is your normal heart rate?" d. "Is your skin usually cool and pale?"

ANS: D Hypokalemia is a side effect of both thiazide and loop diuretics. The client loses electrolytes with fluid. Coughing is not a typical side effect of this medication. Headache may occur with any medication and is not a serious side effect. Bradycardia is not likely to occur with this medication.

A client is taking triamterene-hydrochlorothiazide (Dyazide) and furosemide (Lasix). What assessment finding requires action by the nurse? a. Cough b. Headache c. Pulse of 62 beats/min d. Potassium of 2.9 mEq/L

ANS: A All actions are appropriate, but remembering the ABCs, oxygen would come first. The main problem in a sickle cell crisis is tissue and organ hypoxia, so providing oxygen helps halt the process.

A client presents to the emergency department in sickle cell crisis. What intervention by the nurse takes priority? a. Administer oxygen. b. Apply an oximetry probe. c. Give pain medication. d. Start an IV line.

ANS: B Clients taking loop diuretics should be monitored for potassium deficiency from diuretic therapy. A serum sodium level of 135 mEq/L is a normal value. Heart failure may cause renal insufficiency, but a serum creatinine of 1.0 mg/dL represents a normal value. A diuretic may deplete magnesium, but a serum magnesium level of 1.9 mEq/L represents a normal value.

A client with heart failure is taking furosemide (Lasix). Which finding concerns the nurse with this new prescription? a. Serum sodium level of 135 mEq/L b. Serum potassium level of 2.8 mEq/L c. Serum creatinine of 1.0 mg/dL d. Serum magnesium level of 1.9 mEq/L

ANS: C This client has several indicators of sepsis with systemic inflammatory response. The nurse should notify the health care provider immediately. Documentation needs to be thorough but does not take priority. The client may appreciate warm blankets, but comfort measures do not take priority. The client may or may not need insulin.

A nurse caring for a client notes the following assessments: white blood cell count 3800/mm3, blood glucose level 198 mg/dL, and temperature 96.2° F (35.6° C). What action by the nurse takes priority? a. Document the findings in the client's chart. b. Give the client warmed blankets for comfort. c. Notify the health care provider immediately. d. Prepare to administer insulin per sliding scale.

ANS: B Preventing dehydration in older adults is important because the age-related decrease in the thirst mechanism makes them prone to dehydration. Having older adults drink fluids on a regular schedule will help keep them hydrated without the influence of thirst (or lack of thirst). Telling clients not to get dehydrated is important, but not the best answer because it doesn't give them the tools to prevent it from occurring. Older adults should seek attention for lacerations, but this is not as important an issue as staying hydrated. Taking medications as prescribed may or may not be related to hydration.

A nurse works at a community center for older adults. What self-management measure can the nurse teach the clients to prevent shock? a. Do not get dehydrated in warm weather. b. Drink fluids on a regular schedule. c. Seek attention for any lacerations. d. Take medications as prescribed.

ANS: B Room air is 21% oxygen.

A nursing student caring for a client removes the client's oxygen as prescribed. The client is now breathing what percentage of oxygen in the room air? a. 14% b. 21% c. 28% d. 31%

ANS: C Impaired ability to cough up mucus caused by weakness or very thick secretions indicates a need for suctioning when you know the patient has pneumonia.

A patient is admitted with severe lobar pneumonia. Which of the following assessment findings would indicate that the patient needs airway suctioning? a. Coughing up thick sputum only occasionally b. Coughing up thin, watery sputum easily after nebulization c. Decreased independent ability to cough d. Lung sounds clear only after coughing

ANS: B The ABG results will indicate the acid-base balance of the arterial blood and the partial pressure of oxygen and carbon dioxide. The ABG does not reveal the ratio of hemoglobin and hematocrit, the adequacy of oxygen transport to the cells, or the presence of a pulmonary embolus.

A patient is having the arterial blood gas (ABG) measured. What would the nurse identify as the parameters to be evaluated by this test? a. Ratio of hemoglobin and hematocrit b. Status of acid-base balance in arterial blood c. Adequacy of oxygen transport d. Presence of a pulmonary embolus

ANS: B Methotrexate use can lead to folic acid deficiency. Supplementation with oral folic acid supplements is the usual treatment. The other nutrients would not correct folic acid deficiency, although they would be used to treat other types of anemia.

A patient who is receiving methotrexate for severe rheumatoid arthritis develops a megaloblastic anemia. The nurse will anticipate teaching the patient about increasing oral intake of a. iron. b. folic acid. c. cobalamin (vitamin B12). d. ascorbic acid (vitamin C).

A nurse is providing care to a client with an ECF volume deficit. The nurse suspects that the deficit involves a decrease in vascular volume based on which finding? Select all that apply. a. orthostatic hypotension b. dry mucous membranes c. decreased urine output d. slow-filling peripheral veins e. poor skin turgor

A,C,D The signs and symptoms of an ECF volume deficit reflect decreases in fluid volume in the vascular and interstitial spaces. The signs and symptoms of a decrease in vascular volume include orthostatic or postural changes in pulse rate and blood pressure (i.e., an increase in pulse rate and decrease in blood pressure when the person moves from a lying to a standing position); weak, rapid pulse; decreased urine output; and slow-filling peripheral veins. The signs and symptoms of decreased interstitial volume include dry mucous membranes and poor skin turgor.

Which electrolyte imbalance should be anticipated and monitored in a client with hyperphosphatemia? a. Hypernatremia b. Hypokalemia c. Hypocalcemia d. Hypermagnesemia

C. Hypocalcemia Phosphorus and calcium have an inverse or reciprocal relationship. When one is increased, the other is usually decreased. Therefore, a client with hyperphosphatemia should be monitored for hypocalcemia. Hyperphosphatemia does not cause hypernatremia, hypokalemia, or hypermagnesemia.

The RN is assessing a 70-year-old client admitted to the unit with severe dehydration. Which finding requires immediate intervention by the nurse? a. Client behavior that changes from anxious and restless to lethargic and confused b. Deep furrows on the surface of the tongue c. Poor skin turgor with tenting remaining for 2 minutes after the skin is pinched d. Urine output of 950 mL for the past 24 hours

A. Client behavior that changes from anxious and restless to lethargic and confused The client's change in level of consciousness from anxious and restless to lethargic and confused suggests poor cerebral blood flow, or shrinkage or swelling of brain cells caused by fluid shifts within the brain cells. These changes indicate a need for immediate intervention to prevent further damage to cerebral function. Deep furrows on the surface of the tongue, poor skin turgor, and low urine output are all caused by the fluid volume deficit, but do not indicate complications of dehydration that are immediately life-threatening.

Which drug therapies might be used to manage symptoms of hypocalcemia? Select all that apply. a. Magnesium sulfate b. Calcium chloride c. Potassium chloride (K-Dur) d. Vitamin D (Calcitrol) e. Zinc sulfate (Zinc 220) f. Vitamin E (alpha tocopherol)

A. Magnesium sulfate B. Calcium chloride D. Vitamin D (Calcitrol) Magnesium sulfate may be used to manage neuromuscular symptoms of hypocalcemia. Calcium supplements are given to restore serum calcium levels. Vitamin D enhances the absorption of oral calcium. Potassium, zinc, and vitamin E are not indicated for the management of hypocalcemia.

The nurse is caring for a client who had a parathyroidectomy. Upon evaluation of the client's laboratory studies, the nurse would expect to see imbalances in which electrolytes related to the removal of the parathyroid gland? a. calcium and phosphorus b. potassium and sodium c. chloride and magnesium d. potassium and chloride

A. calcium and phosphorus The parathyroid gland secretes parathyroid hormone, which regulates the level of calcium and phosphorus. Removal of the parathyroid gland will cause calcium and phosphorus imbalances.

A female patient complains of a "scab that just won't heal" under her left breast. During your conversation, she also mentions chronic fatigue, loss of appetite, and slight cough, attributed to allergies. What are the nurse's next steps? a. Continue to conduct a symptom analysis to better understand the patient's symptoms and concerns. b. End the appointment and tell the patient to use skin protection during sun exposure. c. Suggest further testing with a cancer specialist and provide the appropriate literature. d. Tell her to put a bandage on the scab and set a follow-up appointment in one week.

ANS: A A comprehensive health history is vital to treating and caring for the patient. Often times, symptoms are vague. The nurse should conduct a symptom analysis to gather as much information as possible. Questions should address the duration of the symptoms and include the location, characteristics, aggravating and relief factors, and any treatments taken thus far.

The nurse is teaching a 47-year-old woman about recommended screening practices for breast cancer. Which statement by the client indicates understanding of the nurse's instructions? a. "My mother and grandmother had breast cancer, so I am at risk." b. "I get a mammogram every 2 years since I turned 30." c. "A clinical breast examination is performed every month since I turned 40." d. "A computed tomography (CT) scan will be done every year after I turn 50."

ANS: A A strong family history of breast cancer indicates a risk for breast cancer. Annual rather than biannual screening may be indicated for a strong family history. The client may perform a self-breast examination monthly; a clinical examination by a health care provider is indicated annually. An annual mammogram is performed after age 40 or in younger clients with a strong family history.

The nurse is teaching the 47-year-old female client about recommended screening practices for breast cancer. Which statement by the client indicates understanding of the nurse's instructions? a. "My mother and grandmother had breast cancer, so I am at risk." b. "I get a mammography every 2 years since I turned 30." c. "A clinical breast examination is performed every month since I turned 40." d. "A CT scan will be done every year after I turn 50."

ANS: A A strong family history of breast cancer indicates a risk for breast cancer. Annual screening may be indicated for a strong family history. The client may perform a self-breast examination monthly; a clinical examination by a health care provider is indicated annually. An annual mammography is performed after age 40 or in younger clients with a strong family history.

A client has been placed on a ventilator. The physician has ordered that the ventilator be set to deliver a respiratory rate set of 28 breaths/min. The nurse questions the order, citing concerns about which acid-base problem? a. Acid deficit: alkalosis b. Base excess: alkalosis c. Acid excess: acidosis d. Base deficit: acidosis

ANS: A A ventilator set at too high a ventilation rate and/or too high a tidal volume will cause the client to lose too much carbon dioxide, leading to an acid deficit and respiratory alkalosis.

Which diet would the nurse recommend to the mother of a child who is having mild diarrhea? a. Rice, potatoes, yogurt, cereal, and cooked carrots b. Bananas, rice, applesauce, and toast c. Apple juice, hamburger, and salad d. Whatever the child would like to eat

ANS: A Bland but nutritious foods including complex carbohydrates (rice, wheat, potatoes, cereals), yogurt, cooked vegetables, and lean meats are recommended to prevent dehydration and hasten recovery. These foods used to be recommended for diarrhea (BRAT diet). These foods are easily tolerated, but the BRAT diet is low in energy, density, fat, and protein. Fatty foods, spicy foods, and foods high in simple sugars should be avoided. The child should be offered foods he or she likes but should not be encouraged to eat fatty foods, spicy foods, and foods high in simple sugars.

The nurse caring for oncology clients knows that which form of metastasis is the most common? a. Bloodborne b. Direct invasion c. Lymphatic spread d. Via bone marrow

ANS: A Bloodborne metastasis is the most common way for cancer to metastasize. Direct invasion and lymphatic spread are other methods. Bone marrow is not a medium in which cancer spreads, although cancer can occur in the bone marrow.

Which description about genetic screening is correct? a. It identifies genetic risk for specific cancers. b. The tests are performed on cerebrospinal fluid samples. c. A positive test diagnoses cancer in the client. d. The test results are shared with the client's family.

ANS: A Genetic screening helps to identify if a client has a genetic risk for specific cancers. The tests are performed only on blood samples. A positive test indicates the presence of a mutated gene that may cause cancer; however, the cancer may never develop. The test result is not shared with the client's family; it is the client's privilege to maintain secrecy or disclose the contents of the test to the family.

The nurse explains to a client that which risk factor of those listed most likely contributed to the client's primary liver carcinoma? a. Infection with hepatitis B virus b. Consuming a diet high in animal fat c. Exposure to radon d. Familial polyposis

ANS: A Hepatitis B and C are risk factors for primary liver cancer. Alcohol abuse is also a risk factor for the development of liver cancer. Consuming a diet high in animal fat may predispose a person to colon or breast cancer. Exposure to radon is a risk factor for lung cancer. Familial polyposis is a risk factor for colorectal cancer.

While planning care for a patient experiencing fatigue due to chemotherapy, which of the following is the most appropriate nursing intervention? a. Prioritization and administration of nursing care throughout the day b. Completing all nursing care in the morning so the patient can rest the remainder of the day c. Completing all nursing care in the evening when the patient is more rested d. Limiting visitors, thus promoting the maximal amount of hours for sleep

ANS: A Pacing activities throughout the day conserves energy, and nursing care should be paced as well. Fatigue is a common side effect of cancer and treatment; and while adequate sleep is important, an increase in the number of hours slept will not resolve the fatigue. Restriction of visitors does not promote healthy coping and can result in feelings of isolation.

The nurse manager in a long-term care facility is developing a plan for primary and secondary prevention of colorectal cancer. Which tasks associated with the screening plan will be delegated to nursing assistants within the facility? a. Testing of stool specimens for occult blood b. Teaching about the importance of dietary fiber c. Referring clients for colonoscopy procedures d. Giving vitamin and mineral supplements

ANS: A Testing of stool specimens for occult blood is done according to a standardized protocol and can be delegated to nursing assistants. Client education is within the scope of practice of the RN, not of the LPN or nursing assistant. Referral for further care is best performed by the RN. Administration of medications is beyond the nursing assistant's scope of practice and should be done by licensed nursing personnel.

Which of the following statements is essential when teaching a patient who has received an injection of iodine-131? a. "Do not share a toilet with anyone else for 3 days." b. "You need to save all your urine for the next 7 days." c. "No special precautions are needed, because this is a weak type of radiation." d. "You need to avoid contact with everyone except family members until the radiation device is removed."

ANS: A The radiation source is an unsealed isotope that is eliminated from the body mainly through urine and feces. This material is radioactive for about 48 hours after instillation. The patient should not share a toilet with others for 3 days to ensure the isotope has been completely eliminated and is no longer radioactive. Saving the urine is not necessary. Contact should be avoided with anyone who may be ill or immunocompromised. Patients are instructed to avoid crowded areas but isolation is not necessary.

A nurse is caring for a client who has chronic emphysema and is receiving oxygen therapy at 6 L/min via nasal cannula. The following clinical data are available: Arterial Blood Gases Vital Signs pH = 7.28 Pulse rate = 96 beats/min PaO2 = 85 mm Hg Blood pressure = 135/45 PaCO2 = 55 mm Hg Respiratory rate = 6 breaths/min HCO3- = 26 mEq/L O2 saturation = 88% Which action should the nurse take first? a. Notify the Rapid Response Team and provide ventilation support. b. Change the nasal cannula to a mask and reassess in 10 minutes. c. Place the client in Fowler's position if he or she is able to tolerate it. d. Decrease the flow rate of oxygen to 2 to 4 L/min, and reassess.

ANS: A The primary trigger for respiration in a client with chronic respiratory acidosis is a decreased arterial oxygen level (hypoxic drive). Oxygen therapy can inhibit respiratory efforts in this case, eventually causing respiratory arrest and death. The nurse could decrease the oxygen flow rate; eventually, this might improve the client's respiratory rate, but the priority action would be to call the Rapid Response Team whenever a client with chronic carbon dioxide retention has a respiratory rate less than 10 breaths/min. Changing the cannula to a mask does nothing to improve the client's hypoxic drive, nor would it address the client's most pressing need. Positioning will not help the client breathe at a normal rate or maintain client safety.

The nurse includes which factors in teaching regarding the typical warning signs of cancer? Select all that apply. a. Persistent constipation b. Scab present for 6 months c. Curdlike vaginal discharge d. Axillary swelling e. Headache

ANS: A, B, D Change in bowel habits, a sore throat that does not heal, and a lump or thickening in the breast or elsewhere are all potential warning signs of cancer. Curdlike vaginal discharge represents a yeast infection. Headache is not a warning sign, but may be present with multiple problems.

A nurse is planning interventions that regulate acid-base balance to ensure the pH of a client's blood remains within the normal range. Which abnormal physiologic functions may occur if the client experiences an acid-base imbalance? (Select all that apply.) a. Reduction in the function of hormones b. Fluid and electrolyte imbalances c. Increase in the function of selected enzymes d. Excitable cardiac muscle membranes e. Increase in the effectiveness of many drugs

ANS: A, B, D Acid-base imbalances interfere with normal physiology, including reducing the function of hormones and enzymes, causing fluid and electrolyte imbalances, making heart membranes more excitable, and decreasing the effectiveness of many drugs.

In the client with alkalosis, the nurse assesses for which clinical manifestations? (Select all that apply.) a. Positive Chvostek's sign b. Positive Trousseau's sign c. Hyporeflexia d. Bradycardia e. Elevated blood pressure f. Elevated urinary output

ANS: A, B, D The client with alkalosis demonstrates signs of hypocalcemia and decreased heart rate. Many symptoms are the result of low calcium levels (hypocalcemia) and low potassium levels (hypokalemia), which usually occur with alkalosis. These problems change the function of the nervous, neuromuscular, cardiac, and respiratory systems.

Which assessment findings indicate to the nurse that a child has excess fluid volume? Select all that apply. a. Weight gain b. Decreased blood pressure c. Moist breath sounds d. Poor skin turgor e. Rapid bounding pulse

ANS: A, C, E A child with fluid volume excess will have a weight gain, moist breath sounds due to the excess fluid in the pulmonary system, and a rapid bounding pulse. Other signs seen with fluid volume excess are increased blood pressure, edema, and fatigue. Decreased blood pressure and poor skin turgor are signs of fluid volume deficit.

Which dietary modifications can the nurse recommend to a client for preventing cancer development? Select all that apply. a. Increasing broccoli intake b. Increasing red meat consumption c. Consuming more dietary bran d. Eating more sausage and bacon e. Restricting alcohol consumption to less than 2 drinks per day

ANS: A,C,E The dietary modifications that can help in reducing cancer development include eating broccoli, cauliflower, sprouts, cabbage, and dietary bran. Restricting alcohol consumption to less than 2 drinks per day also reduces the risk of developing cancer. Consuming animal fats like red meat, sausage, and bacon increase the risk for developing cancer, so their consumption should be restricted.

The nurse is assessing a client with lung cancer. Which symptom does the nurse anticipate finding? a. Easy bruising b. Dyspnea c. Night sweats d. Chest wound

ANS: B Dyspnea is a sign of lung cancer, as are cough, hoarseness, shortness of breath (SOB), bloody sputum, arm or chest pain, and dysphagia. Easy bruising is a nonspecific finding. Night sweats is a symptom of the lymphomas. A chest wound is not specific to lung cancer.

The nurse reviews the chart of a client admitted with a diagnosis of glioblastoma with a T1NXM0 classification. Which explanation does the nurse offer when the client asks what the terminology means? a. "Two lymph nodes are involved in this tumor of the glial cells, and another tumor is present." b. "The brain tumor measures about 1 to 2 cm and shows no regional lymph nodes and no distant metastasis." c. "This type of tumor in the brain is small with some lymph node involvement; another tumor is present somewhere else in your body." d. "Glioma means this tumor is benign, so I will have to ask your health care provider the reason for the chemotherapy and radiation."

ANS: B T1 means that the tumor is increasing in size to about 2 cm, and that no regional lymph nodes are present in the brain. M0 means that no distant metastasis has occurred. NX means that no regional lymph nodes can be assessed. A glioma is a benign tumor of the brain, but the client is diagnosed with a glioblastoma, which means a malignant tumor of the glial cells of the brain.

A nurse is assessing a client who has acute pancreatitis and is at risk for an acid-base imbalance. For which manifestation of this acid-base imbalance should the nurse assess? a. Agitation b. Kussmaul respirations c. Seizures d. Positive Chvostek's sign

ANS: B The pancreas is a major site of bicarbonate production. Pancreatitis can cause a relative metabolic acidosis through underproduction of bicarbonate ions. Manifestations of acidosis include lethargy and Kussmaul respirations. Agitation, seizures, and a positive Chvostek's sign are manifestations of the electrolyte imbalances that accompany alkalosis.

A nurse is participating in primary prevention efforts directed against cancer. In which activities is this nurse most likely to engage? (Select all that apply.) a. Demonstrating breast self-examination methods to women b. Instructing people on the use of chemoprevention c. Providing vaccinations against certain cancers d. Screening teenage girls for cervical cancer e. Teaching teens the dangers of tanning booths

ANS: B, C, E Primary prevention aims to prevent the occurrence of a disease or disorder, in this case cancer. Secondary prevention includes screening and early diagnosis. Primary prevention activities include teaching people about chemoprevention, providing approved vaccinations to prevent cancer, and teaching teens the dangers of tanning beds. Breast examinations and screening for cervical cancer are secondary prevention methods.

What are the common cancers related to tobacco use? Select all that apply. a. Cardiac cancer b. Lung cancer c. Cancer of the tongue d. Skin cancer e. Cancer of the larynx

ANS: B, C, E Organs exposed to the carcinogens in tobacco are the most likely to develop cancer. Oral cancer is also a risk with "smokeless" tobacco. The heart does not contain cells that divide; therefore, cardiac cancer is unlikely. Skin cancer generally is related to repeated sun and other UV exposure such as that found with tanning beds.

The TNM classification report of a client with lung cancer is TisN0Mx. How is this classification interpreted? Select all that apply. a. No metastasis. b. Carcinoma in situ. c. No evidence of the primary tumor. d. Involvement of regional lymph nodes. e. Presence of metastasis cannot be assessed.

ANS: B, E Staging determines the exact location of the cancer and its degree of metastasis at diagnosis. The acronym TNM refers to "Tumor, Nodes, Metastasis." Per the TNM classification, Tis stands for carcinoma in situ. Mx means that the presence of distant metastasis cannot be assessed. The absence of regional lymph node metastasis is indicated by N0.

The nurse is teaching a client about cancer warning signs. Which signs/symptoms does the nurse include? Select all that apply. a. A sore that heals quickly b. Unusual bleeding and discharge c. Change in bowel or bladder habits d. Nagging cough or a hoarse voice e. Long-lasting warts without any observable change

ANS: B,C,E Unusual bleeding and discharge can be a warning sign for the onset of cancer. A change in bowel or bladder habits may be indicative of a malignancy in the intestine and urinary bladder, respectively. Nagging cough or hoarseness can indicate a malignancy of the respiratory airways. A sore that heals quickly indicates an effective wound-healing mechanism, while a non-healing sore can be a cancer warning sign. Long-lasting moles or warts with observable changes may be a warning sign for cancer.

nurse is learning about the different types of cancers. Which cancer has the highest incidence among men? a. Lung cancer b. Colon cancer c. Prostate cancer d. Thyroid cancer

ANS: C Among all the cancers in men, prostate cancer has the highest incidence (29%). Lung cancer has the highest death rate among men (29%). The incidence of colon cancer in males is 9%. Thyroid cancer is more common in women than men.

In the client with hypoventilation, which change in arterial blood gases does the nurse evaluate to determine whether treatment measures are being effective? a. Decreased arterial blood pH b. Decreased arterial blood carbon dioxide c. Increased arterial blood bicarbonate d. Increased arterial blood oxygen

ANS: C Because kidneys regulate pH by controlling bicarbonate concentration and the lungs regulate pH by controlling carbon dioxide loss, loss of one function can be at least partially compensated for by the other function. When pulmonary function is decreased, so that adequate amounts of carbon dioxide are not excreted, the pH falls, stimulating the kidneys to reabsorb more bicarbonate to balance the increased acid production.

A patient diagnosed with benign lipoma is concerned about the tumor spreading to other parts of the body. Which facts should the nurse include when teaching the patient about benign tumors? a. Benign tumors are poorly differentiated. b. Benign tumors have high recurrence rate. c. Benign tumors are not capable of metastasis. d. Benign tumors have moderate vascularity.

ANS: C Benign tumors are not metastatic and not capable of spreading from one organ to another. Benign tumors are normally differentiated, have low vascularity, and their recurrence is rare.

It is important for the nurse to teach the client which of the following about metformin (Glucophage)? a. It may cause constipation b. It should be taken at night c. It should be taken with meals d. It may increase the effects of aspirin

ANS: C Metformin (Glucophage) is administered with meals to minimize gastrointestinal effects. These adverse effects are abdominal bloating, diarrhea, nausea, vomiting, and an unpleasant metallic taste. Metformin interacts with alcohol and cimetidine and is contraindicated in heart failure and liver disease and in clients with compromised renal function

A client who is scheduled to undergo radiation for prostate cancer is admitted to the hospital by the nurse. Which statement by the client is most important to communicate to the health care provider? a. "I am allergic to iodine." b. "My urinary stream is very weak." c. "My legs are numb and weak." d. "I am incontinent when I cough."

ANS: C Numbness and weakness should be reported to the physician because paralysis caused by spinal cord compression can occur. Prostate cancer may frequently metastasize to the bone, specifically the spine. Allergy to iodine should be reported when contrast media will be used, but dye is not used in radiation therapy. A weak urinary stream and incontinence are common clinical manifestations of prostate cancer. Incontinence associated with coughing is typical of stress incontinence and is not a complication of cancer.

A 33-year-old patient has recently been diagnosed with stage II cervical cancer. The nurse would understand that the patient's cancer a. Is in situ. b. Has metastasized. c. Has spread locally. d. Has spread extensively.

ANS: C Stage II cancer is associated with local spread. Stage 0 denotes cancer in situ; stage III denotes extensive regional spread, and stage V denotes metastasis.

A nurse has taught a client about dietary changes that can reduce the chances of developing cancer. What statement by the client indicates the nurse needs to provide additional teaching? a. "Foods high in vitamin A and vitamin C are important." b. "I'll have to cut down on the amount of bacon I eat." c. "I'm so glad I don't have to give up my juicy steaks." d. "Vegetables, fruit, and high-fiber grains are important."

ANS: C To decrease the risk of developing cancer, one should cut down on the consumption of red meats and animal fat. The other statements are correct.

What are the features of benign tumor cells? Select all that apply. a. They exhibit aneuploidy. b. They have migratory tendency. c. They have orderly growth patterns. d. They grow by hyperplastic expansion. e. They are encapsulated by fibrous connective tissue.

ANS: C, D, E Benign tumor cells have orderly growth patterns. They grow by hyperplastic expansion, causing the tissue to increase in size by increasing the number of cells. Growth may continue beyond an appropriate time or occur in the wrong place, but the growth rate is normal. They continue to make fibronectin and adhere to each other tightly. They are encapsulated by fibrous connective tissue which prevents them from migrating. Abnormal chromosomes or aneuploidy are common in cancer cells as they become more malignant. Benign cells have normal chromosomes. Cancer cells have a migratory tendency as they are not bound to each other with fibronectin.

When the Type 1 diabetic patient asks why his 7 AM insulin has been changed from NPH insulin to 70/30 premixed insulin, the nurse explains that 70/30 insulin: a. is absorbed more rapidly into the bloodstream. b. has no peak action time and lasts all day. c. makes insulin administration easier and safer. d. give a bolus of rapid-acting insulin to prevent hyperglycemia after breakfast. the morning meal.

ANS: D 70/30 insulin is 30% rapid-acting and 70% intermediate-acting insulin. The rapid action of the 7 AM premixed insulin prevents hyperglycemia after the morning meal.

While the nurse is obtaining the health history of a 75-year-old female patient, which of the following has the greatest implication for the development of cancer? a. Being a 75-year-old woman b. Family history of hypertension c. Cigarette smoking as a teenager d. Advancing age

ANS: D According to the American Cancer Society, 2007, the most important risk factor for cancer development is advancing age.

A 72-year-old client recovering from lung cancer surgery asks the nurse to explain how she developed cancer when she has never smoked. Which factor may explain the possible cause? a. A diagnosis of diabetes treated with insulin and diet b. An exercise regimen of jogging 3 miles four times a week c. A history of cardiac disease d. Advancing age

ANS: D Advancing age is the single most important risk factor for cancer. As a person ages, immune protection decreases. Diabetes is not known to cause lung cancer. Regular exercise is not a risk factor for lung cancer, nor does having cardiac disease predispose a person to lung cancer.

A student nurse asks the nursing instructor what "apoptosis" means. What response by the instructor is best? a. Growth by cells enlarging b. Having the normal number of chromosomes c. Inhibition of cell growth d. Programmed cell death

ANS: D Apoptosis is programmed cell death. With this characteristic, organs and tissues function with cells that are at their peak of performance. Growth by cells enlarging is hyperplasia. Having the normal number of chromosomes is euploidy. Inhibition of cell growth is contact inhibition.

Which of the following regimens offers the best blood glucose control for persons with type 1 diabetes? a. A single anti-diabetes drugs b. Once daily insulin injections c. A combination of oral anti-diabetic medications d. Three or four injections per day of different types of insulin.

ANS: D Because persons with type 1 diabetes do not produce insulin, they require insulin and cannot be treated with oral anti-diabetic drugs. Several injections of insulin per day, calibrated to respond to measured blood glucose levels, offer the best blood glucose control and may prevent or postpone the retinal, renal, and neurological complications of diabetes.

A client is diagnosed with familial polyposis. Which cancer may the client be at risk for developing in the future? a. Meningioma b. Breast cancer c. Gonadal cancer d. Colorectal cancer

ANS: D Clients with familial polyposis are at risk for developing colorectal cancer due to a genetic predisposition. Meningioma and gonadal cancer may eventually occur in clients with Turner's syndrome. Breast cancers are often inherited disorders or occur due to familial clustering.

The nurse is caring for a patient who received a bone marrow transplant 10 days ago. The nurse would monitor for which of the following clinical manifestations that could indicate a potentially life-threatening situation? a. Mucositis b. Confusion c. Depression d. Mild temperature elevation

ANS: D During the first 100 days after a bone marrow transplant, patients are at high risk for life-threatening infections. The earliest sign of infection in an immunosuppressed patient can be a mild fever. Mucositis, confusion, and depression are possible clinical manifestations but are representative of less life-threatening complications.

In caring for a patient following lobectomy for lung cancer, which of the following should the nurse include in the plan of care? a. Position the patient on the operative side only. b. Avoid administering narcotic pain medications. c. Keep the patient on strict bed rest. d. Instruct the patient to cough and deep breathe.

ANS: D Postoperative deep breathing and coughing is important to promote oxygenation and clearing of secretions. Pain medications will be given to lessen pain and allow for deep breathing and coughing. Strict bed rest is not instituted, because early ambulation will help lessen postoperative complications such as deep vein thrombosis. Prolonged lying on the operative side is avoided.

A client is in the oncology clinic for a first visit since being diagnosed with cancer. The nurse reads in the client's chart that the cancer classification is TISN0M0. What does the nurse conclude about this client's cancer? a. The primary site of the cancer cannot be determined. b. Regional lymph nodes could not be assessed. c. There are multiple lymph nodes involved already. d. There are no distant metastases noted in the report.

ANS: D TIS stands for carcinoma in situ; N0 stands for no regional lymph node metastasis; and M0 stands for no distant metastasis.

A client has a glioblastoma. The nurse begins to plan care for this client with which type of cancer? a. Liver b. Smooth muscle c. Fatty tissue d. Brain

ANS: D The prefix "glio-" is used when cancers of the brain are named. The prefix "hepato-" is included when cancers of the liver are named. The prefix "leiomyo-" is included when cancers of smooth muscle are named. The prefix "lipo-" is included when cancers of fat or adipose tissue are named.

Which statement about carcinogenesis is accurate? a. An initiated cell will always become clinical cancer. b. Cancer becomes a health problem once it is 1 cm in size. c. Normal hormones and proteins do not promote cancer growth. d. Tumor cells need to develop their own blood supply.

ANS: D Tumors need to develop their own blood supply through a process called angiogenesis. An initiated cell needs a promoter to continue its malignant path. Normal hormones and proteins in the body can act as promoters. A 1-cm tumor is a detectable size, but other events have to occur for it to become a health problem.

The nurse assesses the client with which condition most carefully for the risk of developing acute respiratory acidosis? a. Allergic rhinitis and sinusitis on sulfa antibiotics b. Type 1 diabetes and urinary tract infection c. Emphysema and undergoing nasogastric (NG) tube suctioning d. On patient-controlled analgesia after abdominal surgery

ANS: D Respiratory acidosis often occurs as the result of underventilation. The client who is taking narcotics, especially IV narcotics, is at risk for respiratory depression. The client may also be breathing more shallowly than usual to prevent pain. This gives the client two risk factors for developing hypoventilation and subsequent respiratory acidosis. None of the other clients are at risk for ineffective ventilation.

A nurse is caring for a client who is experiencing moderate metabolic alkalosis. Which action should the nurse take? a. Monitor daily hemoglobin and hematocrit values. b. Administer furosemide (Lasix) intravenously. c. Encourage the client to take deep breaths. d. Teach the client fall prevention measures.

ANS: D The priority nursing care for a client who is experiencing moderate metabolic alkalosis is providing client safety. Clients with metabolic alkalosis have muscle weakness and are at risk for falling. The other nursing interventions are not appropriate for metabolic alkalosis.

ANS: C A, D, E, and K are the fat-soluble vitamins that need to be supplemented.

Absorption of fat-soluble vitamins is decreased in children with cystic fibrosis; therefore supplementation of which vitamins is necessary? A. C, D B. A, E, K C. A, D, E, K D. C, folic acid

ANS: A Children with cystic fibrosis have thick mucus gland secretions. The viscous secretions obstruct small passages in organs such as the pancreas.

Cystic fibrosis may affect single or multiple systems of the body. What is the primary factor responsible for possible multiple clinical manifestations? a. Mechanical obstruction caused by increased viscosity of mucous gland secretions. b. Atrophic changes in mucosal wall of intestines. c. Hypoactivity of the autonomic nervous system. d. Hyperactivity of sweat glands.

ANS: D Prostaglandin E1 is used to maintain a patent ductus arteriosus, thus increasing pulmonary blood flow.

For what reason might a newborn infant with a cardiac defect, such as coarctation of the aorta, that results in a right-to-left shunt receive prostaglandin E1? a. To decrease inflammation b. To control pain c. To decrease respirations d. To improve oxygenation

ANS: D Respiratory infections can trigger an asthmatic attack. Annual influenza vaccine is recommended. All respiratory equipment should be kept clean.

One of the goals for children with asthma is to prevent respiratory infections. Why is this goal so important? a. Respiratory infections encourage exercise-induced asthma. b. Allergen sensitivity is increased in the presence of infection. c. Asthma medication becomes less effective when a respiratory infection is present. d. Respiratory infections can trigger an episode or aggravate the asthmatic state.

ANS: B Hypovolemic shock can be caused by dehydration. A client who has bulimia is at highest risk for dehydration owing to excessive vomiting. Basketball, smoking, and traveling do not put the client at risk for hypovolemic shock.

The emergency department nurse is triaging clients. Which client does the nurse assess most carefully for hypovolemic shock? a. 15-year-old adolescent who plays high school basketball b. 24-year-old computer specialist who has bulimia c. 48-year-old truck driver who has a 40-pack-year history of smoking d. 62-year-old business executive who travels frequently

ANS: C In this irreversible phase, therapy is not effective in saving the client's life, even if the cause of shock is corrected and mean arterial pressure temporarily returns to normal. A discussion on palliative care should be considered. Rehabilitation or returning home is unlikely. The client with sustained tissue hypoxia is not a candidate for organ transplantation.

The nurse is caring for a client in the refractory stage of cardiogenic shock. Which intervention does the nurse consider? a. Admission to rehabilitation hospital for ambulatory retraining b. Collaboration with home care agency for return to home c. Discussion with family and provider regarding palliative care d. Enrollment in a cardiac transplantation program

ANS: B Cultures must be taken to identify the organism for more targeted antibiotic treatment before antibiotics are administered. Antibiotics are not administered until after all cultures are taken. A signed consent is not needed for medication administration. Monitoring the client's vital signs is important, but the antibiotic must be administered within 1 to 3 hours; timing is essential.

The nurse plans to administer an antibiotic to a client newly admitted with septic shock. What action does the nurse take first? a. Administer the antibiotic immediately b. Ensure that blood cultures were drawn c. Obtain signature for informed consent d. Take the client's vital signs

ANS: C SCA is inherited as an autosomal recessive disorder. In this inheritance pattern, there is a 25% chance that each subsequent child will have the disorder. SCA is an inherited hemoglobinopathy. In autosomal recessive disorders, there is a chance that 25% of the children will not have either SCA or sickle cell trait. There is a chance that 50% of the siblings will have sickle cell trait.

The parents of a child with sickle cell anemia (SCA) are concerned about subsequent children having the disease. Which response by the nurse is most accurate? a. "SCA is not inherited." b. "All siblings will have SCA." c. "There is a 25% chance of a sibling having SCA." d. "There is a 50% chance of a sibling having SCA."

ANS: B Thalassemia is a group of autosomal recessive diseases that involve inadequate production of normal hemoglobin. Hemolysis also occurs in thalassemia, but insufficient production of normal hemoglobin is the predominant problem. Erythropoietin deficiency is associated with a renal disorder, and S-shaped hemoglobin is associated with sickle cell disease.

The primary pathophysiology underlying thalassemia is a. erythropoietin deficiency. b. abnormal hemoglobin synthesis. c. autoimmunity. d. S-shaped hemoglobin.

ANS: A Parietal cell function can be assssed with a Schilling test. After radioactive B12 is administered to the patient, the amount of cobalamin excreted in the urine is measured. An individual who cannot absorb cobalamin excretes only a small amount of this radioactive form.

The Schilling test for pernicious anemia involves a. Administration of radioactive B12 and measuring its excretion in the urine. b. Blood cultures for organism identification. c. The measurement of serum iron. d. The administration of iron and blood assessment of total iron binding in 24 hours.

ANS: B Vitamin C aids in the absorption of iron, whereas food and milk interfere with the absorption of iron.

The nurse would instruct the parent to give ferrous sulfate drops to the child: a. with milk. b. with orange juice. c. with water. d. on a full stomach.

ANS: D. By humidifying the inspired air, the membranes inflamed by the infection and dry air are soothed.

Why is a humidified atmosphere recommended for a young child with an upper respiratory tract infection? a. It liquefies secretions. b. It improves oxygenation. c. It promotes ventilation. d. It is soothing to inflamed mucous membrane.

Which client has more extracellular fluid? a) Adolescent man b) Newborn c) Female school-age child d) Adult woman

b) Newborn

ANS: A Because clients in sickle cell crisis are often dehydrated, the fluid of choice is a hypotonic solution such as 0.45% normal saline. 0.9% normal saline and lactated Ringer's solution are isotonic. D50 is hypertonic and not used for hydration.

A client in sickle cell crisis is dehydrated and in the emergency department. The nurse plans to start an IV. Which fluid choice is best? a. 0.45% normal saline b. 0.9% normal saline c. Dextrose 50% (D50) d. Lactated Ringer's solution

ANS: C An incentive spirometer is used to encourage deep breathing to inflate alveoli and open pores of Kohn. The rest are used to treat atelectasis and increased mucus production.

Which nursing intervention is appropriate for preventing atelectasis in the postoperative patient? a. Postural drainage b. Chest percussion c. Incentive spirometer d. Suctioning

ANS: A Air trapping is not present in all cases of impaired gas exchange. Delayed development does not occur unless the condition is chronic or acutely damaging. The early phase of impaired gas exchange does not cause injury or dehydration, although fatigue can occur.

A child in the early stages of impaired gas exchange will often have which diagnosis as well? a. Anxiety related to hypoxia b. Fatigue related to air trapping c. Injury related to fatigue and dehydration d. Delayed Development related to hypoxia

ANS: B Airway is the priority, followed by breathing and circulation (IVs and direct pressure). Obtaining consent is done by the physician.

A client arrives in the emergency department after being in a car crash with fatalities. The client has a nearly amputated leg that is bleeding profusely. What action by the nurse takes priority? a. Apply direct pressure to the bleeding. b. Ensure the client has a patent airway. c. Obtain consent for emergency surgery. d. Start two large-bore IV catheters.

A client who has just been notified that the breast biopsy indicates a malignancy tells the nurse, "I just don't know how this could have happened to me." Which of these responses by the nurse will be most appropriate? a. "Tell me what you mean when you say you don't know how this could have happened to you." b. "Do you have a family history that might make you more likely to develop breast cancer?" c. "Would you like me to help you find more information about how breast cancer develops?" d. "Many risk factors for breast cancer have been identified, so it is difficult to determine what might have caused it."

ANS: A The client's statement may indicate shock and denial or a request for more information. To provide appropriate care, further assessment is needed about the client's psychosocial status. The first action by the nurse in this situation is to obtain more data by asking open-ended questions.

Which response is an example of compensation for an acid-base imbalance? a. Increase in the rate and depth of respirations when exercising b. Increased urinary output when blood pressure increases during exercise c. Increased thirst when spending time in an excessively dry environment d. Increased release of acids from kidneys during exacerbation of chronic obstructive pulmonary disease (COPD)

ANS: A The respiratory system increases its activity by blowing off excess carbon dioxide. This occurs as a result of the occurrence of lactic acidosis in skeletal muscle, when blood flow and oxygenation are insufficient to meet the increased demand for oxygen (oxygen debt) created during increased skeletal muscle metabolism. The other three options are not compensatory mechanisms for acid-base imbalances.

A client has the following arterial blood gases (ABGs): pH 7.30, HCO3- 22 mEq/L, PCO2 55 mm Hg, PO2 86 mm Hg. Which intervention by the nurse takes priority? a. Assessing the airway b. Administering bronchodilators c. Administering mucolytics d. Providing oxygen

ANS: A All interventions are important for clients with respiratory acidosis; this is indicated by the ABGs. However, the priority is assessing and maintaining an airway. Without a patent airway, other interventions will not be helpful.

In a client with less than the normal amount of bicarbonate in the blood and other extracellular fluids, what response does the nurse anticipate? a. Increased risk for acidosis b. Decreased risk for acidosis c. Increased risk for alkalosis d. Decreased risk for alkalosis

ANS: A Bicarbonate (H2CO3-) is a weak base with an overall negative charge. When hydrogen ions are present in slight or mild excess (mild acidosis), bicarbonate can buffer or absorb the excess hydrogen ions, reducing the hydrogen ion concentration and bringing the pH back up to normal. If the total body bicarbonate concentration is low, especially in the blood, the action of buffering or absorbing excess hydrogen ions is reduced, and the person is at increased risk for acidosis.

Which statement made by a client with type 2 diabetes taking nateglinide (Starlix) indicates understanding of this therapy? a. "I'll take this medicine with my meals." b. "I'll take this medicine right before I eat." c. "I'll take this medicine just before I go to bed." d. "I'll take this medicine when I wake up in the morning."

ANS: B Nateglinide is a meglitinide that is designed to increase meal-related insulin secretion. It should be taken just before a meal. The other options are incorrect.

A nurse assesses a client who is experiencing an acid-base imbalance. The client's arterial blood gas values are pH 7.34, PaO2 88 mm Hg, PaCO2 38 mm Hg, and HCO3- 19 mEq/L. Which assessment should the nurse perform first? a. Cardiac rate and rhythm b. Skin and mucous membranes c. Musculoskeletal strength d. Level of orientation

ANS: A Early cardiovascular changes for a client experiencing moderate acidosis include increased heart rate and cardiac output. As the acidosis worsens, the heart rate decreases and electrocardiographic changes will be present. Central nervous system and neuromuscular system changes do not occur with mild acidosis and should be monitored if the acidosis worsens. Skin and mucous membrane assessment is not a priority now, but will change as acidosis worsens.

The nurse reads in the medical record that a client has Kussmaul respirations. Which assessment finding is consistent with this condition? a. Deep, rapid respirations b. Respirations with an irregular pattern c. Shallow, grunting respirations d. Use of accessory muscles when breathing

ANS: A Kussmaul respirations are described as deep and rapid and are the body's attempt to compensate for acidosis by "blowing off" excess H+ in the form of carbon dioxide.

The nurse monitors the client with which condition most carefully for metabolic alkalosis? a. A critical illness receiving total parenteral nutrition b. Type 1 diabetes on once-daily insulin therapy c. Metastatic breast cancer on continuous IV morphine d. Asthma using an adrenergic agonist inhaler

ANS: A The IV fluid mixture for total parenteral nutrition (TPN) has an overall basic pH. One common substance in TPN is lactate, which is rapidly converted in the body to bicarbonate. In addition, the TPN mixture is often administered as a continuous slow infusion. A client with diabetes would be at higher risk of metabolic acidosis. The client on IV morphine is more at risk for respiratory acidosis, as is the client with asthma.

The nurse is caring for a client who has taken a large quantity of furosemide (Lasix) to promote weight loss. The nurse anticipates the finding of which acid-base imbalance? a. PaO2 of 78 mm Hg b. HCO3- of 34 mEq/L c. PaCO2 of 56 mm Hg d. pH of 7.31

ANS: B Diuretics (non-potassium-sparing), evidenced by a finding of HCO3- of 34 mEq/L, cause metabolic alkalosis. A PaO2 of 78 mm Hg demonstrates mild hypoxemia consistent with respiratory disorders, not with diuretic use. CO2 retention results from hypoventilation, which is not consistent with diuretic use. A pH of 7.31 is acidotic; diuretics promote metabolic alkalosis.

A client has just experienced a 90-second tonic-clonic seizure and has these arterial blood gas values: pH 6.88, HCO3- 22 mEq/L, PCO2 60 mm Hg, PO2 50 mm Hg. Which intervention by the nurse is most appropriate? a. Apply oxygen by mask or nasal cannula. b. Apply a paper bag over the client's nose and mouth. c. Administer 50 mL of sodium bicarbonate intravenously. d. Administer 50 mL of 20% glucose and 20 units of regular insulin.

ANS: A The client has experienced a combination of metabolic and acute respiratory acidosis through heavy skeletal muscle contractions and no gas exchange. When the seizures have stopped and the client can breathe again, the fastest way to return to acid-base balance is to administer oxygen. Sodium bicarbonate should not be administered because the client's arterial bicarbonate level is normal. Applying a paper bag over the client's nose and mouth would worsen the acidosis.

The nurse has given a client an injection of glucagon. Which action does the nurse take next? a. Apply pressure to the injection site. b. Position the client on his or her side. c. Have a padded tongue blade available. d. Elevate the head of the bed.

ANS: B Glucagon administration often induces vomiting, increasing the client's risk for aspiration. The other actions are not required.

Which client is most likely to exhibit the following ABG results: pH, 7.30; PaCO2, 49; HCO3-, 26; PaO2, 76? a. Client with kidney failure b. Client taking hydromorphone (Dilaudid) c. Client with anxiety disorder d. Client with hyperkalemia

ANS: B Hydromorphone (Dilaudid), a narcotic analgesic, can cause respiratory depression, hypoventilation, and respiratory acidosis, as this blood gas reading demonstrates. Kidney failure causes metabolic acidosis. Anxiety will cause hyperventilation and subsequent respiratory alkalosis. Although hyperkalemia can be caused by acidosis, it is not a cause of acidosis.

A nurse assesses a client with diabetes mellitus who is admitted with an acid-base imbalance. The client's arterial blood gas values are pH 7.36, PaO2 98 mm Hg, PaCO2 33 mm Hg, and HCO3- 18 mEq/L. Which manifestation should the nurse identify as an example of the client's compensation mechanism? a. Increased rate and depth of respirations b. Increased urinary output c. Increased thirst and hunger d. Increased release of acids from the kidneys

ANS: A This client has metabolic acidosis. The respiratory system compensates by increasing its activity and blowing off excess carbon dioxide. Increased urinary output, thirst, and hunger are manifestations of hyperglycemia but are not compensatory mechanisms for acid-base imbalances. The kidneys do not release acids.

A client who has type 2 diabetes is prescribed glipizide (Glucotrol). Which precautions does the nurse include in the teaching plan related to this medication? a. "Change positions slowly when you get up." b. "Avoid taking nonsteroidal anti-inflammatory drugs." c. "If you miss a dose of this drug, you can double the next dose." d. "Discontinue the medication if you develop an infection."

ANS: B Nonsteroidal anti-inflammatory drugs potentiate the hypoglycemic effects of sulfonylurea agents. Glipizide is a sulfonylurea. The other statements are not applicable to glipizide.

A client has a diagnosis of lung cancer. To which areas does the nurse anticipate that this client's tumor may metastasize? Select all that apply. a. Brain b. Bone c. Lymph node d. Kidney e. Liver

ANS: A, B, C, E Typical sites of metastasis of lung cancer include the brain, bone, liver, lymph nodes, and pancreas. Kidneys are not a typical site of lung cancer metastasis.

If dietary trays are usually brought to the nursing unit at 8:00 am, the nurse should plan to administer intermediate-acting insulin (Humulin N), 40 units, subcutaneously to a client between: a. 5:00 and 5:30 am b. 6:30 and 7:00 am c. 9:30 and 10:30 am d. 11:00 and 11:30 am

ANS: B Intermediate-acting insulin, such as Humulin N, should be given 60 to 90 minutes before a meal. Therefore, if the breakfast tray arrived at 8:00 am, a client would need to receive the insulin between 6:30 and 7:30 am. Regular insulin usually is administered 30 minutes before a meal, and insulin lispro is given immediately (15 minutes) before or after meals.

A client wishes to undergo genetic testing to determine cancer predisposition. What preliminary information does the nurse provide to this client about genetic testing? Select all that apply. a. Genetic testing is expensive. b. The test is performed on saliva. c. The test is performed on blood. d. It is helpful in diagnosing cancer. e. Insurance usually covers the cost.

ANS: A, C Genetic testing is an expensive procedure that uses the client's blood to rule out a person's genetic risk for a few specific cancers. Genetic testing is helpful only in detecting the risk for cancer, not for diagnosing the presence of cancer. Insurance companies generally do not cover the cost of genetic testing.

What are the features of a normal cell? Select all that apply. a. Low mitotic index b. Anaplastic c. Tight adherence d. Euploidy e. Migratory

ANS: A, C, D At any given time, there are few actively dividing cells. Normal cells have a low mitotic index. Normal cells with the exception of red blood cells produce proteins that protrude from the membranes, allowing the cells to bind tightly together. This results in normal cells being nonmigratory and prevents cells wandering from one tissue to the next. Normal chromosomes or euploidy is a characteristic feature of most normal human cells. These cells have 23 pairs of chromosomes, the correct number for human beings. Malignant cells or cancer cells are anaplastic; they lose the specific appearance of their parent cells. As a cancer cell becomes more malignant, it becomes smaller and rounded. They have large nuclear-to-cytoplasmic ratio. Normal cells have specific morphological features with small nuclear-to-cytoplasmic ratio. Cancer cells migrate because they have many enzymes on their cell surfaces and do not bind tightly to each other.

The labor nurse is providing care to a patient at 37 weeks' gestation who is an insulin-dependent diabetic. The health care provider prescribes an infusion of insulin throughout her induction to be titrated to keep her blood glucose levels below 110 mg/dL. What type of insulin will the nurse select to prepare the infusion? a. NPH insulin b. Regular insulin c. Lispro (Humalog) d. Aspart (Novolog)

ANS: B Continuous infusion of a regular insulin solution combined with a separate intravenous solution containing glucose, such as 5% dextrose in Ringer's lactate, allows titration to maintain blood glucose levels between 80 and 110 mg/dL, or as designated by facility policy. The insulin solution is raised, lowered, or discontinued to maintain euglycemia based on hourly capillary blood glucose levels.

The nurse administers 6 units of regular insulin and 10 units NPH insulin at 7 AM. At what time does the nurse assess the client for problems related to the NPH insulin? a. 8 AM b. 4 PM c. 8 PM d. 11 PM

ANS: B NPH is an intermediate-acting insulin with an onset of 1.5 hours, peak of 4 to 12 hours, and duration of action of 22 hours. Checking the client at 8:00 AM would be too soon; 8:00 PM and 11:00 PM would be too late.

A nurse assesses a client who is receiving total parenteral nutrition. For which adverse effects related to an acid-base imbalance should the nurse assess? (Select all that apply.) a. Positive Chvostek's sign b. Elevated blood pressure c. Bradycardia d. Increased muscle strength e. Anxiety and irritability

ANS: A, E A client receiving total parenteral nutrition is at risk for metabolic alkalosis. Manifestations of metabolic alkalosis include positive Chvostek's sign, normal or low blood pressure, increased heart rate, skeletal muscle weakness, and anxiety and irritability.

A client asks if there are any drugs which help to prevent the development of cancer. How does the nurse respond? Select all that apply. a. Aspirin (acetylsalicylic acid) reduces the risk of colon cancer. b. Lycopene reduces the risk of prostate cancer. c. Tamoxifen (Nalvodex) reduces the risk of breast cancer. d. There are no drugs which prevent the development of cancer. e. Anticancer drugs can be used as preventive therapy.

ANS: A,B,C The drug therapies that have proven preventive roles for various cancers are aspirin for colon cancer, lycopene for prostate cancer, and tamoxifen for breast cancer. The client should not be encouraged to use anti-cancer drugs because they are associated with a higher risk for developing serious toxic effects. At the same time, they increase the risk of developing other cancers.

Which client would be appropriate to assign to the new nurse working on the medical-surgical unit? a. Client with diabetic ketoacidosis and change in mental status who has a pH of 7.18 b. Client with emphysema and cellulitis with a PaCO2 level of 58 mm Hg c. Client with reactive airway disease, wheezing, and a PaO2 level of 62 mm Hg d. Client with a small bowel obstruction and vomiting with a bicarbonate level of 40 mEq/L

ANS: B A PaCO2 level of 58 mm Hg in the client with emphysema and cellulitis, although abnormal, is anticipated for a client with chronic obstructive pulmonary disease and is stable for assignment to a new graduate. The client with diabetic ketoacidosis and change in mental status is unstable and requires care by a more experienced nurse. The client with reactive airway disease is still wheezing, and requires experienced nursing care. The client with a small bowel obstruction is unstable and may likely require surgery, which requires more experienced nursing care.

Glipizide (Glucotrol) 10 mg bid PO has been ordered for an adult client with type 2 diabetes. The nurse would explain to the client that the medication reduces the blood sugar level by what process? a. Delays the cellular uptake of potassium and insulin b. Stimulates insulin release from the pancreas c. Decreases the body's need for and utilization of insulin at the cellular level d. Interferes with the absorption and metabolism of fats and carbohydrates

ANS: B The sulfonylureas reduce the blood glucose level by stimulating insulin release from the pancreas. Over a long period of time, sulfonylureas may actually increase insulin effects at the cellular level and decrease glucose production by the liver. This is the reason that sulfonylureas are prescribed for clients with type 2 diabetes who still have a functioning pancreas.

A client has acidosis. Which laboratory finding is of greatest concern to the nurse? a. Sodium 154 mEq/L b. Potassium 5.9 mEq/L c. Calcium 8.9 mg/dL d. Magnesium 2.1 mg/dL

ANS: B In the client with acidosis, intracellular buffering leads to entry of hydrogen ions (H+) into cells, and in return potassium leaves the cell. This leads to elevated serum potassium levels. Many severe problems with acidosis are due to the accompanying hyperkalemia.

Which client does the nurse assess for potential metabolic acidosis? a. Client admitted after collapsing during a marathon run b. Young adult following a carbohydrate-free diet c. Older adult with asthma who is on long-term steroid therapy d. Older client on antacids for gastroesophageal reflux disease

ANS: B One cause of acidosis is a strict low-calorie diet or one that is low in carbohydrate content. Such a diet increases the rate of fat catabolism and results in the formation of excessive ketoacids. Dehydration is not directly associated with acid-base disorders. In the client with asthma, acid-base status will be determined by a combination of depth of respirations and oxygen saturation. Excessive intake of sodium bicarbonate may increase the risk of metabolic alkalosis

A client is being discharged from the emergency department with several broken ribs. For which acid-base imbalance does the nurse provide discharge teaching? a. Respiratory alkalosis from anxiety and hyperventilation b. Respiratory acidosis from inadequate ventilation c. Metabolic acidosis from calcium loss from broken bones d. Metabolic alkalosis from taking base-containing analgesics

ANS: B Pain from broken ribs often causes the client to breathe more shallowly to avoid moving his or her ribs and increasing pain. If respiration is shallow enough, ventilation is inadequate, leading to poor gas exchange and respiratory acidosis. Hyperventilation would more likely cause respiratory alkalosis. The calcium loss from broken ribs probably would not affect acid-base balance. Taking analgesics as prescribed for pain probably also would not affect acid-base balance.

A client is in the emergency department after an overdose of an unknown substance. Which assessment findings does the nurse correlate with possible salicylate poisoning? a. Increased deep tendon reflexes b. Increased rate and depth of respiration c. Decreased capillary refill d. Decreased intestinal motility and paralytic ileus

ANS: B Salicylates are acidic, and salicylate poisoning increases the rate and depth of ventilation in two ways. First, salicylates directly stimulate the respiratory centers. Second, by causing a metabolic acidosis and reducing the pH of the blood, the respiratory centers are stimulated to compensate.

A client who was malnourished is being discharged. The nurse evaluates that teaching to decrease risk for the development of metabolic acidosis has been effective when the client states, "I will: a. Increase my milk intake to at least three glasses daily." b. Be sure to eat three well-balanced meals and a snack daily." c. Avoid taking pain medication and antihistamines together." d. Not add salt to food when cooking or during meals."

ANS: B Starvation or a diet with too few carbohydrates can lead to metabolic acidosis by forcing cells to switch to using fats for fuel and by creating ketoacids as a by-product of excessive fat metabolism. Eating sufficient calories from all food groups helps reduce this risk.

A nurse evaluates the following arterial blood gas values in a client: pH 7.48, PaO2 98 mm Hg, PaCO2 28 mm Hg, and HCO3- 22 mEq/L. Which client condition should the nurse correlate with these results? a. Diarrhea and vomiting for 36 hours b. Anxiety-induced hyperventilation c. Chronic obstructive pulmonary disease (COPD) d. Diabetic ketoacidosis and emphysema

ANS: B The elevated pH level indicates alkalosis. The bicarbonate level is normal, and so is the oxygen partial pressure. Loss of carbon dioxide is the cause of the alkalosis, which would occur in response to hyperventilation. Diarrhea and vomiting would cause metabolic alterations, COPD would lead to respiratory acidosis, and the client with emphysema most likely would have combined metabolic acidosis on top of a mild, chronic respiratory acidosis.

The nurse prepares to administer bicarbonate intravenously to the client with which clinical manifestations? a. pH 7.28, HCO3- 22 mEq/L, PCO2 52 mm Hg, PO2 82 mm Hg secondary to an acute asthma attack b. pH 7.28, HCO3- 16 mEq/L, PCO2 45 mm Hg, PO2 98 mm Hg secondary to excessive diarrhea c. Client with chronic emphysema and bronchitis who has the following arterial blood gases: pH 7.30, HCO3- 30 mEq/L, PCO2 60 mm Hg, PO2 72 mm Hg secondary to chronic bronchitis and emphysema d. pH 7.31, HCO3- 20 mEq/L, PCO2 34 mm Hg, PO2 96 mm Hg secondary to a urinary tract infection and type 2 diabetes

ANS: B The only client who has lower than normal bicarbonate levels is the client with diarrhea. This deficit is most likely the result of an actual bicarbonate loss, and bicarbonate should be replaced to help return this client's acid-base balance to normal. Giving bicarbonate to any of the other clients listed would be adding too much base and would risk the development of alkalosis.

Which of the following diabetes drugs acts by decreasing the amount of glucose produced by the liver? a. Sulfonylureas b. Meglitinides c. Biguanides d. Alpha-glucosidase inhibitors

ANS: C Biguanides, such as metformin, lower blood glucose by reducing the amount of glucose produced by the liver. Sulfonylureas and Meglitinides stimulate the beta cells of the pancreas to produce more insulin. Alpha-glucosidase inhibitors block the breakdown of starches and some sugars, which helps to reduce blood glucose levels

A nurse is assessing clients who are at risk for acid-base imbalance. Which clients are correctly paired with the acid-base imbalance? (Select all that apply.) a. Metabolic alkalosis - Young adult who is prescribed intravenous morphine sulfate for pain b. Metabolic acidosis - Older adult who is following a carbohydrate-free diet c. Respiratory alkalosis - Client on mechanical ventilation at a rate of 28 breaths/min d. Respiratory acidosis - Postoperative client who received 6 units of packed red blood cells e. Metabolic alkalosis - Older client prescribed antacids for gastroesophageal reflux disease

ANS: B, C, E Respiratory acidosis often occurs as the result of underventilation. The client who is taking opioids, especially IV opioids, is at risk for respiratory depression and respiratory acidosis. One cause of metabolic acidosis is a strict low-calorie diet or one that is low in carbohydrate content. Such a diet increases the rate of fat catabolism and results in the formation of excessive ketoacids. A ventilator set at a high respiratory rate or tidal volume will cause the client to lose too much carbon dioxide, leading to an acid deficit and respiratory alkalosis. Citrate is a substance used as a preservative in blood products. It is not only a base, it is also a precursor for bicarbonate. Multiple units of packed red blood cells could cause metabolic alkalosis. Sodium bicarbonate antacids may increase the risk of metabolic alkalosis.

Which of the following findings during a female breast examination should the nurse report as suspicious for breast cancer? a. Multiple nodules of round, lumpy, tender tissue in both breasts b. A single soft, mobile, lobular nodule that is nontender c. A poorly defined, firm lump that is nontender and nonmovable d. A single soft lump that is well-defined and tender

ANS: C A poorly defined, firm lump that is nontender, nonmovable, and fixed to the skin is characteristic of breast cancer. All other choices are usually associated with benign processes. All patients should have a diagnosis of cancer based upon physical assessment and tissue pathology.

A client has respiratory acidosis. The nurse evaluates that treatment is being effective with which arterial blood gas values? a. pH 7.28, HCO3- 12 mEq/L, PCO2 45 mm Hg, PO2 96 mm Hg b. pH 7.32, HCO3- 17 mEq/L, PCO2 25 mm Hg, PO2 98 mm Hg c. pH 7.35, HCO3- 36 mEq/L, PCO2 65 mm Hg, PO2 78 mm Hg d. pH 7.48, HCO3- 12 mEq/L, PCO2 35 mm Hg, PO2 85 mm Hg

ANS: C A pH of 7.35 is normal, indicating acid-base balance (fully compensated). A respiratory problem with carbon dioxide retention and inadequate gas exchange is apparent from the high PCO2 and the low PO2. The bicarbonate level is greatly elevated, indicating renal synthesis and reabsorption of HCO3-, a powerful acid-base compensatory mechanism. Thus, the amount of bicarbonate (base) in the blood adequately compensates for the increased carbon dioxide level, so that the pH is normal, although no other arterial blood gas value is normal.

In clients with any type of acid-base imbalance, the nurse places the priority on monitoring which electrolyte? a. Sodium b. Calcium c. Potassium d. Magnesium

ANS: C Any type of acid-base imbalance usually alters the blood potassium level. Both potassium and hydrogen ions carry or express an overall positive charge (cations). Body fluids maintain electroneutrality by keeping the number of positive ions matched with an equal number of negative ions (anions). In acidosis, hydrogen ions enter cells in exchange for potassium ions. Thus, a relative hyperkalemia accompanies acidosis. In alkalosis, hydrogen ions leave the cells and enter the blood in exchange for potassium ions. Thus, a relative hypokalemia usually accompanies alkalosis as extracellular potassium ions move into cells in exchange for intracellular hydrogen ions entering the extracellular fluid. The normal potassium level of the blood has a narrow range (3.5 to 5.0 mEq/L). When blood potassium levels are too high, lethal cardiac dysrhythmias may occur. When blood potassium levels are too low, skeletal muscle weakness and respiratory failure may occur.

The nurse monitors for which acid-base imbalance in a client who has hypoxemia? a. Reduced carbon dioxide production leading to alkalosis b. Reduced carbon dioxide retention leading to alkalosis c. Excess carbon dioxide production leading to acidosis d. Excess carbon dioxide retention leading to acidosis

ANS: C Hypoxemia (lower than normal blood oxygen level) causes some organs, tissues, and cells to have anaerobic metabolism. This situation leads to a buildup of carbon dioxide. Elevated levels of carbon dioxide lead to an increase in blood hydrogen ion levels and acidosis.

A client has the following arterial blood gases: pH 7.30, HCO3- 17 mEq/L, PCO2 25 mm Hg, PO2 98 mm Hg. Which intervention by the nurse is most appropriate? a. Prepare to give intravenous sodium bicarbonate. b. Document the findings and continue to assess. c. Assist the physician in determining the cause. d. Administer oxygen at 2 L per nasal cannula.

ANS: C The client has a partially compensated metabolic acidosis. Interventions are aimed at reducing or eliminating the cause. The nurse needs to assist in determining the cause so that proper interventions can be initiated. Sodium bicarbonate is rarely used for acidosis unless the pH is life threatening, or for specific causes of acidosis wherein bicarbonate deficit is known to be the problem. Simply documenting the findings will not help the client. Because the client's PO2 is 98 mm Hg, oxygen therapy is not indicated on the basis of these arterial blood gases.

A client is admitted with mixed respiratory and metabolic acidosis secondary to bronchitis and diabetic ketoacidosis. The nurse evaluates that teaching about the client's confusion was effective when a family member makes which statement? a. "It is too early to tell if the ketoacidosis will cause permanent changes." b. "Her memory will improve, but loss of some brain cells has occurred." c. "The confusion should clear when oxygen and electrolyte levels are normal." d. "The confusion should clear when blood glucose levels and other laboratory tests are normal."

ANS: C The pH abnormality alone is not responsible for the confusion. Most of the confusion is caused by hypoxia in combination with electrolyte imbalances that accompany severe combined acidosis. None of the other options address the client's hypoxia.

The nurse presents a cancer prevention program to teens. Which instruction will have the greatest impact in cancer prevention? a. Avoid asbestos. b. Wear sunscreen. c. Get the human papilloma virus (HPV) vaccine. d. Do not smoke cigarettes.

ANS: D All of these actions are part of cancer prevention; however, tobacco is the single most important source of preventable carcinogenesis. Asbestos may be found in older homes and buildings. Most schools have been through an asbestos abatement program so should not pose a risk. It would be important to share with teens who may be involved in the construction industry during the summer to be aware of asbestos risks. Although asbestos may present a risk for lung cancer, it is not a likely exposure for teens. Lifetime exposure to the sun and the use of tanning beds will increase the risk for cancer, but not as much as tobacco use. The HPV vaccine will decrease the risk for cervical cancer, but will not have as much of an impact on cancer prevention as avoiding tobacco.

When a client has an arterial blood pH of 7.48, which buffer action will bring the pH back to normal? a. Absorption of bicarbonate ions from the blood b. Release of bicarbonate ions into the blood c. Absorption of hydrogen ions from the blood d. Release of hydrogen ions into the blood

ANS: D Buffers can act as an acid (releasing a hydrogen ion) or as a base (absorbing a hydrogen ion) to assist in keeping the pH and hydrogen ion concentration of body fluids within the normal range. An arterial pH of 7.48 indicates a deficiency of hydrogen ions. This situation would cause buffers to act like acids and release hydrogen ions into the blood.

The nurse expects to find renal compensation for an acid-base imbalance in which situation? a. Mild to moderate dehydration in a middle-aged client who jogged for 2 hours b. Acute asthma attack with wheezing of 6 hours' duration in an older man c. Food poisoning with vomiting for 12 hours in a middle-aged woman d. Hypoxemia for 4 days from pneumonia in an adult woman

ANS: D Renal compensation (change in excretion or reabsorption of hydrogen ions and bicarbonate ions) for an acid-base imbalance is very potent and requires from many hours up to several days to begin. It does not provide immediate compensation, nor does it respond to acute imbalances. For a person who has been hypoxemic for several days, renal compensation with increased excretion of hydrogen ions and increased reabsorption of bicarbonate would have been initiated.

A nurse assesses a client who is prescribed furosemide (Lasix) for hypertension. For which acid-base imbalance should the nurse assess to prevent complications of this therapy? a. Respiratory acidosis b. Respiratory alkalosis c. Metabolic acidosis d. Metabolic alkalosis

ANS: D Many diuretics, especially loop diuretics, increase the excretion of hydrogen ions, leading to excess acid loss through the renal system. This situation is an acid deficit of metabolic origin.

When the nurse is counseling a 60-year-old African-American male client with all of these risk factors for lung cancer, teaching should focus most on which risk factor? a. Tobacco use b. Ethnicity c. Gender d. Increased age

ANS: A Although all of these are risk factors for lung cancer, the client's tobacco use is the only factor that he can change. Ethnicity, gender, and increasing age are associated with lung cancer, but they are not modifiable risks.

A client has moderate acidosis. Which assessment does the nurse perform first? a. Take the client's pulse and blood pressure, and analyze the electrocardiogram (ECG) strip. b. Assess respiratory rate and depth and work of breathing. c. Perform assessments of musculoskeletal strength. d. Determine whether the client is awake, alert, and oriented.

ANS: A Priority assessments for the client with acidosis relate to the cardiovascular system. Acidosis can lead to lethal cardiac dysrhythmias

The nursing student learning about cancer development remembers characteristics of normal cells. Which characteristics does this include? (Select all that apply.) a. Differentiated function b. Large nucleus-to-cytoplasm ratio c. Loose adherence d. Nonmigratory e. Specific morphology

ANS: A, D, E Normal cells have the characteristics of differentiated function, nonmigratory, specific morphology, a smaller nucleus-to-cytoplasm ratio, tight adherence, and orderly and well-regulated growth.

ANS: A The incorrect options do not contain evidence of abnormal gas exchange values. Pallor, tachycardia, hypertension, and fever can occur with Impaired Gas Exchange but alone do not yield that nursing diagnosis. Bradycardia, lethargy, flushed, and hypothermia could be an option in unusual circumstances but are not the typical picture of Impaired Gas Exchange. Elevated bicarbonate, metabolic alkalosis, irritability, and pallor do not reflect gas exchange abnormalities.

A 12-year-old is being treated for acute respiratory distress syndrome. Which assessment finding would be indicative of the nursing diagnosis Impaired Gas Exchange? a. Oxygen saturation of 62% b. Heart rate of 100 bpm c. Respiratory rate of 30/minute d. Bicarbonate level of 38

A client has an arterial blood gas pH of 7.48. How does the nurse interpret this client's acid-base status? a. An unknown acid-base balance status b. A normal blood hydrogen ion concentration c. A deficit in blood hydrogen ion concentration d. An excess in blood hydrogen ion concentration

ANS: C The pH is the negative log of the hydrogen ion concentration. The normal pH of arterial blood ranges between 7.35 and 7.45. A pH of 7.48 indicates a decrease in the hydrogen ion concentration (alkalosis).

A client with diabetes receives 10 units of regular insulin at 6:00 am and does not eat breakfast. About noon, what observation would the nurse expect to see? a. Polydipsia b. Polyphagia c. Polyuria d. Diaphoresis

ANS: D The nurse would expect symptoms of hypoglycemia, which include diaphoresis, shakiness, fatigue, hunger, and low blood sugar. The three Ps—polydipsia, polyphagia, and polyuria—are observed in hyperglycemia.

ANS: B, C, D Tetralogy of Fallot is a cyanotic lesion with decreased pulmonary blood flow. The hypoxia results in baseline oxygen saturations as low as 75% to 85%. Even with oxygen administration, saturations do not reach normal range. Pulmonary atresia is a cyanotic lesion with decreased pulmonary blood flow. The hypxoia results in baseline oxygen saturations as low as 75% to 85%. Even with oxygen administration, saturations do not reach the normal range. Transposition of the great arteries is a cyanotic lesion with increased pulmonary blood flow. PDA is a failure of the fetal shunt between the aorta and the pulmonary artery to close. PDA is not classified as a cyanotic heart disease. Prostaglandin E1 is often given to maintain dudctal patency in children with cyanotic heart diseases. VSD is the most common type of cardiac defect. The VSD is a left-to-right shunting defect; however, it may be accompanied by other defects.

As a nurse working in the newborn nursery, you notice an infant who is having circumoral cyanosis. Which CHD do you suspect the child may have? Select all that apply. a. Patent ductus arteriosus (PDA) b. Tetralogy of Fallot c. Pulmonary atresia d. Transposition of the great arteries e. Ventricular septal defect

When the nurse reviews the client's laboratory reports revealing sodium, 140 mEq/L; potassium, 4.1 mEq/L; calcium 7.9 mg/dL, and magnesium 1.9 mg/dL; the nurse should notify the physician of the client's: a. high magnesium. b. low calcium. c. high sodium. d. low potassium.

B. low calcium. Normal total serum calcium levels range between 8.9 and 10.1 mg/dL.

ANS: B A patent ductus arteriosus allows blood to flow from the high-pressure aorta to the low-pressure pulmonary artery, resulting in a left-to-right shunt. Patent ductus arteriosus is failure of the fetal shunt between the aorta and the pulmonary artery to close. Patent ductus arteriosus is not a stenotic lesion. Patent ductus arteriosus can be closed both medically and surgically. Atrioventricular defect occurs when fetal development of the endocardial cushions is disturbed, resulting in abnormalities in the atrial and ventricular septa and the atrioventricular valves.

Before preparing a teaching plan for the parents of an infant with ductus arteriosus, it is important that the nurse understands this condition. Which statement best describes patent ductus arteriosus? a. Patent ductus arteriosus involves a defect that results in a right-to-left shunting of blood in the heart. b. Patent ductus arteriosus involves a defect in which the fetal shunt between the aorta and the pulmonary artery fails to close. c. Patent ductus arteriosus is a stenotic lesion that must be surgically corrected at birth. d. Patent ductus arteriosus causes an abnormal opening between the four chambers of the heart.

ANS: B Primary polycythemia often requires phlebotomy to reduce blood volume. The increased risk of thrombus formation that accompanies the disease requires regular exercises and ambulation. Deep-breathing and coughing exercises do not directly address the cause or common sequelae of polycythemia, and neurologic manifestations are not typical.

Caring for a patient with a diagnosis of polycythemia vera will likely require you to a. Encourage deep breathing and coughing. b. Assist with or perform phlebotomy at the bedside. c. Teach the patient how to maintain a low-activity lifestyle. d. Perform thorough and regularly scheduled neurologic assessments.

A nurse who has diagnosed a client as having "fluid volume excess" related to compromised regulatory mechanism (kidneys) may have been alerted by what symptom? a. muscle twitching b. nausea and vomiting c. fingerprinting over sternum d. distended neck veins

D. distended neck veins Fluid volume excess causes the heart and lungs to work harder, leading to the veins in the neck becoming distended. Muscle twitching, and nausea and vomiting may signify electrolyte imbalances. The sternum is not an area assessed during fluid volume excess.

ANS: B Poor tissue oxygenation at the cellular level causes anaerobic metabolism, with the by-product of lactic acid. Elevated partial pressure of carbon dioxide occurs with hypoventilation, which may be related to respiratory muscle fatigue, secretions, and causes other than hypoxia. Coagulation times reflect the ability of the blood to clot, not oxygenation at the cellular level. Elevation in potassium appears in septic shock due to acidosis; this value is decreased and is not consistent with septic shock.

How does the nurse caring for a client with septic shock recognize that severe tissue hypoxia is present? a. PaCO2 58 mm Hg b. Lactate 9.0 mmol/L c. Partial thromboplastin time 64 seconds d. Potassium 2.8 mEq/L

ANS: B Plasmanate expands the blood volume and helps maintain MAP greater than 65 mm Hg, which is a desired outcome in shock. Urine output should be 0.5 mL/kg/hr, or greater than 30 mL/hr. Albumin levels reflect nutritional status, which may be poor in shock states due to an increased need for calories. Plasmanate expands blood volume by exerting increasing colloid osmotic pressure in the bloodstream, pulling fluid into the vascular space; this does not improve an abnormal hemoglobin.

How does the nurse recognize that a positive outcome has occurred when administering plasma protein fraction (Plasmanate)? a. Urine output 20 to 30 mL/hr for the last 4 hours b. Mean arterial pressure (MAP) 70 mm Hg c. Albumin 3.5 g/dL d. Hemoglobin 7.6 g/dL

ANS: A The incidence of congenital heart disease is approximately 50% in children with trisomy 21 (Down syndrome). A family history of congenital heart disease, not acquired heart disease, increases the risk of giving birth to a child with CHD. Infants born to mothers who are insulin dependent have an increased risk of CHD. Infants identified as having certain genetic defects, such as Turner syndrome, have a higher incidence of CHD. A family history is not a risk factor.

In which situation is there a risk that a newborn infant will have a congenital heart defect (CHD)? a. Trisomy 21 detected on amniocentesis b. Family history of myocardial infarction c. Father has type 1 diabetes mellitus d. Older sibling born with Turner syndrome

ANS: B Pain is the most common clinical manifestation of a crisis and usually requires large doses of continuous opioids for control. Fluid intake should be increased to reduce blood viscosity and improve perfusion. Rest is usually ordered to decrease metabolic requirements. Patients are instructed about the need for dietary folic acid, but high-protein, high-calorie diets are not emphasized.

It is important for the nurse providing care for a patient with sickle cell crisis to a. limit the patient's intake of oral and IV fluids. b. evaluate the effectiveness of opioid analgesics. c. encourage the patient to ambulate as much as tolerated. d. teach the patient about high-protein, high-calorie foods.

ANS: D Diapers must be weighed for an accurate record of output. The head of the bed should be raised to decrease the work of breathing. Oxygen should be administered during stressful periods such as when the child is crying. Nursing care should be planned to allow for periods of undisturbed rest.

Nursing care for the child in congestive heart failure includes a. Counting the number of saturated diapers b. Putting the infant in the Trendelenburg position c. Removing oxygen while the infant is crying d. Organizing care to provide rest periods

ANS: A An experienced UAP will have been taught how to obtain a stool specimen for the Hemoccult slide test, because this is a common screening test for hospitalized patients. Having the patient sign an informed consent form should be done by the physician who will be performing the colonoscopy. Administering medications and checking for allergies are within the scope of practice of licensed nursing staff. Focus: Delegation

These activities are included in the care plan for a 78-year-old patient admitted to the hospital with anemia caused by possible gastrointestinal bleeding. Which activity can you delegate to an experienced UAP? a. Obtaining stool specimens for fecal blood test (Hemoccult) slides b. Having the patient sign a colonoscopy consent form c. Giving the prescribed polyethylene glycol electrolyte solution (GoLYTELY) d. Checking for allergies to contrast dye or shellfish

ANS: C Taking iron with vitamin C (ascorbic acid) or orange juice, which contains ascorbic acid, also enhances iron absorption. Milk may interfere with iron absorption. Ginger ale and water do not facilitate iron absorption.

When providing teaching for the patient with iron-deficiency anemia who has been prescribed iron supplements, you should include taking the iron with which beverage? a. Milk b. Ginger ale c. Orange juice d. Water

ANS: C The manifestations, including growth and developmental deficits, develop in childhood by 2 years of age.

You anticipate the onset of manifestations related to thalessemia to occur by a. 6 months of age. b. age 1 year. c. age 2 year. d. adolescence.

ANS: A Fever increases the metabolic demands of the body, increasing production of carbon dioxide. The body hyperventilates to get rid of excess carbon dioxide. Anxiety can cause hyperventilation, but this is not the direct cause from a fever. Hyperventilation decreases the drive to breathe. The cause of the fever in this question is unknown.

A nurse is caring for a patient whose temperature is 100.2° F. The nurse expects this patient to hyperventilate owing to a. Increased metabolic demands. b. Anxiety over illness. c. Decreased drive to breathe. d. Infection destroying lung tissues.

ANS: B Lower blood volume will decrease MAP. The other answers are not accurate.

A student is caring for a client who suffered massive blood loss after trauma. How does the student correlate the blood loss with the client's mean arterial pressure (MAP)? a. It causes vasoconstriction and increased MAP. b. Lower blood volume lowers MAP. c. There is no direct correlation to MAP. d. It raises cardiac output and MAP.

Which intervention does the nurse include in caring for a client with generalized edema who is receiving a loop diuretic? a. Turning the client every 2 hours. b. Encouraging oral fluids. c. Restricting dietary potassium. d. Applying antiembolism stockings.

A. Turning the client every 2 hours. The client with generalized edema/fluid overload is at risk for developing skin breakdown, especially at pressure points over bony prominences. Changing position frequently reduces this risk. Fluids are more likely to be restricted with fluid overload, not encouraged. Loop diuretics cause potassium loss so dietary potassium is encouraged, not restricted. Antiembolism hose have no role in generalized edema from fluid overload.

The hand grasps of a client with acidosis have diminished since the previous assessment 1 hour ago. What action does the nurse take next? a. Assess client's rate, rhythm, and depth of respiration. b. Measure the client's pulse and blood pressure. c. Document findings and continue to monitor. d. Notify the physician as soon as possible.

ANS: A Progressive skeletal muscle weakness is associated with increasing severity of acidosis. Muscle weakness can lead to severe respiratory insufficiency. Acidosis does lead to dysrhythmias (due to hyperkalemia) but these would best be assessed with cardiac monitoring. Findings should be documented, but simply continuing to monitor is not sufficient. Before notifying the physician, the nurse needs to have more data to report.

A postoperative client received six units of packed red blood cells (PRBCs) for intraoperative blood loss. The nurse monitors the client for which acid-base imbalance? a. Metabolic alkalosis b. Metabolic acidosis c. Respiratory alkalosis d. Respiratory acidosis

ANS: A Citrate is a substance used as a preservative in blood products. It is not only a base, it is also a precursor for bicarbonate (bicarbonate can be formed from citrate). Rapid administration of blood products can cause metabolic alkalosis by infusing large amounts of citrate intravenously. Although this problem is more likely to occur with administration of whole blood or blood plasma, multiple transfusions with packed red cells can also result in excessive amounts of citrate being received by the client.

A nurse assesses a client who is admitted with an acid-base imbalance. The client's arterial blood gas values are pH 7.32, PaO2 85 mm Hg, PaCO2 34 mm Hg, and HCO3- 16 mEq/L. What action should the nurse take next? a. Assess client's rate, rhythm, and depth of respiration. b. Measure the client's pulse and blood pressure. c. Document the findings and continue to monitor. d. Notify the physician as soon as possible.

ANS: A Progressive skeletal muscle weakness is associated with increasing severity of acidosis. Muscle weakness can lead to severe respiratory insufficiency. Acidosis does lead to dysrhythmias (due to hyperkalemia), but these would best be assessed with cardiac monitoring. Findings should be documented, but simply continuing to monitor is not sufficient. Before notifying the physician, the nurse must have more data to report.

The nurse working with oncology clients understands that interacting factors affect cancer development. Which factors does this include? (Select all that apply.) a. Exposure to carcinogens b. Genetic predisposition c. Immune function d. Normal doubling time e. State of euploidy

ANS: A, B, C The three interacting factors needed for cancer development are exposure to carcinogens, genetic predisposition, and immune function.

A nurse assesses a client who is experiencing an acid-base imbalance. The client's arterial blood gas values are pH 7.32, PaO2 94 mm Hg, PaCO2 34 mm Hg, and HCO3- 18 mEq/L. For which clinical manifestations should the nurse assess? (Select all that apply.) a. Reduced deep tendon reflexes b. Drowsiness c. Increased respiratory rate d. Decreased urinary output e. Positive Trousseau's sign

ANS: A, B, C Metabolic acidosis causes neuromuscular changes, including reduced muscle tone and deep tendon reflexes. Clients usually present with lethargy and drowsiness. The respiratory system will attempt to compensate for the metabolic acidosis; therefore, respirations will increase rate and depth. A positive Trousseau's sign is associated with alkalosis. Decreased urine output is not a manifestation of metabolic acidosis.

The nurse would incorporate which of the following into the plan of care as a primary prevention strategy for reduction of the risk for cancer? a. Yearly mammography for women aged 40 years and older b. Using skin protection during sun exposure while at the beach c. Colonoscopy at age 50 and every 10 years as follow-up d. Yearly prostate specific antigen (PSA) and digital rectal exam for men aged 50 and over

ANS: B Primary prevention of cancer involves avoidance to known causes of cancer, such as sun exposure. Secondary screening involves physical and diagnostic examination.

The nurse understands the following about the correct administration of insulin lispro: a. It needs to be taken after the meals. b. It should be taken within 15 minutes of beginning a meal. c. It is to be taken once daily at the noon meal. d. It is taken only in the evenings with a snack before bedtime.

ANS: B Rapid-acting insulins, such as insulin lispro (Humalog) and insulin aspart (Novolog), are able to more closely mimic the body's natural rapid insulin output after consumption of a meal, which is why both medications usually are administered within 15 minutes of beginning a meal.

The nurse is planning care for a patient with hypercalcemia secondary to bone metastasis. Which of the following interventions will be included in the plan of care? (Select all that apply.) a. Increasing oral fluids b. Placement of an oral airway at the bedside c. Monitoring for Chvostek's sign d. Implementing seizure precautions e. Hyperactive reflex assessment f. Observation for muscle weakness

ANS: A, D Serious complications of hypercalcemia include severe muscle weakness, dehydration, loss of deep tendon reflexes, paralytic ileus, and electrocardiographic changes. Early manifestations of hypercalcemia include fatigue, loss of appetite, nausea, vomiting, constipation, and polyuria (increased urine output).

A client with diabetes receives a combination of regular and NPH insulin at 0700 hours. The nurse teaches the client to be alert for signs of hypoglycemia at: a. 12 pm and 1 pm (1200 and 1300 hours) b. 9am and 5pm (0900 and 1700 hours) c. 10 am and 10 pm (1000 and 2200 hours) d. 8am and 11 am (0800 and 1100 hours)

ANS: B Regular insulin (a short-acting insulin) peaks in 2 to 3 hours, and NPH (an intermediate-acting insulin) peaks in 4 to 10 hours. Hypoglycemia would most likely occur between 9 am and 5 pm (0900 to 1700 hours).

A client is at risk for developing colon cancer. What are the various preventive interventions that the nurse may plan to implement? Select all that apply. a. Suggest taking aspirin. b. Administer the vaccine Gardasil. c. Encourage a diet rich in fiber and fat. d. Suggest limiting the number of sexual partners. e. Refer for a polypectomy if the client has associated polyps.

ANS: A, E Clients who are at risk for developing colon cancer may begin taking aspirin, and removing at-risk tissues like associated polyps, if present, to reduce their risk. Gardasil is a vaccine effective against cervical cancer. The client should be encouraged to have a diet rich in fiber and low in fat. Limiting sexual partners is a practice helpful in preventing cervical cancer.

A client has moderate metabolic alkalosis. What is the priority intervention for the nurse? a. Monitor daily laboratory values. b. Assess the client's muscle strength. c. Determine the cause of the problem. d. Teach the client preventive measures.

ANS: B Although all options are viable nursing interventions, the priority is providing for client safety. Clients with metabolic alkalosis have muscle weakness and thus are at risk for falling.

What is the best response by the nurse to a parent asking about antidiarrheal medication for her 18-month-old child? a. "It is okay to give antidiarrheal medication to a young child as long as you follow the directions on the box for correct dosage." b. "Antidiarrheal medication is not recommended for young children because it slows the body's attempt to rid itself of the pathogen." c. "I'm sure your child won't like the taste, so give extra fluids when you give the medication." d. "Antidiarrheal medication will lessen the frequency of stools, but give your child Gatorade to maintain electrolyte balance."

ANS: B Antidiarrheal medications are not recommended for children younger than 2 years old. Antidiarrheal medications may actually prolong diarrhea because the body will retain the organism causing the diarrhea, further increasing fluid and electrolyte losses. The use of these medications is not recommended for children younger than 2 years old because of their binding nature and potential for toxicity. This action is inappropriate because antidiarrheal medications should not be given to a child younger than 2 years old. It is not appropriate to advise a parent to use antidiarrheal medication for a child younger than 2 years old. Education about appropriate oral replacement fluids includes avoidance of sugary drinks, apple juice, sports beverages, and colas.

The nurse providing care for a patient with suspected cancer recalls that the only diagnostic procedure that is definitive for a diagnosis of cancer is: a. MRI b. Biopsy c. CT scan d. Tumor marker

ANS: B Only a biopsy is a definitive means of diagnosing cancer, because it actually identifies the pathological cells. Many tests, such as MRI, CT scan, and tumor markers, are indicative of cancer, but they do not confirm the presence of cancer cells as examination of a specimen obtained by biopsy does.

What is the most important factor in determining the rate of fluid replacement in the dehydrated child? a. The child's weight b. The type of dehydration c. Urine output d. Serum potassium level

ANS: B The child's weight determines the amount of fluid needed, not the rate of fluid replacement. One milliliter of body fluid is equal to 1 g of body weight; therefore a loss of 1 kg (2.2 lb) is equal to 1 L of fluid. Isonamtremic and hyponatremic dehydration resuscitation involves fluid replacement over 24 hours. Hypernatremic dehydration involves a slower replacement rate to prevent a sudden decrease in the sodium level. Urine output is not a consideration for determining the rate of administration of replacement fluids. Potassium level is not as significant in determining the rate of fluid replacement as the type of dehydration.

A nurse is caring for a client who has the following arterial blood values: pH 7.12, PaO2 56 mm Hg, PaCO2 65 mm Hg, and HCO3- 22 mEq/L. Which clinical situation should the nurse correlate with these values? a. Diabetic ketoacidosis in a person with emphysema b. Bronchial obstruction related to aspiration of a hot dog c. Anxiety-induced hyperventilation in an adolescent d. Diarrhea for 36 hours in an older, frail woman

ANS: B Arterial blood gas values indicate that the client has acidosis with normal levels of bicarbonate, suggesting that the problem is not metabolic. Arterial concentrations of oxygen and carbon dioxide are abnormal, with low oxygen and high carbon dioxide levels. Thus, this client has respiratory acidosis from inadequate gas exchange. The fact that the bicarbonate level is normal indicates that this is an acute respiratory problem rather than a chronic problem, because no renal compensation has occurred.

A nurse is planning care for a client who is hyperventilating. The client's arterial blood gas values are pH 7.30, PaO2 94 mm Hg, PaCO2 31 mm Hg, and HCO3- 26 mEq/L. Which question should the nurse ask when developing this client's plan of care? a. "Do you take any over-the-counter medications?" b. "You appear anxious. What is causing your distress?" c. "Do you have a history of anxiety attacks?" d. "You are breathing fast. Is this causing you to feel light-headed?

ANS: B The nurse should assist the client who is experiencing anxiety-induced respiratory alkalosis to identify causes of the anxiety. The other questions will not identify the cause of the acid-base imbalance.

In a client 4 minutes post cardiac arrest, the nurse correlates the largest source of excess hydrogen ions with which cause? a. Excess renal retention of carbon dioxide due to hypoxia b. Release of intracellular acids due to widespread tissue destruction c. Anaerobic metabolism, leading to the buildup of lactic acid d. Using fat as a fuel source, resulting in increased fat degradation

ANS: C Glucose metabolism continues under anaerobic conditions to supply the body with chemical energy (adenosine triphosphate [ATP]). However, this metabolism is incomplete, stopping at lactic acid production instead of continuing into the Krebs' cycle. This results in a large buildup of lactic acid, which releases excessive amounts of hydrogen ions into the blood.

The nurse is caring for a group of clients with acidosis. The nurse recognizes that Kussmaul respirations are consistent with which situation? a. Client receiving mechanical ventilation b. Use of hydrochlorothiazide c. Aspirin overdose d. Administration of sodium bicarbonate

ANS: C If acidosis is metabolic in origin, the rate and depth of breathing increase as the hydrogen ion level rises; this is known as Kussmaul respirations. Metabolic acidosis is caused by alcoholic beverages, methyl alcohol, and acetylsalicylic acid (aspirin). Mechanical ventilation is used to correct hypoxemia and hypercapnia (elevated PaCO2). Hydrochlorothiazide causes metabolic alkalosis. Sodium bicarbonate is used in the treatment of metabolic acidosis; administration of this buffer may cause metabolic alkalosis.

A nurse is planning care for a client who is anxious and irritable. The client's arterial blood gas values are pH 7.30, PaO2 96 mm Hg, PaCO2 43 mm Hg, and HCO3- 19 mEq/L. Which questions should the nurse ask the client and spouse when developing the plan of care? (Select all that apply.) a. "Are you taking any antacid medications?" b. "Is your spouse's current behavior typical?" c. "Do you drink any alcoholic beverages?" d. "Have you been experiencing any vomiting?" e. "Are you experiencing any shortness of breath?"

ANS: B, C This client's symptoms of anxiety and irritability are related to a state of metabolic acidosis. The nurse should ask the client's spouse or family members if the client's behavior is typical for him or her, and establish a baseline for comparison with later assessment findings. The nurse should also assess for alcohol intake because alcohol can change a client's personality and cause metabolic acidosis. The other options are not causes of metabolic acidosis.

A client has a very high risk for developing breast cancer. What preventive measures may the nurse recommend? Select all that apply. a. Encourage a diet rich in fiber and fat. b. Discuss the need for a mastectomy. c. Suggest the long-term use of vitamin D. d. Explain the need for long-term use of tamoxifen. e. Recommend limiting the number of sexual partners.

ANS: B, C, D Preventive measures for clients who are at high risk for developing breast cancer include the removal of the breast (mastectomy), and long-term intake of vitamin D and tamoxifen. The client should be encouraged to consume a diet rich in fiber and low in fat. Limiting the number of sexual partners is helpful in preventing cervical cancer, not breast cancer.

A client who is hospitalized has been diagnosed with Epstein-Barr viral infection. What are future malignancies that the client is at risk for developing? Select all that apply. a. Cervical cancer b. B-cell lymphoma c. Burkitt's lymphoma d. Primary liver cancer e. Nasopharyngeal carcinoma

ANS: B, C, E The Epstein-Barr virus predisposes the client to developing B-cell lymphoma, Burkitt's lymphoma, and nasopharyngeal carcinoma. Human papilloma viral infection is a risk factor for cervical cancer. Hepatitis B and C infections are risk factors for primary liver cancer.

A school-age child with acute diarrhea and mild dehydration is being given oral rehydration solutions (ORS). The child's mother calls the clinic nurse to report the child has occasional vomiting. What is the appropriate recommendation by the nurse? A. Bring the child to the hospital for intravenous fluids. B. Alternate giving the child ORS and carbonated drinks. C. Continue to give the child ORS frequently in small amounts. D. Maintain the child on NPO for 8 hours and resume ORS if vomiting subsides.

ANS: C A school-age child with mild dehydration can be rehydrated safely at home with oral solutions. Carbonated drinks should not be given to the child. They may have a high carbohydrate content and contain caffeine, which is a diuretic. Vomiting is not a contraindication to the use of ORS unless it is severe. The mother should continue to give the ORS in small amounts and at frequent intervals. NPO status is not indicated. Frequent intake of ORS in small amounts is recommended.

The nurse assesses for acidosis in the client with which assessment data? a. Serum sodium level of 130 mEq per liter and peripheral edema b. Serum sodium level of 144 mEq per liter and tachycardia c. Serum potassium level of 6.5 mEq per liter and flaccid paralysis d. Serum potassium level of 4.5 mEq per liter and hyperactive deep tendon reflexes

ANS: C When acidosis is present, the hydrogen ion concentration of the extracellular fluid (ECF) is increased above normal. The physiologic action to reduce the ECF hydrogen ion concentration is to move the hydrogen ions into the cells in exchange for potassium ions, thereby maintaining the electroneutrality of the intracellular fluid. As a result, acidosis is accompanied by hyperkalemia, which diminishes nerve and skeletal muscle excitability, causing flaccid paralysis.

A client has metabolic alkalosis. Which laboratory results is the nurse most likely to assess as consistent with this condition? a. Na+ 134 mg/dL b. Mg2+ 1.5 mg/dL c. K+ 3.1 mEq/L d. Ca2+ 11.5 mg/dL

ANS: C Both potassium and hydrogen ions carry or express an overall positive charge (cations). Body fluids maintain electroneutrality by keeping the number of positive ions matched with an equal number of negative ions (anions). A compensation of alkalosis is the movement of hydrogen ions into cells inside the blood and other extracellular fluids. To prevent the blood from expressing too many positive charges, another positive ion must leave the blood and enter the cells. Potassium is the positive ion that usually is exchanged for a hydrogen ion. Thus, a relative hypokalemia usually accompanies alkalosis as extracellular potassium ions move into cells in exchange for intracellular hydrogen ions entering the extracellular fluid.

A client has a prolonged fever. For which acid-base imbalance does the nurse assess the client further? a. Metabolic acidosis from excess bicarbonate production b. Metabolic alkalosis from dehydration and hyperkalemia c. Metabolic acidosis from increased production of hydrogen ions d. Respiratory alkalosis from impaired gas exchange

ANS: C Increased body temperature is associated with hypermetabolism and increases the rate at which hydrogen ions are produced. Increased bicarbonate production would lead to metabolic alkalosis. Hyperkalemia leads to metabolic acidosis. Having a fever would not directly lead to gas exchange problems.

A nurse is evaluating an infant brought to the clinic with severe diarrhea. What signs and symptoms indicate that the infant has severe dehydration? A. Tachycardia, decreased tears, 5% weight loss B. Normal pulse and blood pressure, intense thirst C. Irritability, moderate thirst, normal eyes and fontanel D. Tachycardia, capillary refill greater than 3 seconds, sunken eyes and fontanel

ANS: D In severe dehydration, there is a 15% weight loss in infants. Tachycardia, orthostatic hypotension and shock, and intense thirst would be expected. The infant would be extremely irritable, with sunken eyes and fontanel. Tachycardia, capillary refill greater than 3 seconds, and sunken eyes and fontanel are the symptoms of severe dehydration.

Which statement best describes why infants are at greater risk for dehydration than older children? a. Infants have an increased ability to concentrate urine. b. Infants have a greater volume of intracellular fluid. c. Infants have a smaller body surface area. d. Infants have an increased extracellular fluid volume.

ANS: D Because the kidneys are immature in early infancy, there is a decreased ability to concentrate the urine. Infants have a larger proportion of fluid in the extracellular space. Infants have proportionately greater body surface area in relation to body mass, which creates the potential for greater fluid loss through the skin and gastrointestinal tract. The larger ratio of extracellular fluid to intracellular fluid predisposes the infant to dehydration.

A client has a new insulin pump. Which is the nurse's priority instruction in teaching the client? a. "Test your urine daily for ketones." b. "Use only buffered insulin." c. "Keep the insulin frozen until you need it." d. "Change the needle every 3 days."

ANS: D Having the same needle remain in place through the skin for longer than 3 days drastically increases the risk for infection in or through the delivery system. Having an insulin pump does not require the client to test for ketones in the urine. Insulin should not be frozen. Insulin is not buffered.

Which statement about the process of malignant transformation is correct? a. Mutation of genes is an irreversible event that always leads to cancer development in the initiation phase. b. Insulin and estrogen enhance the division of an initiated cell during the promotion phase. c. Tumors form when carcinogens invade the gene structure of the cell in the latency phase. d. Nutrition of cancer cells is provided by tumor angiogenesis factor (TAF) in the promotion stage.

ANS: D The promotion phase consists of progression when the blood supply changes from diffusion to TAF.Insulin and estrogen increase cell division. If cell division is halted, mutation of genes does not lead to cancer development in the initiation phase. In the initiation phase, carcinogens invade the DNA of the nucleus of a single cell. A 1-cm tumor consists of 1 billion cells. The latency phase occurs between initiation and tumor formation.

When a newly diagnosed type 2 diabetes mellitus patient asks the nurse why she has to take a pill instead of insulin, you reply that in type 2 diabetes, the body makes insulin but: a. overweight and underactive people simply cannot use the insulin produced. b. metabolism is slowed in some people so they have to take a pill to speed up their metabolism. c. sometimes the autoimmune system works against the action of the insulin. d. the cells become resistant to the action of insulin. Pills are given to increase the sensitivity.

ANS: D Type 2 diabetes mellitus is a disease in which the cells become resistant to the action of insulin and the blood glucose level rises. Oral hyperglycemic agents make the cells more sensitive.

A client has severe metabolic alkalosis. Which nursing diagnosis does the nurse choose as the client's priority problem? a. Fluid volume excess related to reduced kidney function b. Fluid volume deficit related to increased insensitive fluid loss through lungs c. Risk for impaired skin integrity related to accompanying peripheral edema d. Risk for injury related to increased neuronal sensitivity from hypocalcemia

ANS: D Metabolic alkalosis is manifested by a high pH, which causes serum calcium to bind and reduces the concentration of free calcium. This relative hypocalcemia increases the risk for increased neuromuscular activity, including tetany and seizures.

ANS: B To perform diaphragmatic breathing correctly, the client should place his or her hands on his or her abdomen to create resistance. This type of breathing cannot be performed effectively while lying on the side or with hands over the head. This type of breathing would not be as effective lying prone.

After teaching a client how to perform diaphragmatic breathing, the nurse assesses the client's understanding. Which action demonstrates that the client correctly understands the teaching? a. The client lays on his or her side with his or her knees bent. b. The client places his or her hands on his or her abdomen. c. The client lays in a prone position with his or her legs straight. d. The client places his or her hands above his or her head.

A nurse is caring for a patient who has fluid imbalance related to the development of ascites. Which imbalances would the nurse monitor for in this patient? Select all that apply. a) Extracellular fluid volume deficit b) Protein deficit c) Metabolic alkalosis d) Sodium deficit e) Plasma-to-interstitial fluid shift f) Metabolic acidosis

a, b, d, e. Patients with fluid loss due to ascites are at risk for extracellular fluid volume deficit, protein deficit, sodium deficit, and plasma-interstitial fluid shift.

ANS: A The alveolus is a capillary membrane that allows gas exchange of oxygen and carbon dioxide during respiration. The alveoli do not store oxygen, regulate tidal volume, or produce hemoglobin.

The nurse knows that the primary function of the alveoli is to a. Carry out gas exchange. b. Store oxygen. c. Regulate tidal volume. d. Produce hemoglobin.

ANS: A, B, D Hypertension, obesity, and excessive stress are all modifiable risk factors for coronary artery disease. Age is a nonmodifiable risk factor.

A nursing student learns about modifiable risk factors for coronary artery disease. Which factors does this include? (Select all that apply.) a. Hypertension b. Stress c. Age d. Obesity

ANS: A, C, E Septic shock manifests with decreased cardiac output, increased blood glucose, and increased serum lactate. The other parameters do not correlate with septic shock.

A client has septic shock. Which hemodynamic parameters does the nurse correlate with this type of shock? (Select all that apply.) a. Decreased cardiac output b. Increased cardiac output c. Increased blood glucose d. Decreased blood glucose e. Increased serum lactate f. Decreased serum lactate

ANS: A A sign of early sepsis is low-grade fever. Both early sepsis and thrombus may cause tachycardia, tachypnea, and hypotension.

A client recovering from an open reduction of the femur suddenly feels light-headed, with increased anxiety and agitation. Which key vital sign differentiates a pulmonary embolism from early sepsis? a. Temperature b. Pulse c. Respiration d. Blood pressure

A decrease in arterial blood pressure will result in the release of: a) insulin. b) renin. c) protein. d) thrombus.

b) renin Decreased arterial blood pressure, decreased renal blood flow, increased sympathetic nerve activity, and/or low-salt diet can stimulate renin release.

A nurse is preparing an education plan for a client with heart failure who is experiencing edema. As part of the plan, the nurse wants to describe the underlying mechanism for why the edema develops. Which mechanism would the nurse most likely address? a. increased hydrostatic pressure b. decreased colloid oncotic pressure c .increased capillary permeability d. blockage of the lymph nodes

A. increased hydrostatic pressure The edema that occurs with heart failure is caused by decreased cardiac output with a back-up of blood resulting from increased hydrostatic pressure. Decreased colloid oncotic pressure is the mechanism responsible for edema of malnutrition, liver failure, and nephrosis. Lymph node blockage is the mechanism responsible for edema associated with a mastectomy or lymphoma. Increased capillary permeability is the mechanism responsible for edema associated with allergies, septic shock and pulmonary edema.

The nurse is administering metformin (Glucophage) to a client. What nursing observations would cause the nurse concern regarding side effects of the medication? a. Gastrointestinal upset b. Photophobia c. Hyperglycemia d. Skin eruptions

ANS: A Anorexia, nausea, and a metallic taste in the mouth are common side effects, but can contribute to the client not taking the medication if unaware of the expected side effects. Over time, the gastrointestinal symptoms subside and can be relieved by taking the medication with food or by starting at a lower dose.

ANS: B The clinical manifestations of digoxin toxicity are often vague and nonspecific and include anorexia, fatigue, blurred vision, and changes in mental status, especially in older adults. Older adults are more likely than other patients to become toxic because of decreased renal excretion.

An older adult taking digoxin and hydrochlorothiazide (HCTZ) for chronic heart failure is admitted to the emergency department (ED) with an apical pulse of 48. A family member states that the client has reported blurred vision and loss of appetite for 2 weeks. What is the nurse's first action? a. Call the ED physician immediately b. Draw a serum digoxin level c. Assess for signs of hypokalemia d. Establish the client's airway

As adults age, which common physiologic change is likely to alter their hydration status? a. Adrenal gland growth b. Decreased muscle mass c. Increased thirst mechanism d. Poor skin turgor

B. Decreased muscle mass Decreased muscle mass causes decreased total body water, thus altering hydration status in the older adult. Adrenal growth is not a common age-related change. A decreased, not increased, thirst reflex is a common change related to aging. Poor skin turgor is a sign, not a cause, of altered hydration status.

A 70-year-old woman is admitted to the hospital with heart failure, shortness of breath (SOB), and 3+ pitting edema in her lower extremities. Her current medications are furosemide (Lasix), digoxin (Lanoxin), and an angiotensin-converting enzyme (ACE) inhibitor (Lotensin). She states that she stopped taking her Lasix because she did not think that it was helping her heart failure. Her health care provider orders furosemide (Lasix) 5 mg IV push. Which client assessment determines that the medication is working? a. Decreased blood pressure (BP) b. Increased heart rate c. Increased urine output d. Weight gain

C. Increased urine output When giving Lasix, the nurse monitors the client for response to drug therapy, especially weight loss and increased urine output. Although a fall in the client's BP may occur with the decrease in body fluid, this is not the most important assessment to be monitored. Urinary output is most important. Lasix may cause a decrease in heart rate as it lowers the client's body fluid, but this effect would take some time to note. Weight loss, rather than weight gain, is often the effect of Lasix, but it does not occur immediately.

A client is admitted to the nursing unit with a diagnosis of hypokalemia. Which assessment does the nurse complete first? a. Auscultating bowel sounds b. Checking deep tendon reflexes (DTRs) c. Determining the level of consciousness (LOC) d. Obtaining a pulse oximetry reading

D. Obtaining a pulse oximetry reading Because hypokalemia may cause respiratory insufficiency and respiratory arrest, the client's respiratory status should be assessed first. Bowel sounds, DTRs, and LOC may change in a client with hypokalemia, but these changes are not immediately life-threatening.

ANS: B Improvement in activity tolerance, less orthopnea, and improved symptoms represent a positive response to beta blockers. An ejection fraction of 25% is well below the normal of 50% to 70% and indicates poor cardiac output. Repeated hospitalization for acute exacerbation of left-sided heart failure does not demonstrate a positive outcome. Although metoprolol decreases the heart rate, palpitations are defined as the feeling of the heart beating fast in the chest; this is not a positive outcome.

How does the nurse in the cardiac clinic recognize that the client with heart failure has demonstrated a positive outcome related to the addition of metoprolol (Lopressor) to the medication regimen? a. Ejection fraction is 25% b. Client states that she is able to sleep on one pillow c. Client was hospitalized five times last year with pulmonary edema d. Client reports that she experiences palpitations

ANS: A Patients with severe anemia (hemoglobin <6 g/dL) exhibit the following cardiovascular and pulmonary manifestations: tachycardia, increased pulse pressure, systolic murmurs, intermittent claudication, angina, heart failure, myocardial infarction; tachypnea, orthopnea, dyspnea at rest.

In a severely anemic patient, you expect to find a. dyspnea and tachycardia. b. cyanosis and pulmonary edema. c. cardiomegaly and pulmonary fibrosis. d. ventricular dysrhythmias and wheezing.

ANS: C As a result of accelerated RBC breakdown, the patient may have characteristic clinical findings of hemolysis, including jaundice and elevated serum bilirubin levels.

In addition to altered red blood cells (RBCs), which laboratory finding does the nurse expect for the patient with sickle cell disease? a. Leukocytosis b. Hypouricemia c. Hyperbilirubinemia d. Hypercholesteremia

ANS: A & B The stool guaiac test is done to determine whetherthe cause of the iron-deficiency anemia is related to gastrointestinal bleeding. Iron should be increased in the diet. Teach the patient which foods are good sources of iron. If nutrition is already adequate, increasing iron intake by dietary means may not be practical. The patient with iron deficiency related to acute blood loss may require a transfusion of packed red blood cells (RBCs).

Nursing interventions for a patient with severe anemia related to peptic ulcer disease include (select all that apply) a. Monitoring stools for guaiac. b. Instructions about a high-iron diet. c. Taking vital signs every 8 hours. d. Teaching self-injection of erythropoietin.

ANS: D You must assist the patient in reducing infection risk. The patient is susceptible to infection and is at risk for septic shock and death. Even a low-grade temperature (>100.4° F) should be considered a medical emergency. Thrombocytopenia manifests as a predisposition to bleeding evidenced by petechiae, ecchymosis, and epistaxis. Pain is not experienced nor is diarrhea. Nausea and malnutrition are not related to this disease except as a by-product of infection.

The care plan for a patient with aplastic anemia should include activities to minimize the risk for which complications? a. Dyspnea and pain b. Diarrhea and fatigue c. Nausea and malnutrition d. Infection and hemorrhage

NS: C A blood pressure change (increase or decrease) of greater than 20 mm Hg during or after activity indicates poor cardiac tolerance of the activity. A significant decrease (>20%) in blood pressure during or after activity is especially ominous, because it indicates an inability of the left ventricle to maintain sufficient cardiac output.

The rehabilitation nurse is assisting a client with heart failure to increase activity tolerance. During ambulation of the client, identification of what symptom causes the nurse to stop the client's activity? a. Decrease in oxygen saturation from 98% to 95% b. Respiratory rate change from 22 to 28 breaths/min c. Systolic blood pressure change from 136 to 96 mm Hg d. Increase in heart rate from 86 to 100 beats/min

ANS: B Prevention of dehydration, which can trigger the sickling process, is a priority goal in the care of a child with sickle cell disease.

The statement made by a parent indicating understanding of health maintenance of a child with sickle cell disease is: a. "I should give my child a daily iron supplement." b. "It is important for my child to drink plenty of fluids." c. "He needs to wear protective equipment if he plays contact sports." d. "He shouldn't receive any immunizations until he is older."

ANS: A, B, C, D, E All of the options are classic signs of thalassemia major.

What are the classic symptoms of thalassemia major (Cooley's anemia)? Select all that apply. a. Hepatomegaly b. Jaundice c. Protruding teeth d. Pathological fractures e. Cardiac failure

ANS: B, D, E, F Difficulty breathing—especially with activity—is common with anemia. Lower levels of hemoglobin carry less O2 to the cells of the body. Fatigue is a classic symptom of anemia; lowered O2 levels contribute to a faster pulse (i.e., cardiac rate) and tend to "wear out" a client's energy. Lowered O2 levels deliver less oxygen to all cells, making clients with anemia pale—especially their ears, nail beds, palms, and conjunctivae and around the mouth. Respiratory problems with anemia do not include changes in breath sounds; dyspnea and decreased oxygen saturation levels are present. Skin is cool to the touch, and an intolerance to cold is noted; elevated temperature would signify something additional, such as infection.

What are the typical clinical manifestations of anemia? a. Decreased breath sounds B. Dyspnea on exertion c. Elevated temperature d. Fatigue e. Pallor f. Tachycardia

ANS: A Hypoxia is due to inadequate tissue oxygen at the cellular level. The earliest sign of hypoxia is restlessness; as it progresses, mental status changes, cardiac changes, and cyanosis can occur. Early hypoxia results in an elevated blood pressure. In later hypoxia, vital sign changes such as increased heart and respiratory rate occur. Cyanosis is a late sign of hypoxia.

What assessment finding is the earliest sign of hypoxia? a. Restlessness b. Decreased blood pressure c. Cardiac dysrhythmias d. Cyanosis

ANS: A Because all marrow elements are affected, hemoglobin, WBC, and platelet values are decreased in aplastic anemia. Other RBC indices usually are normal.

You are evaluating the laboratory data of the patient with suspected aplastic anemia. Which findings support this diagnosis? a. Reduced RBCs, reduced white blood cells (WBCs), and reduced platelets b. Reduced RBCs, normal WBCs, and normal platelets c. Normal RBCs, reduced WBCs, and reduced platelets d. Elevated RBCs, increased WBCs, and increased platelets

ANS: A Regardless of how much cobalamin (B12) is ingested, the patient is not able to absorb it if intrinsic factor is lacking or if there is impaired absorption in the ileum. For this reason, increasing dietary cobalamin does not correct the anemia. However, the patient should be instructed about adequate dietary intake to maintain good nutrition. Parenteral or intranasal administration of cobalamin is the treatment of choice. Without cobalamin administration, these individuals will die in 1 to 3 years.

You encourage the patient with cobalamin (B12) deficiency to seek treatment because untreated pernicious anemia may result in a. Death. b. Liver failure. c. Heart failure. d. Gastrectomy.

The nurse is performing discharge dietary teaching for a client with hyperkalemia. Which statement does the nurse include in the teaching? a. "You may eat avocados, broccoli, and cantaloupe." b. "You may use salt substitutes." c. "You may eat apples, strawberries, and peaches." d. "You don't need to restrict dairy products."

C. "You may eat apples, strawberries, and peaches." The client with hyperkalemia should be instructed to consume foods low in potassium such as apples, strawberries, and peaches. The client should avoid foods high in potassium, which include avocados, broccoli, cantaloupe, and dairy products. Salt substitutes contain potassium.

A nurse identifies a nursing diagnosis of Excess Fluid Volume related to heart failure as evidenced by edema and weight gain. The nurse reviews the client's laboratory test results. Which plasma osmolality value would support the nurse's nursing diagnosis? a. 310 mOsm/kg b. 280 mOsm/kg c. 260 mOsm/kg d. 300 mOsm/kg

C. 260 mOsm/kg Normal osmolality is 280 to 300 mOsm/kg. Plasma osmolality decreases in water excess and elevates in water deficit. Therefore a result of 260 mOsm/kg would support the diagnosis of excess fluid volume.

Which written order does the nurse clarify with the provider when caring for a client with a serum sodium level of 149 mEq/L? a. Institute seizure precautions. b. Weigh the client daily. c. Place the client on nothing by mouth (NPO) status. d. Monitor intake and output.

C. Place the client on nothing by mouth (NPO) status Ensuring adequate water intake is an important nutritional therapy in the treatment of hypernatremia; the nurse should ask for clarification of the NPO order. The other orders are appropriate in the management of clients with hypernatremia.

A client is diagnosed with body fluid hypoosmolality. Treatment involves restricting his intake of free water. Which fluids would the nurse most likely restrict? Select all that apply. a. Tomato juice b. Milk c. Broth d. Apple juice e. Tea

D,E Management of water excess typically involves free water restriction. Limited fluids include water, coffee, tea, and simple fruit juices such as apple juice. More concentrated fluids such as milk, broth, or tomato juice may be given

A client is being monitored for daily weights. The night nurse asks the nursing assistant for the morning weight, and the assistant replies, "She was sleeping so well, I didn't want to wake her to get her weight." How does the nurse respond? a. "Fast thinking! She really needs to rest after the night she had." b. "Get the information now, or I'll report you for not doing your job." c. "Never mind—I will do it myself." d. "Weigh her now. We need her weight daily, at the same time."

D. "Weigh her now. We need her weight daily, at the same time." The nurse should educate the nursing assistant as to why obtaining the client's weight at the same time each day is important. Although the nursing assistant may be hesitant to wake the client, assessing the client's fluid balance is more important. The responses of telling the nursing assistant to get the information now or she'll be reported or that the nurse will get the information herself do not demonstrate good leadership. The assistant needs to understand the rationale for waking and weighing the client. She should not be dismissed and belittled by the nurse.

ANS: A Placing the client in high Fowler's position, with pillows under the arms, allows for maximum chest expansion.

The nurse is caring for a client with severe heart failure. What is the best position in which to place this client? a. High Fowler's, pillows under arms b. Semi-Fowler's, with legs elevated c. High Fowler's, with legs elevated d. Semi-Fowler's, on the left side

ANS: C Vaso-occlusive crisis, or painful crisis, is caused by obstruction of blood flow by sickle cells, infarctions, and some degrees of vasospasm.

A 3-year-old child with sickle cell disease is admitted to the hospital in sickle cell crisis with severe abdominal pain. The nurse recognizes that the type of crisis the child is most likely experiencing is: a. aplastic. b. hyperhemolytic. c. vaso-occlusive. d. splenic sequestration.

ANS: B Ringer's lactate is an isotonic solution that acts as a volume expander. Also, the lactate acts as a buffer in the presence of acidosis. The other solutions do not contain any substance that would buffer or correct the client's acidosis.

A client who has acidosis resulting from hypovolemic shock has been prescribed intravenous fluid replacement. Which fluid does the nurse prepare to administer? a. Normal saline b. Ringer's lactate c. 5% dextrose in water d. 5% dextrose in 0.45% normal saline

ANS: C The sickle cell gene is inherited from both parents; therefore each offspring has a one in four chance of inheriting the disease.

A newly married couple is seeking genetic counseling because they are both carriers of the sickle cell trait. How can the nurse best explain the children's risk of inheriting this disease? a. Every fourth child will have the disease; two others will be carriers. b. All of their children will be carriers, just as they are. c. Each child has a one in four chance of having the disease and a two in four chance of being a carrier. d. The risk levels of their children cannot be determined by this information.

ANS: B Increased myocardial contractility -- thus increasing CO

A nurse is administering a dopamine infusion at a moderate dose to a client who has severe HF. Which of the following is an expected effect? a. Lowered heart rate b. Increased myocardial contractility c. Decreased conduction through the AV node d. Vasoconstriction of the renal blood vessels

ANS: B Monitoring hourly urine output is included in nursing assistant education and does not require special clinical judgment; the nurse evaluates the results. Obtaining vital signs, monitoring oxygen saturation, and assessing mental status in critically ill clients requires the clinical judgment of the critical care nurse because immediate intervention may be needed.

A postoperative client is admitted to the intensive care unit with hypovolemic shock. Which nursing action does the nurse delegate to an experienced nursing assistant? a. Obtain vital signs every 15 minutes b. Measure hourly urine output c. check oxygen saturation d. Assess level of alertness

ANS: B The client who is in a tripod position and using accessory muscles is working to breathe. This client must be assessed first to establish how well the client is breathing and provide interventions to minimize respiratory failure. The other clients are not in acute distress.

A pulmonary nurse cares for clients who have chronic obstructive pulmonary disease (COPD). Which client should the nurse assess first? a. A 46-year-old with a 30-pack-year history of smoking b. A 52-year-old in a tripod position using accessory muscles to breathe c. A 68-year-old who has dependent edema and clubbed fingers d. A 74-year-old with a chronic cough and thick, tenacious secretions

A client with uncontrolled diabetes develops hypophosphatemia. Which finding would the nurse most likely assess? Select all that apply. a. respiratory muscle weakness b. confusion c. ventricular dysrhythmia d. constipation e. abdominal distention

A,B,C With hypophosphatemia, findings include neuromuscular dysfunction; weakness, especially respiratory muscles; fatigue; myocardial depression; ventricular dysrhythmias; rhabdomyolysis; confusion, coma; decreased oxygen delivery to tissues; renal loss of bicarbonate, calcium, magnesium, and glucose; bone changes (osteomalacia); and endocrine changes (insulin resistance). Abdominal distention and constipation are more commonly associated with hypokalemia.

The nurse is teaching proper nutrition to a client who has been prescribed high-ceiling diuretic therapy. Which client response indicates a need for further teaching? a. "I should eat more eggs and cereals to maintain a high potassium intake." b. "Eating meat will help with my potassium needs." c. "I should eat fish a few times a week." d. "I should have fruits such as oranges, kiwi, and bananas every day."

A. "I should eat more eggs and cereals to maintain a high potassium intake." Breads, eggs, and cereal grains contain the least amount of natural potassium. Meats, fish, fruits, and some vegetables are highest in potassium. Diuretics that increase the kidney excretion of potassium can cause hypokalemia. The client should be encouraged to eat foods high in potassium when taking high-ceiling diuretic agents to help compensate for potassium loss.

A client's electrocardiogram demonstrates a heart rate of 52 beats/min, prolonged PR interval with widened QRS complex, and the client is also hypotensive. The nurse reviews the morning labs for which conditions? a. Hyperkalemia, hypercalcemia, hypermagnesemia b. Hypokalemia, hypocalcemia, hypermagnesemia c. Hypocalcemia, hyperkalemia, hypernatremia d. Hypernatremia, hypercalcemia, hypophosphatemia

A. Hyperkalemia, hypercalcemia, hypermagnesemia Cardiovascular changes are consistent with elevated potassium, calcium, and magnesium. These signs and symptoms may progress to life-threatening cardiac emergency. Although hypocalcemia can interfere with cardiac contractility, hypokalemia and hypophosphatemia do not.

A client is admitted to the hospital with a heart rate of 166 beats/min, increased thirst, restlessness, and agitation. Which electrolyte imbalance does the nurse suspect? a. Hypernatremia b. Hypomagnesemia c. Hypercalcemia d. Hyperphosphatemia

A. Hypernatremia These symptoms are indicative of hypernatremia. Clinical manifestations of hypomagnesemia are seen in the neuromuscular, central nervous, and intestinal systems. Hypercalcemia manifests with an altered level of consciousness that can range from confusion and lethargy to coma, and severe hypercalcemia depresses electrical conduction, slowing heart rate. Hyperphosphatemia causes few direct problems with body function (although hypocalcemia is usually also present).

Which nursing action is recommended when providing care to a client with hypokalemia? a. Question the continued administration of bumetanide (Bumex). b. Obtain the prescribed vial of IV potassium chloride from the pharmacy and dilute before administration. c. Establish a peripheral IV, preferably in the hand, for administering IV potassium chloride. d. Administer prescribed oral potassium chloride (K-Dur) before a meal or snack.

A. Question the continued administration of bumetanide (Bumex). Bumetanide is a loop diuretic, which contributes to potassium loss and should be questioned. The Joint Commission has mandated that all concentrated electrolytes be mixed by a pharmacist and that vials of KCl should not be available in client care areas. A large vein with high blood flow should be accessed to avoid phlebitis; it is recommended that the hand be avoided. Oral potassium supplements should be given with or following a snack or meal to avoid nausea.

Which action does the nurse take first for the client who is admitted to the emergency department (ED) with a panic attack and whose blood gases indicate respiratory alkalosis? a. Encourage the client to take slow breaths b. Obtain a prescription for a fluid and electrolyte infusion c. Administer oxygen using ED standard orders d. Place an emergency cart close to the client's room

ANS: A Because respiratory alkalosis is caused by hyperventilation, the nurse's first action should be to assist the client in slowing the respiratory rate. Respiratory alkalosis is caused by hyperventilation; fluid and electrolyte replacement will not help correct hyperventilation. No evidence suggests that hypoxemia exists; therefore, oxygen is not needed at this time. Ongoing respiratory alkalosis may eventually cause fluid and electrolyte shifts requiring IV emergency drugs; however, slowing the breathing and rebreathing of CO2 are the priority interventions.

A 2-month-old breastfed infant is successfully rehydrated with oral rehydration solutions (ORS) for acute diarrhea. What instructions to the mother about breastfeeding should be included by the nurse? A. Continue breastfeeding. B. Stop breastfeeding until breast milk is cultured. C. Stop breastfeeding until diarrhea is absent for 24 hours. D. Express breast milk and dilute with sterile water before feeding.

ANS: A Breastfeeding should continue. Culturing the breast milk is not necessary. Breastfeeding can continue along with ORS to replace the continuing fluid loss from the diarrhea. Breast milk should not be diluted.

A client asks the nurse if eating only preservative- and dye-free foods will decrease cancer risk. What response by the nurse is best? a. "Maybe; preservatives, dyes, and preparation methods may be risk factors." b. "No; research studies have never shown those things to cause cancer." c. "There are other things you can do that will more effectively lower your risk." d. "Yes; preservatives and dyes are well known to be carcinogens."

ANS: A Dietary factors related to cancer development are poorly understood, although dietary practices are suspected to alter cancer risk. Suspected dietary risk factors include low fiber intake and a high intake of red meat or animal fat. Preservatives, preparation methods, and additives (dyes, flavorings, sweeteners) may have cancer-promoting effects. It is correct to say that other things can lower risk more effectively, but this does not give the client concrete information about how to do so, and also does not answer the client's question.

A client has mild acidosis but after a day has not compensated for it. Which action by the nurse is best? a. Review the client's daily hemoglobin and hematocrit. b. Ask the laboratory to rerun today's arterial blood gases. c. Document the finding and notify the physician. d. Apply 2 L of oxygen via nasal cannula.

ANS: A Hemoglobin is part of the buffering system. Low hemoglobin affects acid-base balance by decreasing the body's ability to compensate for mild acidosis. Rerunning the specimen would take time and might require another sample. The nurse may need to notify the physician but would need more information to report, such as hemoglobin and hematocrit values. Adding 2 liters of oxygen would not help the client as much as he or she would be helped if the cause of the refractory acidosis was determined.

The nurse is caring for a critically ill client with septic shock. The serum lactate level is 6.2. For which acid-base disturbance should the nurse assess? a. Metabolic acidosis b. Metabolic alkalosis c. Respiratory acidosis d. Respiratory alkalosis

ANS: A Increased lactate levels are associated with hypoxia and metabolic acidosis secondary to anaerobic metabolism. Metabolic alkalosis is related to bicarbonate therapy, diuretic use, vomiting, and nasogastric suction. Respiratory acidosis is caused by CO2 retention and impaired pulmonary function, which is inconsistent with elevated lactate levels. Respiratory alkalosis is caused by excessive loss of CO2 through hyperventilation, inconsistent with elevated lactate levels.

A nurse is providing community education on the seven warning signs of cancer. Which signs are included? (Select all that apply.) a. A sore that does not heal b. Changes in menstrual patterns c. Indigestion or trouble swallowing d. Near-daily abdominal pain e. Obvious change in a mole

ANS: A, B, C, E The seven warning signs for cancer can be remembered with the acronym CAUTION: changes in bowel or bladder habits, a sore that does not heal, unusual bleeding or discharge, thickening or lump in the breast or elsewhere, indigestion or difficulty swallowing, obvious change in a wart or mole, and nagging cough or hoarseness. Abdominal pain is not a warning sign.

A preschooler with severe vomiting and diarrhea was admitted to the hospital. The vomiting has stopped and rehydration has begun intravenously. When should the nurse begin feeding the child solid food? A. When the parents give their permission to feed their child B. After the child has been rehydrated C. After the diarrhea has stopped for 24 hours D. When the IV rehydration can be stopped

ANS: B It is not up to the parents to decide when resumption of solid food begins. Feeding of solids or formula is started as soon as the child is rehydrated. Children should be encouraged to eat frequently—every 3 to 4 hours. Parents should be instructed that although stool output may increase, feeding will not prolong diarrhea, and the child will be absorbing necessary nutrients and calories. Parents should be instructed that although stool output may increase, feeding will not prolong diarrhea, and the child will be absorbing necessary nutrients and calories. The intravenous solutions may run a little longer to ensure that the child remains hydrated.

A nurse is caring for a client who is experiencing excessive diarrhea. The client's arterial blood gas values are pH 7.28, PaO2 98 mm Hg, PaCO2 45 mm Hg, and HCO3- 16 mEq/L. Which provider order should the nurse expect to receive? a. Furosemide (Lasix) 40 mg intravenous push b. Sodium bicarbonate 100 mEq diluted in 1 L of D5W c. Mechanical ventilation d. Indwelling urinary catheter

ANS: B This client's arterial blood gas values represent metabolic acidosis related to a loss of bicarbonate ions from diarrhea. The bicarbonate should be replaced to help restore this client's acid-base balance. Furosemide would cause an increase in acid fluid and acid elimination via the urinary tract; although this may improve the client's pH, the client has excessive diarrhea and cannot afford to lose more fluid. Mechanical ventilation is used to treat respiratory acidosis for clients who cannot keep their oxygen saturation at 90%, or who have respirator muscle fatigue. Mechanical ventilation and an indwelling urinary catheter would not be prescribed for this client.

The nurse is conducting a community health education class on diet and cancer risk reduction. What should be included in the discussion? (Select all that apply.) a. Limit sodium intake. b. Avoid beef and processed meats. c. Increase consumption of whole grains. d. Eat "colorful fruits and vegetables," including greens. e. Avoid gas-producing vegetables such as cabbage.

ANS: B, C, D Consuming bran and whole grains and avoiding red meat and processed foods such as lunchmeats can reduce cancer risk. Consuming foods high in vitamin A, including apricots, carrots, and leafy green and yellow vegetables, can also reduce cancer risk. Reducing sodium is helpful in the treatment of hypertension and heart and kidney failure; no evidence suggests that lowering of sodium intake decreases the incidence of cancer. Eating cruciferous vegetables such as broccoli, cauliflower, Brussels sprouts, and cabbage may actually reduce cancer risk.

The nurse is instructing a client on how to perform breast self-examination (BSE). Which techniques will the nurse include in teaching the client about BSE? Select all that apply. a. Instruct the client to keep her arm by her side while performing the examination. b. Ensure that the setting in which BSE is demonstrated is private and comfortable. c. Ask the client to remove her shirt. The bra may be left in place. d. Ask the client to demonstrate her own method of BSE. e. Use the fingertips, which are more sensitive than the finger pads, to palpate the breasts.

ANS: B, D The setting should be private and comfortable to promote an environment conducive to learning and to prevent potential client embarrassment. Before teaching breast palpation, ask the client to demonstrate her own method, so that the nurse can assess the client's understanding of BSE. For better visualization, the arm should be placed over the head.The client should undress from the waist up. The finger pads, which are more sensitive than the fingertips, are used when palpating the breasts.

A patient with breast cancer asks the nurse why 6 weeks of daily radiation treatments is necessary. What is the nurse's best response? a. "Your cancer is widespread and requires more than the usual amount of radiation treatment." b. "The cost of larger doses of radiation for a shorter period of time is justified by the results." c. "Research has shown more cancer cells are killed if the radiation is given in smaller doses over a longer period of time." d. "It is less likely your hair will fall out or you will become anemic if radiation is given in smaller doses over a longer period of time."

ANS: C Because of the varying responses of all the cancer cells within a given tumor, smaller doses of radiation given on a daily basis for a set period of time provides multiple opportunities for the destruction of cancer cells while minimizing damage to normal tissues.

The nursing instructor explains the difference between normal cells and benign tumor cells. What information does the instructor provide about these cells? a. Benign tumors grow through invasion of other tissue. b. Benign tumors have lost their cellular regulation from contact inhibition. c. Growing in the wrong place or time is typical of benign tumors. d. The loss of characteristics of the parent cells is called anaplasia.

ANS: C Benign tumors are basically normal cells growing in the wrong place or at the wrong time. Benign cells grow through hyperplasia, not invasion. Benign tumor cells retain contact inhibition. Anaplasia is a characteristic of cancer cells.

Which assessment is most relevant to the care of an infant with dehydration? a. Temperature, heart rate, and blood pressure. b. Respiratory rate, oxygen saturation, and lung sounds. c. Heart rate, sensorium, and skin color. d. Diet tolerance, bowel function, and abdominal girth.

ANS: C Children can compensate and maintain an adequate cardiac output when they are hypovolemic. Blood pressure is not as reliable an indicator of shock as are changes in heart rate, sensorium, and skin color. Respiratory assessments will not provide data about impending hypovolemic shock. Changes in heart rate, sensorium, and skin color are early indicators of impending shock in the child. Diet tolerance, bowel function, and abdominal girth are not as important indicators of shock as heart rate, sensorium, and skin color.

ANS: A A positive inotrope is a medication that increases the strength of the heart's contractions. The other options are not correct.

The provider requests the nurse start an infusion of an inotropic agent on a client. How does the nurse explain the action of these drugs to the client and spouse? a. "It increases the force of the heart's contractions." b. "It dilates vessels, which lessens the work of the heart." c. "It slows the heart rate down for better filling." d. "It constricts vessels, improving blood flow."

A nurse reviews the medication list of a client recovering from a computed tomography (CT) scan with IV contrast to rule out small bowel obstruction. Which medication should alert the nurse to contact the provider and withhold the prescribed dose? a. Pioglitazone (Actos) b. Glimepiride (Amaryl) c. Glipizide (Glucotrol) d. Metformin (Glucophage)

ANS: D Glucophage should not be administered when the kidneys are attempting to excrete IV contrast from the body. This combination would place the client at high risk for kidney failure. The nurse should hold the metformin dose and contact the provider. The other medications are safe to administer after receiving IV contrast.

What would be important for the nurse to include in the teaching plan for clients who are taking insulin? a. The client should use only the injection sites that are most accessible. b. During times of illness, clients should increase their insulin dosage by 25%. c. When mixing insulins, the NPH insulin should be drawn up into the syringe first. d. When mixing insulins, regular insulin should be drawn up into the syringe first.

ANS: D If mixing insulins, the regular insulin should always be drawn up into the syringe first. Remember: clear to cloudy; regular insulin first, followed by cloudy ones, such as NPH and Ultralente. Clients should always rotate injection sites (preferably in the abdomen) and should notify their physicians if they become ill.

Which condition or manifestation in the client with a serum sodium level of 149 mEq/L indicates to the nurse that this electrolyte imbalance may be caused by excessive fluid loss? a. The client has twitching muscle contractions in the lower extremities b. The client's skin is cool and clammy c. The urine specific gravity is increased d. The hematocrit is 52%

ANS: D The serum sodium level is elevated, indicating hypernatremia. The elevation could be from an actual increase in sodium or from a loss of fluids only. A relative hypernatremia can occur as a result of dehydration (excessive fluid loss) without sodium loss. Such dehydration is usually accompanied by hemoconcentration. The higher than normal hematocrit suggests hemoconcentration.

The benefits of using an insulin pump include all of the following except: a. By continuously providing insulin they eliminate the need for injections of insulin b. They simplify management of blood sugar and often improve A1C c. They enable exercise without compensatory carbohydrate consumption d. They help with weight loss

ANS: D Using an insulin pump has many advantages, including fewer dramatic swings in blood glucose levels, increased flexibility about diet, and improved accuracy of insulin doses and delivery; however, the use of an insulin pump has been associated with weight gain.

A client has been NPO after a colectomy with nasogastric (NG) suction in place. On assessment, the nurse finds the client reporting cramps in the calves. Which action by the nurse is most appropriate? a. Document findings and notify the physician. b. Stop suction and request that the laboratory draw arterial blood gases. c. Prepare to administer lorazepam (Ativan). d. Raise the siderails and notify the physician.

ANS: D The client has a metabolic alkalosis probably caused by prolonged suctioning. The client also is experiencing tetany, caused by the accompanying hypocalcemia, and is at risk for seizures. The priority is to maintain the client's safety; this includes raising the siderails and then notifying the physician. Documentation is important but not as important as providing safety. The nurse would not stop the suction without an order. The client may need lorazepam if he or she has seizures, but this is not the first action the nurse would perform.

ANS: C Salmeterol is designed to prevent an asthma attack; it does not relieve or reverse symptoms. Salmeterol has a slow onset of action; therefore, it should not be used as a rescue drug. The drug must be shaken well because it has a tendency to separate easily. Poor technique on the client's part allows the drug to escape through the nose and mouth.

After teaching a client who is prescribed salmeterol (Serevent), the nurse assesses the client's understanding. Which statement by the client indicates a need for additional teaching? a. "I will be certain to shake the inhaler well before I use it." b. "It may take a while before I notice a change in my asthma." c. "I will use the drug when I have an asthma attack." d. "I will be careful not to let the drug escape out of my nose and mouth."

ANS: B The nurse responds with the most informative, accurate response. The decision not to use antibiotics for viral pneumonia was based on sound rationale about the etiology of the illness, not cost.

An 8-year-old child is diagnosed with viral pneumonia and sent home from the clinic with no antibiotic prescription. The symptoms worsen, and the child returns to the clinic a week later with signs of a higher fever, listlessness, and a harsh, productive cough. The child's mother states, "I knew a prescription for antibiotics was needed." Which response by the nurse is the most appropriate? a. "It is better to wait to make sure so we don't use antibiotics unnecessarily. This approach also saves healthcare dollars." b. "Antibiotics are not effective for viral pneumonia. Bacteria can grow later in the course of the illness, requiring the need for antibiotics at that time." c. "You do not want to expose your child to medication unnecessarily. Now it is necessary, because it is bacterial pneumonia." d. "Sometimes we just do not know. I'm glad you came back in."

ANS: C The low pH, the elevated carbon dioxide level, and the low oxygen concentration all indicate that the client is experiencing poor gas exchange and has acidosis. The low pH and the low oxygen concentration could occur without hypercarbia. Only the elevated carbon dioxide concentration confirms hypercarbia.

Which blood gas value indicates that the client is experiencing hypercarbia? a. pH = 7.33 b. Bicarbonate = 20 mEq/L c. PaCO2 = 60 mm Hg d. PaO2 = 80 mm Hg

A client admitted to the facility is diagnosed with metabolic alkalosis based on arterial blood gas values. When obtaining the client's history, which statement would the nurse interpret as a possible underlying cause? a. I was breathing so fast because I was so anxious and in so much pain." b. "I've been taking antacids almost every 2 hours over the past several days." c. "I've had a fever for the past 3 days that just doesn't seem to go away." d. "I've had a GI virus for the past 3 days with severe diarrhea."

B. "I've been taking antacids almost every 2 hours over the past several days." Metabolic alkalosis occurs when there is excessive loss of body acids or with unusual intake of alkaline substances. It can also occur in conjunction with an ECF deficit or potassium deficit (known as contraction alkalosis). Vomiting or vigorous nasogastric suction frequently causes metabolic alkalosis. Endocrine disorders and ingestion of large amounts of antacids are other causes. Hyperventilation, commonly caused by anxiety or pain, would lead to respiratory alkalosis. Fever, which increases carbon dioxide excretion, would also be associated with respiratory alkalosis. Severe diarrhea is associated with metabolic acidosis.

A 77-year-old woman is brought to the emergency department by her family after she has had diarrhea for 3 days. The family tells the nurse that she has not been eating or drinking well, but that she has been taking her diuretics for congestive heart failure (CHF). She is receiving lactated Ringer's solution IV for rehydration. What clinical manifestations does the nurse monitor during rehydration of the client? Select all that apply. a. Blood serum glucose b. Blood pressure c. Pulse rate and quality d. Urinary output e. Urine specific gravity levels

B. Blood pressure C. Pulse rate and quality D. Urinary output E. Urine specific gravity levels The two most important areas to monitor during rehydration are pulse rate and quality and urine output; however, decreasing specific gravity of urine is also an indication of rehydration. Blood pressure is also important to monitor during rehydration. Blood glucose changes do not have a direct relation to a client's rehydration status.

A client develops fluid overload while in the intensive care unit. Which nursing intervention does the nurse perform first? a. Draws blood for laboratory tests b. Elevates the head of the bed c. Places the extremities in a dependent position d. Puts the client in a side-lying position

B. Elevates the head of the bed Elevating the head of the bed will ease breathing for the client, so it should be done first. Although drawing blood for laboratory tests may be indicated, the nurse should perform interventions that will help with physiological changes caused by fluid overload first. Placing the extremities in a dependent position increases peripheral edema, and positioning the client in a side-lying position increases the work of breathing.

Which order does the nurse clarify with the provider based on the following available client data? Sodium 149 mEq/L Potassium 4.7 mEq/L Glucose 112 mg/dL Serum osmolality 316 mOsm/L HR 112 bpm RR 22 bpm BP 96/62 mm Hg Temp 38.1 C New-onset confusion Follows simple commands Reports fatigue Dry mucous membranes Presence of "tenting" of skin on back of hand a. Measure intake and output for 48 hours. b. No activity restrictions; ambulate ad lib. c. Encourage oral rehydration/intake. d. Report urine output less than 0.5 mL/kg/hr.

B. No activity restrictions; ambulate ad lib. The client data suggest dehydration. Safety is a priority concern in the care of this client. The nurse should clarify the activity order as fall precautions and activity limitations may be indicated for safety. The other orders are appropriate in the care of a client with dehydration.

The nurse is administering sodium chloride 0.9% (normal saline) intravenously to a client who is dehydrated. Which assessments does the nurse perform to evaluate the effectiveness of rehydration therapy? Select all that apply. a. Temperature b. Pulse rate and quality c. Neck vein distention d. Urinary output e. Bowel sounds

B. Pulse rate and quality D. Urinary output Pulse rate and quality as well as urinary output best reflect improving volume status with rehydration therapy. Temperature, neck vein distention, and bowel sounds are not indicators of an improving volume state.

When caring for a client with hyponatremia, which intervention does the nurse implement? a. Intravenous administration of 0.45% normal saline b. Administration of furosemide (Lasix) c. Small-volume intravenous infusions of 3% normal saline d. 2-gram sodium diet

C. Small-volume intravenous infusions of 3% normal saline 3% saline is hypertonic and is given in small volumes to replenish serum sodium. 0.45% saline is hypotonic and will further dilute serum sodium levels. Furosemide (Lasix) causes sodium loss in the kidneys and would further contribute to hyponatremia. A 2-gram sodium diet restricts sodium intake; the goal of nutritional therapy with hyponatremia is to increase sodium intake.

After receiving change-of-shift report, which client does the RN assess first? a. 26-year-old with nausea and vomiting who complains of dizziness when standing b. 36-year-old with a nasogastric (NG) tube who has dry oral mucosa and is complaining of thirst c. 46-year-old receiving IV diuretics whose blood pressure is 95/52 mm Hg d. 56-year-old with normal saline infusing at 150 mL/hr whose hourly urine output has been averaging 75 mL

C. 46-year-old receiving IV diuretics whose blood pressure is 95/52 mm Hg The client with the history of receiving IV diuretics and having low blood pressure may be experiencing hypoperfusion caused by hypovolemia, and immediate assessment and interventions are needed. The client with nausea and vomiting, the client with an NG tube complaining of thirst, and the client receiving normal saline with an hourly urine output of 75 mL/hr have problems that are not urgent at this time.

The charge nurse on a medical-surgical unit is completing assignments for the day shift. Which client is assigned to the LPN/LVN? a. 44-year-old with congestive heart failure (CHF) who has gained 3 pounds since the previous day b. 58-year-old with chronic renal failure (CRF) who has a serum potassium level of 6 mEq/L c. 76-year-old with poor skin turgor who has a serum osmolarity of 300 mOsm/L d. 80-year-old with 3+ peripheral edema and crackles throughout the posterior chest

C. 76-year-old with poor skin turgor who has a serum osmolarity of 300 mOsm/L Although the 76-year-old client has poor skin turgor, the serum osmolarity indicates that fluid balance is normal; this client is the most stable of the four clients described and can be assigned to the LPN/LVN. The data about the 44-year-old with CHF who has gained 3 pounds since the previous day indicate reduced stability; assessments and interventions performed by an RN are needed. The data about the 58-year-old client with CRF and a serum potassium level of 6 mEq/L indicate reduced stability; assessments and interventions performed by an RN are needed. The data about the 80-year-old client with edema and congested lungs indicate that the client is not stable, and that assessments and interventions by an RN are needed.

A client reports painful muscle spasms in the lower legs at rest, a tingling sensation in the hands and lips, and abdominal cramping and diarrhea. The nurse reviews the client's laboratory results for the presence of which disorder? a. Hypophosphatemia b. Hypermagnesemia c. Hypocalcemia d. Hypernatremia

C. Hypocalcemia The primary symptoms of hypocalcemia are neuromuscular changes, specifically painful muscle cramps, and paresthesias that may spread to the face, progressing to tetany. Abdominal cramping and diarrhea may also occur. Muscle spasms in lower legs at rest, tingling sensation in the hands and lips, and abdominal cramping and diarrhea are not primary characteristics of hypernatremia, hypermagnesemia, or hypophosphatemia.

A 68-year-old man is admitted to the hospital with dehydration. He has a history of atrial fibrillation, congestive heart failure (CHF), and hypertension. His current medications are digoxin (Lanoxin), chlorothiazide (Diuril), and oral potassium supplements. He tells the nurse that he has had flulike symptoms for the past week and has been unable to drink for the past 48 hours. The health care provider requests laboratory specimens to be drawn and an isotonic IV to be started. Which IV fluid does the nurse administer? a. 0.45% saline b. 5% dextrose in 0.45% saline c. 5% dextrose in Ringer's lactate d. 5% dextrose in water (D5W)

D. 5% dextrose in water (D5W) 5% dextrose in water (D5W) is an isotonic solution. 0.45% saline is a hypotonic solution, while 5% dextrose in 0.45% saline and 5% dextrose in Ringer's lactate are hypertonic solutions.

The nurse instructs an older adult client to increase intake of dietary potassium when the client is prescribed which classification of drugs? a. Alpha antagonists b. Beta blockers c. Corticosteroids d. High-ceiling (loop) diuretics

D. High-ceiling (loop) diuretics High-ceiling (loop) diuretics are potassium-depleting drugs. The client should increase intake of dietary potassium to compensate for this depletion. Alpha antagonists, beta blockers, and corticosteroids are not potassium-depleting drugs.

Which newly written prescription does the nurse administer first? a. Intravenous (IV) normal saline to a client with a serum sodium of 132 mEq/L b. Oral calcium supplements to a client with severe osteoporosis c. Oral phosphorus supplements to a client with acute hypophosphatemia d. Oral potassium chloride (KCl) to a client whose serum potassium is 3 mEq/L

D. Oral potassium chloride (KCl) to a client whose serum potassium is 3 mEq/L Because minor changes in serum potassium level can cause life-threatening dysrhythmias, the first priority should be to administer potassium supplements to the client with hypokalemia. The electrolyte disturbance (sodium level of 132 and low phosphorus level) and the need for calcium in this client are not immediately life-threatening.

ANS: D This condition is known as glossitis, and is characteristic of B12 anemia. If the anemia is a pernicious anemia, it is treated with cobalamin. Genetic testing is not a priority for this condition. The client does not need high-fiber foods or protective precautions.

The nurse assesses a client's oral cavity and finds the tongue to be swollen with a smooth appearance. What action by the nurse is most appropriate? a. Encourage the client to have genetic testing. b. Instruct the client on high-fiber foods. c. Place the client in protective precautions. d. Teach the client about cobalamin therapy.

An older adult admitted with dehydration and a history of stress incontinence expresses embarrassment about the disorder and the need for absorbent undergarments. Which question about nutritional metabolic needs would be best to ask this client related to the reason for admission? a. What is your typical urinary elimination pattern and amount? b. How is your appetite? c. Have you noticed a change in the tightness of your shoes? d. What is your typical daily fluid intake and what types of fluids do you drink?

D. What is your typical daily fluid intake and what types of fluids do you drink? Asking the client about the amount and types of fluids will provide additional information as to how the client deals with incontinence through fluid management and possible causes for the dehydration. Asking the client about appetite, urinary pattern, and tightness of shoes is not helpful in assessing the client's current health problem.

ANS: B A systolic ejection murmur that may be accompanied by a palpable thrill is a manifestation of pulmonary stenosis. The classic murmur associated with patent ductus arteriosus is a machinery-like one that can be heard through both systole and diastole. The characteristic murmur associated with ventricular septal defect is a loud, harsh, holosystolic murmur. A systolic murmur that is accompanied by an ejection click may be heard on auscultation when coarctation of the aorta is present.

The nurse assessing a premature newborn infant auscultates a continuous machinery-like murmur. This finding is associated with which congenital heart defect? a. Pulmonary stenosis b. Patent ductus arteriosus c. Ventricular septal defect d. Coarctation of the aorta

ANS: C Clients with left-sided heart failure report weakness or fatigue while performing normal activities of daily living, as well as difficulty breathing, or "catching their breath." This occurs as fluid moves into the alveoli. Nocturia is often seen with right-sided heart failure. Thirst and blurred vision are not related to heart failure.

The nurse is assessing a client in an outpatient clinic. Which client statement alerts the nurse to possible left-sided heart failure? a. "I have been drinking more water than usual." b. "I have been awakened by the need to urinate at night." c. "I have to stop halfway up the stairs to catch my breath." d. "I have experienced blurred vision on several occasions."

ANS: C Pulmonic stenosis is an obstruction to blood flowing from the ventricles. Tricuspid atresia results in decreased pulmonary blood flow. The atrial septal defect results in increased pulmonary blood flow. Blood flows from the left atrium (higher pressure) into the right atrium (lower pressure) and then to the lungs via the pulmonary artery. Transposition of the great arteries results in mixed blood flow.

Which defect results in increased pulmonary blood flow? a. Pulmonic stenosis b. Tricuspid atresia c. Atrial septal defect d. Transposition of the great arteries

ANS: D Saline has been found to cause more side effects when suctioning and does not increase the amount of secretions removed.

Which of the following statements made by a student nurse indicates the need for further teaching about suctioning a patient with an endotracheal tube? a. "Suctioning the patient requires sterile technique." b. "I'll apply suction while rotating and withdrawing the suction catheter." c. "I'll suction the mouth after I suction the endotracheal tube." d. "I'll instill 5 mL of normal saline into the tube before hyperoxygenating the patient."

ANS: D The vasodilating effects of this drug frequently cause clients to have headaches during the initial period of therapy. Clients should be told about this side effect and encouraged to take the medication with food. Some clients obtain relief with mild analgesics, such as acetaminophen.

The client who just started taking isosorbide dinitrate (Imdur) reports a headache. What is the nurse's best action? a. Titrate oxygen to relieve headache. b. Hold the next dose of Imdur. c. Instruct the client to drink water. d. Administer PRN acetaminophen.

ANS: B Because the client is at risk for pulmonary edema and hypoxemia, the first action should be to assess breath sounds. Assessment of edema may be delayed until after breath sounds are assessed. After a full assessment, the nurse should notify the health care provider. After physiologic stability is attained, then ask the client about behaviors that may have caused the weight gain, such as increased sodium intake or changes in medications.

The home health nurse visits a client with heart failure who has gained 5 pounds in the past 3 days. The client states, "I feel so tired and short of breath." Which action does the nurse take first? a. Assess the client for peripheral edema b. Auscultate the client's posterior breath sounds c. Notify the health care provider about the client's weight gain. d. Remind the client about dietary sodium restrictions

ANS: A The presence of an S3 gallop is an early diastolic filling sound indicative of increasing left ventricular pressure and left ventricular failure. The other actions are not warranted.

The nurse assesses a client and notes the presence of an S3 gallop. What is the nurse's best intervention? a. Assess for symptoms of left-sided heart failure. b. Document this as a normal finding. c. Call the health care provider immediately. d. Transfer the client to the intensive care unit.

ANS: A Excess fluid volume may not be overtly visible. Weight changes may indicate fluid retention. Weighing the infant on the same scale at the same time each day ensures consistency. An excessive weight gain for an infant is an increase of more than 50 g/day. With fluid volume excess, skin will be edematous. The infant's position should be changed frequently to prevent undesirable pooling of fluid in certain areas. Lanoxin is used in the treatment of congestive heart failure to improve cardiac function. Diuretics will help the body get rid of excess fluid.

What intervention should be included in the plan of care for an infant with the nursing diagnosis of Excess Fluid Volume related to congestive heart failure? a. Weigh the infant every day on the same scale at the same time. b. Notify the physician when weight gain exceeds more than 20 g/day. c. Put the infant in a car seat to minimize movement. d. Administer digoxin (Lanoxin) as ordered by the physician.

ANS: D Orthostatic hypotension is not present with coarctation of the aorta. Systolic hypertension may be detected in the upper extremities. The left arm may not accurately reflect systolic hypertension because the left subclavian artery can be involved in the coarctation. The classic finding in children with coarctation of the aorta is a disparity in pulses and blood pressures between the upper and lower extremities.

What is an expected assessment finding in a child with coarctation of the aorta? a. Orthostatic hypotension b. Systolic hypertension in the lower extremities c. Blood pressure higher on the left side of the body d. Disparity in blood pressure between the upper and lower extremities

ANS: B As the client's PaO2 rises, the client's color and pulse oximetry improve and cannot be used to determine hypoventilation. As the client's PaO2 rises, respirations decrease in depth and rate, indicating hypoventilation.

Which clinical manifestation alerts you to the presence of hypoventilation when you are monitoring a client with chronic lung disease and hypercarbia who is receiving oxygen therapy? a. Coarse crackles and wheezes on auscultation b. Slow, shallow respirations c. Pulse oximetry of 90% d. Clubbing of the fingers

ANS: A Eggs and whole grain breads are high in iron. The other choices are appropriate for other nutritional deficiencies but are not the best choice for a patient with iron-deficiency anemia.

Which menu choice indicates that the patient understands the nurse's teaching about best dietary choices for iron-deficiency anemia? a. Omelet and whole wheat toast b. Cantaloupe and cottage cheese c. Strawberry and banana fruit plate d. Cornmeal muffin and orange juice

Which nursing diagnosis would the nurse make based on the effects of fluid and electrolyte imbalance on human functioning? a) Risk for Infection related to inadequate personal hygiene b) Acute Confusion related to cerebral edema c) Pain related to surgical incision d) Constipation related to immobility

b) Acute Confusion related to cerebral edema Edema in and around the brain increases intracranial pressure, leading to the likelihood of confusion. Constipation related to immobility, Pain related to surgical incision, Risk for Infection related to inadequate personal hygiene are nursing diagnoses that have no connection to fluid and electrolyte imbalance.

A patient has been encouraged to increase fluid intake. Which measure would be most effective for the nurse to implement? a) Explaining the mechanisms involved in transporting fluids to and from intracellular compartments b) Keeping fluids readily available for the patient c) Emphasizing the long-term outcome of increasing fluids when the patient returns home d) Planning to offer most daily fluids in the evening

b) Keeping fluids readily available for the patient Having fluids readily available helps promote intake. Explanation of the fluid transportation mechanisms (a) is inappropriate and does not focus on the immediate problem of increasing fluid intake. Meeting short-term outcomes rather than long-term ones (c) provides further reinforcement, and additional fluids should be taken earlier in the day.

A nurse carefully assesses the acid-base balance of a patient who is unable to effectively control his carbonic acid supply. This is most likely a patient with damage to which of the following? a) Kidneys b) Lungs c) Adrenal glands d) Blood vessels

b) Lungs The lungs are the primary controller of the body's carbonic acid supply and thus, if damaged, can affect acid-base balance. The kidneys are the primary controller of the body's bicarbonate supply. The adrenal glands secrete catecholamines and steroid hormones. The blood vessels act only as a transport system.

An intravenous hypertonic solution containing dextrose, proteins, vitamins, and minerals is known as a) Volume expander b) Total parenteral nutrition c) Blood transfusion therapy d) Cellular hydration

b) Total parenteral nutrition Total parenteral nutrition is a hypertonic solution containing 20% to 50% dextrose, proteins, vitamins, and minerals that is administered into the venous system.

A home care nurse is teaching a client and family about the importance of a balanced diet. The nurse determines that the education was successful when the client identifies which of the following as a rich source of potassium? a) Bread products b) Processed meat c) Apricots d) Dairy products

c) Apricots Apricots are a rich source of potassium. Dairy products are rich sources of calcium. Processed meat and bread products provide sodium.

The primary extracellular electrolytes are: a) phosphorous, calcium, and phosphate. b) potassium, phosphate, and sulfate. c) sodium, chloride, and bicarbonate. d) magnesium, sulfate, and carbon.

c) sodium, chloride, and bicarbonate. The primary extracellular electrolytes are sodium, chloride, and bicarbonate.

A 70-year-old client is scheduled for a colonoscopy and is prescribed a bowel preparation solution. The nurse would be alert for which potential imbalance? Select all that apply. a) Hypophosphatemia b) Hypercalemia c) Hypocalcemia d) Hyperphosphatemia e) Hypokalemia f) Hyperkalemia

c, d, e Older adults are at increased risk for electrolyte imbalances during and after bowel preparation for procedures such as a colonoscopy or barium enema. Research has shown that bowel preparation solutions in clients over age 65 years are associated with vascular volume deficit, hyperphosphatemia, hypokalemia, and hypocalcemia.

A client has the following arterial blood gas results: pH: 7.33 PaCO2: 42 mm Hg HCO3: 19 mEq/L PaO2: 95 mm Hg Which imbalance would the nurse suspect? a) Metabolic alkalosis b) Respiratory acidosis c) Respiratory alkalosis d) Metabolic acidosis

d) Metabolic acidosis The results reveal metabolic acidosis, which is characterized by a pH lower than 7.35 and a plasma HCO3 concentration lower than 22 mEq/L. Respiratory acidosis is indicated by a low pH accompanied by an increased arterial concentration of carbon dioxide, which often is clinically defined as a PaCO2 of greater than 45 mm Hg. Respiratory alkalosis is present when a high pH is accompanied by a blood carbon dioxide concentration lower than 35 mm Hg. Metabolic alkalosis is characterized by a pH higher than 7.45 and a plasma HCO3 concentration above 26 mEq/L.

A nurse is initiating a peripheral venous access IV infusion for a patient. Following the procedure, the nurse observes that the fluid does not flow easily into the vein and the skin around the insertion site is edematous and cool to the touch. What would be the nurse's next action related to these findings? a) Reposition the extremity and raise the height of the IV pole. b) Apply pressure to the dressing on the IV. c) Pull the catheter out slightly and reinsert it. d) Put on gloves; remove the catheter; apply pressure with a sterile pad.

d) Put on gloves; remove the catheter; apply pressure with a sterile pad. This IV has been infiltrated. The nurse should put on gloves and remove the catheter. The nurse should also apply pressure with a sterile gauze pad, secure the gauze with tape over the insertion site, and restart the IV in a new location.

ANS: A Depression can occur in clients with heart failure, especially older adults. Having the client talk about his or her feelings will help the nurse focus on the actual problem. Open-ended statements allow the client to respond safely and honestly.

An older adult client with heart failure states, "I don't know what to do. I don't want to be a burden to my daughter, but I can't do it alone. Maybe I should die." What is the nurse's best response? a. "Would you like to talk about this more?" b. "You're lucky to have such a devoted daughter." c. "You must feel as though you are a burden." d. "Would you like an antidepressant medication?"

ANS: C All of the children are acutely ill. A child with asthma who was wheezing and now has decreased breath sounds is acutely ill. This child's ability to move air is decreasing and is approaching respiratory arrest. Intubation protects the airway from closing in epiglottitis and a chest tube is the treatment for tension pneumothorax in a different room; therefore these children are stable. The infant with RSV is sleeping with a normal respiratory rate so there is no immediate danger here.

Following assessment, the nurse anticipates potential respiratory arrest for which child? a. A 5-month-old infant with RSV who is sleeping and has a respiratory rate of 24. b. A 2-year-old with epiglottitis who was intubated in the emergency department. c. A 6-year-old with asthma who was previously wheezing and now has decreased breath sounds. d. A 4-year-old, status post-tension pneumothorax from a motor vehicle accident with a chest tube in place, who complains of pain.

ANS: A, B, C, & D The CBC count is monitored. Infections are common with an elevated white blood cell (WBC) count, and anemia may occur with low hemoglobin and red blood cell (RBC) levels. Oxygen may be administered to treat hypoxia and control sickling. Rest may be instituted to reduce metabolic requirements and deep vein thrombosis prophylaxis (using anticoagulants) prescribed. Transfusion therapy is indicated when an aplastic crisis occurs. Patients may require iron chelation therapy to reduce transfusion-produced iron overload. Pain occurring during an acute crisis usually is undertreated. Patients should have optimal pain control with opioid analgesics, nonsteroidal antiinflammatory agents, antineuropathic pain medications, local anesthetics, or nerve blocks.

The nursing management of a patient in sickle cell crisis includes (select all that apply) a. monitoring of the complete blood cell (CBC) count. b. blood transfusions if required and iron chelation. c. optimal pain management and oxygen therapy. d. rest as needed and deep vein thrombosis prophylaxis.

ANS: D An adequate dosage of iron turns the stools a tarry green color.

The parent of a child receiving an iron preparation tells the nurse that the child's stools are a tarry green color. The nurse should explain that this is a/an: a. Symptom of iron-deficiency anemia. b. Adverse effect of the iron preparation. c. Indicator of an iron preparation overdose. d. Normally expected change caused by the iron preparation.

Which of these does the nurse recognize as the goal of palliative surgery for the client with cancer? a. Cure of the cancer b. Relief of symptoms or improved quality of life c. Allowing other therapies to be more effective d. Prolonging the client's survival time

ANS: B The focus of palliative surgery is to improve quality of life during the survival time.

Which nursing action does the nurse include in the care of an alert older adult who is mildly dehydrated? a. Advising the client and family that strict bed rest is recommended due to fall risk. b. Considering dietary restrictions and ability to swallow, and offering oral fluids every 2 hours. c. Restricting oral fluids if the client is incontinent. d. Assessing weight and vital signs every 4 hours.

B. Considering dietary restrictions and ability to swallow, and offering oral fluids every 2 hours. Oral fluid replacement is a priority when correcting mild to moderate dehydration in an alert client who can swallow. Because risk for falls is increased, the client should be offered assistance when ambulating. Bedrest is not recommended and may even contribute to other complications. Oral fluids should not be withheld due to incontinence. Initially, it is recommended to assess vital signs every 2 hours and weigh the client every 8 hours.

ANS: D All current assessment findings are important. However, the pain in the hip, the slow capillary refill, and the yellow appearance of the roof of the mouth are related to the crises and are expected. The facial drooping as a new finding indicates the possibility of reduced brain perfusion and stroke. This new development requires immediate attention and intervention.

Which new assessment finding in a client with sickle cell disease who currently is in crises does the nurse report immediately to the health care provider? a. Pain in the right hip with limited range of motion b. Slow capillary refill in the toes of the right foot c. Yellow appearance of the roof of the mouth d. Facial drooping on the right side

Which electrolyte imbalance does the nurse anticipate in association with a serum magnesium reading of 1.1 mEq/L? a. Potassium 5.7 mEq/L b. Calcium 7.8 mg/dL c. Sodium 149 mEq/L d. Phosphorus 2.6 mg/dL

B. Calcium 7.8 mg/dL Hypocalcemia often occurs with hypomagnesemia. A calcium level of 7.8 mg/dL is low. A sodium level of 149 mEq/L is slightly elevated, but not related to the low magnesium level. A phosphorus level of 2.6 mg/dL is slightly low, but not related to hypomagnesemia.

ANS: B Hydroxyurea (Droxia) has been used successfully to reduce sickling of cells and pain episodes associated with sickle cell disease (SCD). Clients with SCD are not prescribed anticoagulants such as heparin or warfarin (Coumadin). t-PA is used as a "clot buster" in clients who have had ischemic strokes.

A 32-year-old client is recovering from a sickle cell crisis. His discomfort is controlled with pain medications and he is to be discharged. What medication does the nurse expect to be prescribed for him before his discharge? a. Heparin (Heparin) b. Hydroxyurea (Droxia) c. Tissue plasminogen activator (t-PA) d. Warfarin (Coumadin)

ANS: A The medication in a metered-dose inhaler is sprayed into the spacer. The child can then inhale the medication without having to coordinate the spraying and breathing.

A 4-year-old child needing to use a metered-dose inhaler to treat asthma cannot coordinate her breathing to use it effectively. The appropriate intervention by nurse is to use which piece of respiratory equipment? a. A spacer b. A nebulizer c. A peak expiratory flow meter d. Chest physiotherapy

ANS: A The drug reduces local immunity and increases the risk for local infection, especially Candida albicans. Rinsing the mouth after using the inhaler will decrease the risk for developing this infection. Use of mouthwash and broad-spectrum antibiotics is not warranted in this situation. The nurse should document the finding, but the best action to take is to have the client start rinsing his or her mouth after using fluticasone. An oral specimen for culture and sensitivity will not provide information necessary to care for this client.

A nurse assesses a client who is prescribed fluticasone (Flovent) and notes oral lesions. Which action should the nurse take? a. Encourage oral rinsing after fluticasone administration. b. Obtain an oral specimen for culture and sensitivity. c. Start the client on a broad-spectrum antibiotic. d. Document the finding as a known side effect.

A nurse is performing a physical assessment of a patient who is experiencing fluid volume excess. Upon examination of the patient's legs, the nurse documents: "Pitting edema; 6 mm pit; pit remains several seconds after pressing with obvious skin swelling." What grade of edema has this nurse documented? a. 1+ pitting edema b. 2+ pitting edema c. 3+ pitting edema d. 4+ pitting edema

C. 3+ pitting edema 3+ pitting edema is represented by a deep pit (6 mm) that remains seconds after pressing with skin swelling obvious by general inspection. 1+ is a slight indentation (2 mm) with normal contours associated with interstitial fluid volume 30% above normal. 2+ is a 4-mm pit that lasts longer than 1+ with fairly normal contour. +4 is a deep pit (8 mm) that remains for a prolonged time after pressing with frank swelling.

The nurse is reviewing lab values for a client recently admitted to the medical-surgical unit. Which lab result is severely abnormal? a. Potassium, 3.5 mEq/L b. Sodium, 137 mEq/L c. Chloride, 107 mEq/L d. Magnesium, 6.2 mEq/L

D. Magnesium, 6.2 mEq/L A magnesium level of 6.2 mEq/L is greatly elevated. Clients with severe hypermagnesemia are in grave danger of cardiac arrest. The normal magnesium level is 1.3-2.1 mEq/L. The sodium and potassium results are within normal limits. The chloride level is just slightly elevated, with the normal range being between 98-106 mEq/L.

ANS: C Prophylactic therapy with twice-daily oral penicillin reduces the incidence of pneumonia and other streptococcal infections and is the correct drug to use. It is a standard protocol for long-term prophylactic use in clients with sickle cell disease. Cefaclor (Ceclor) and vancomycin (Vancocin) are antibiotics more specific for short-term use and would be inappropriate for this client. Gentamicin (Garamycin) is a drug that can cause liver and kidney damage with long-term use.

A 32-year-old client recovering from a sickle cell crisis is to be discharged. The nurse says, "You and all clients with sickle cell disease are at risk for infection because of your decreased spleen function. For this reason, you will most likely be prescribed an antibiotic before discharge." Which drug does the nurse anticipate the health care provider will request? a. Cefaclor (Ceclor) b. Gentamicin (Garamycin) c. Penicillin V (Pen-V K) d. Vancomycin (Vancocin)

ANS: B Hypoxia and deoxygenation of the RBCs are the most common cause of sickling, so administration of oxygen is the priority intervention here. Pain control and hydration are also important interventions for this patient and should be accomplished rapidly. Vaccination may help prevent future sickling episodes by decreasing the risk of infection, but it will not help with the current sickling crisis. Focus: Prioritization

A 32-year-old patient with sickle cell anemia is admitted to the hospital during a sickle cell crisis. Which action prescribed by the health care provider will you implement first? a. Give morphine sulfate 4 to 8 mg IV every hour as needed. b. Administer 100% oxygen using a nonrebreather mask. c. Start a 14-gauge IV line and infuse normal saline at 200 mL/hr. d. Give pneumococcal (Pneumovax) and Haemophilus influenzae (ActHIB) vaccines.

ANS: A All responses indicate conditions that are beneficial to the child. Respiratory distress and hypoxia cause anxiety as this vital life function is threatened. When anxiety improves, the nurse knows that the respiratory status must be improving as well even if signs and symptoms continue.

A 4-year-old child with croup is brought to the emergency department. The child is anxious and crying and has a high-pitched stridor, retractions, and a barky cough. After administration of cool mist therapy, which assessment finding would indicate significant improvement in the child's respiratory status? a. The child is less anxious. b. The respiratory rate is decreased. c. Wheezing is less loud. d. The child drinks 8 ounces of fluid.

ANS: A Aspirin causes an increase in carbon dioxide; the body compensates for this by increasing ventilations to blow off excess CO2. Hypoventilation would cause the body to retain even more carbon dioxide and therefore respiratory acidosis. Flail chest occurs with trauma to the chest wall. Shallow respirations would increase serum pH.

A 5-year-old who has strep throat was given aspirin for fever. The nurse knows to expect which change in the child's respiratory pattern? a. Hyperventilation to decrease serum levels of carbon dioxide b. Hypoventilation to compensate for metabolic alkalosis c. Flail chest to decrease the work of breathing d. Shallow respirations to decrease serum pH

ANS: C Because pernicious anemia prevents the absorption of vitamin B12, this patient requires injections or intranasal administration of cobalamin. Alcohol use does not cause cobalamin deficiency. Proton pump inhibitors decrease the absorption of vitamin B12. Eating more foods rich in vitamin B12 is not helpful because the lack of intrinsic factor prevents absorption of the vitamin.

A 52-year-old patient has a new diagnosis of pernicious anemia. The nurse determines that the patient understands the teaching about the disorder when the patient states, "I a. need to start eating more red meat and liver." b. will stop having a glass of wine with dinner." c. could choose nasal spray rather than injections of vitamin B12." d. will need to take a proton pump inhibitor like omeprazole (Prilosec)."

ANS: A When the body cannot meet the increased oxygenation need, the increased metabolic rate causes breakdown of protein and wasting of respiratory muscles, increasing the work of breathing.

A 6-year-old boy is admitted to the pediatric unit with chills and a fever of 104°F (40°C). What physiological process explains why the child is at risk for developing dyspnea? a. Fever increases metabolic demands, requiring increased oxygen need. b. Blood glucose stores are depleted, and the cells do not have energy to use oxygen. c. Carbon dioxide production increases as result of hyperventilation. d. Carbon dioxide production decreases as a result of hypoventilation.

ANS: D Sickle cell disease is an autosomal recessive disorder; therefore both parents must have the trait for the child to have the disease.

A child is diagnosed with sickle cell disease. The parents are unsure of how their child contracted the disease. What is the most appropriate explanation by the nurse? a. The mother has the trait, but the father does not. b. The father has the trait, but the mother does not. c. The mother has the disease, but the father has neither the trait nor the disease. d. The mother and father have the trait; therefore the child has a 25% chance for having the disease.

ANS: B-C-E Children should have a diet high in calcium or be placed on calcium supplements to reduce the risk of osteopenia. Live plants and fresh vegetables should be avoided because they carry bacteria. Practicing good hand hygiene is essential to prevent the spread of infection .Children cannot return to school for 6-12 months after HSCT. Either in-hospital or home schooling is required. Children and their families should be encouraged to get yearly influenza vaccination.

A child is status post hematopoietic stem cell transplantation (HSCT) and is preparing for discharge home. Based on the nurse's knowledge of HSCT, which concepts are important to include in the discharge teaching plan of care? (Select all that apply.) a. Preparing the child to return to school within six weeks b. Keeping the child on a high-calcium diet c. Avoiding live plants and fresh vegetables d. Avoiding influenza vaccinations e. Practicing good hygiene

ANS: D The forced expiratory volume measures the maximum amount of air that can be forcefully exhaled in the first second. This can provide an objective measure of pulmonary function compared with the child's baseline.

A child with asthma is having pulmonary function tests. Which phrase explains the purpose of the forced expiratory volume (FEV1)? a. It confirms the diagnosis of asthma. b. It determines the cause of asthma. c. It identifies the "triggers" of asthma. d. It assesses the severity of asthma.

ANS: A Polycythemia is a compensatory response to chronic hypoxia. The body attempts to improve tissue oxygenation by producing additional red blood cells and thereby increases the oxygen-carrying capacity of the blood. Infection is not a clinical consequence of cyanosis. Although dehydration can occur in cyanotic heart disease, it is not a compensatory mechanism for chronic hypoxia. It is not a clinical consequence of cyanosis. Anemia may develop as a result of increased blood viscosity. Anemia is not a clinical consequence of cyanosis.

A child with pulmonary atresia exhibits cyanosis with feeding. On reviewing this child's laboratory values, the nurse is not surprised to notice which abnormality? a. Polycythemia b. Infection c. Dehydration d. Anemia

Which nursing intervention takes priority for a client admitted with severe metabolic acidosis? a. Perform medication reconciliation b. Assess the client's strength in the extremities c. Obtain a diet history for the past 3 days d. Initiate cardiac monitoring

ANS: D The nurse follows the ABCs and initiates cardiac monitoring to observe for signs of hyperkalemia or cardiac arrest. Medication reconciliation should be performed as soon as possible; however, this client is at risk for cardiac and neurologic complications of acidosis. Starvation may precipitate ketosis/acidosis, but this is not the priority.

ANS: C Severe chest pain, fever, a cough, and dyspnea are the signs and symptoms of chest syndrome. The nurse must notify the practitioner immediately. Breathing 100% oxygen to relieve hypoxia may be ordered by the practitioner, but the first action is notification because these symptoms indicate a medical emergency. Pain medications may be indicated, but evaluation is necessary first. Severe chest pain, fever, cough, and dyspnea are not signs of a stroke.

A child with sickle cell anemia develops severe chest pain, fever, a cough, and dyspnea. The nurse's first action is to a. Administer 100% oxygen to relieve hypoxia. b. Administer pain medication to relieve symptoms. c. Notify practitioner because chest syndrome is suspected. d. Notify practitioner because child may be having a stroke.

ANS: D As a result of repeated blood transfusions, excessive deposits of iron (hemosiderosis) are stored in tissues.

A child with thalassemia major receives blood transfusions frequently. The nurse is aware that a complication of repeated blood transfusions is: a. hemarthrosis. b. hematuria. c. hemoptysis. d. hemosiderosis.

ANS: D A side effect of hypertransfusion therapy is often iron overload. Deferoxamine is an iron-chelating drug that binds excess iron; therefore, it can be excreted by the kidneys. Deferoxamine does not prevent blood transfusions. Deferoxamine does not stimulate red cell production. Deferoxamine is not a vitamin supplement.

A child with β-thalassemia is receiving numerous blood transfusions. In addition, the child is receiving deferoxamine (Desferal) therapy. The child's parents ask the nurse what deferoxamine does. The most appropriate response by the nurse is a. "The medication helps to prevent blood transfusion reactions." b. "The medication stimulates red blood cell production." c. "The medication provides vitamin supplementation." d. "The medication helps to prevent iron overload."

ANS: D The best indicator of fluid volume gain or loss is daily weight; because each kilogram represents approximately 1 liter, this client has lost approximately 2500 mL of fluid. Diuresis of 400 mL in 24 hours represents oliguria. Although a blood pressure of 122/84 mm Hg is a normal finding, alone it is not significant for relief of hypervolemia. Although an apical pulse of 82 beats/min is a normal finding, alone it is not significant to determine whether hypervolemia is relieved.

A client admitted for heart failure has a priority problem of hypervolemia related to compromised regulatory mechanisms. Which assessment result obtained the day after admission is the best indicator that the treatment has been effective? a. The client has diuresis of 400 mL in 24 hours b. The client's blood pressure is 122/84 mm Hg c. The client has an aplical pulse of 82 beats/min d. The client's weight decreases by 2.5 kg

ANS: B The best response is for the nurse to offer self, a therapeutic communication technique that uses presence. Attempting to assign blame to both parents will not help the client feel better. There is genetic testing available, so it is inaccurate to state there is no way to know who will have the disease. Stating that good treatments exist belittles the client's feelings.

A client admitted for sickle cell crisis is distraught after learning her child also has the disease. What response by the nurse is best? a. "Both you and the father are equally responsible for passing it on." b. "I can see you are upset. I can stay here with you a while if you like." c. "It's not your fault; there is no way to know who will have this disease." d. "There are many good treatments for sickle cell disease these days."

ANS: D Pulse oximetry is not always the most accurate assessment tool for hypoxia as many factors can interfere, producing normal or near-normal readings in the setting of hypoxia. The nurse should conduct a more thorough assessment. The other actions are not appropriate for a hypoxic client.

A client appears dyspneic, but the oxygen saturation is 97%. What action by the nurse is best? a. Obtain a new oximeter from central supply. b. Change the sensor on the pulse oximeter. c. Tell the client to take slow, deep breaths. d. Assess for other manifestations of hypoxia.

ANS: A Daily weights are needed to document fluid retention or fluid loss. One liter of fluid equals 2.2 pounds.

A client asks the nurse why it is important to be weighed every day if he has right-sided heart failure. What is the nurse's best response? a. "Weight is the best indication that you are gaining or losing fluid." b. "Daily weights will help us make sure that you're eating properly." c. "The hospital requires that all inpatients be weighed daily." d. "You need to lose weight to decrease the incidence of heart failure."

ANS: B Angiotensin-converting enzyme (ACE) inhibitors are associated with first-dose hypotension and orthostatic hypotension, which are more likely in those older than 75 years. Although desirable, understanding of teaching is not essential. ACE inhibitors are vasodilators; they do not affect heart rate. Renal function, not liver function, may be altered by ACE inhibitors.

A client begins therapy with lisinopril (Prinivil, Zestril). What does the nurse consider at the start of therapy with this medication? a. The client's ability to understand medication teaching b. The risk for hypotension c. The potential for bradycardia d. Liver function tests

ANS: A Malabsorption syndromes such as Crohn's disease leave a client prone to folic acid deficiency. Fanconi's anemia, hemolytic anemia, and vitamin B12 anemia are not related to Crohn's disease.

A client has Crohn's disease. What type of anemia is this client most at risk for developing? a. Folic acid deficiency b. Fanconi's anemia c. Hemolytic anemia d. Vitamin B12 anemia

ANS: C A large blood clot in the lungs will significantly impair gas exchange and oxygenation. Unless the clot is dissolved, this process will continue unabated. Hyperventilation can interfere with oxygenation by shallow breathing, but there is no evidence that the client is hyperventilating, and this is also not the most precise physiologic answer. Respiratory distress syndrome can occur, but this is not as likely. The client may need to be mechanically ventilated, but without concrete data on FiO2 and SaO2, the nurse cannot make that judgment.

A client has a pulmonary embolism and is started on oxygen. The student nurse asks why the client's oxygen saturation has not significantly improved. What response by the nurse is best? a. "Maybe the client has respiratory distress syndrome." b. "Breathing so rapidly interferes with oxygenation." c. "The blood clot interferes with perfusion in the lungs." d. "The client needs immediate intubation and mechanical ventilation."

ANS: B Because of skin pigmentation, translucent areas of high blood flow such as mucous membranes are best to check for cyanosis, which is a sign of hypoxia. It is important to remember that cyanosis is a late sign of hypoxia.

The nurse is caring for an African American patient with COPD. The nurse knows that the best location to assess for hypoxia is the a. Nailbeds. b. Oral mucosa. c. Earlobe. d. Lower extremities.

ANS: B This client has laboratory findings indicative of iron deficiency anemia. The most common cause of this disorder is blood loss, often from the GI tract. The nurse should perform a Hemoccult test on the client's stools. High-protein foods may help the condition, but dietary interventions take time to work. That still does not determine the cause. Frequent oral care is not related. Cobalamin injections are for pernicious anemia.

A client has a serum ferritin level of 8 ng/mL and microcytic red blood cells. What action by the nurse is best? a. Encourage high-protein foods. b. Perform a Hemoccult test on the client's stools. c. Offer frequent oral care. d. Prepare to administer cobalamin (vitamin B12).

ANS: C During a sickle cell crisis, the tissue distal to the occlusion has decreased blood flow and ischemia, leading to pain. Due to decreased blood flow, the client's legs will be cool or cold. The UAP can attempt to keep the client's legs warm. Ice and elevation will further decrease perfusion. Elastic bandage wraps are not indicated and may constrict perfusion in the legs.

A client has a sickle cell crisis with extreme lower extremity pain. What comfort measure does the nurse delegate to the unlicensed assistive personnel (UAP)? a. Apply ice packs to the client's legs. b. Elevate the client's legs on pillows. c. Keep the lower extremities warm. d. Place elastic bandage wraps on the client's legs.

ANS: A Assessment of respiratory and oxygenation status is the priority nursing intervention for the prevention of complications. Monitoring electrolytes and inserting a catheter are important but do not take priority over assessing respiratory status. The client needs IV access, but fluids may need to be administered judiciously.

A client has been admitted to the acute care unit for an exacerbation of heart failure. Which is the nurse's priority intervention? a. Assess respiratory status. b. Monitor electrolyte levels. c. Administer intravenous fluids. d. Insert a Foley catheter.

ANS: D The client with worsening heart failure is most at risk for pulmonary edema as a consequence of fluid retention. Administering diuretics will decrease the fluid overload, thereby decreasing the incidence of pulmonary edema. High Fowler's position might help the client breathe easier but will not solve the problem. CPR is not warranted in this situation. Rest is important for clients with heart failure, but this is not the priority.

A client has been admitted to the intensive care unit with worsening pulmonary manifestations of heart failure. What is the nurse's best action? a. Place the client in a high Fowler's position. b. Begin cardiopulmonary resuscitation (CPR). c. Promote rest and minimize activities. d. Administer loop diuretics as prescribed.

ANS: A The nurse's priority is to care for the client. Since the client has gunshot wounds and is bleeding, the nurse applies personal protective equipment (i.e., gloves) prior to care. This takes priority over calling law enforcement. Requesting blood bank products can be delegated. The nurse may or may not have to prepare the client for emergency surgery.

A client has been brought to the emergency department after being shot multiple times. What action should the nurse perform first? a. Apply personal protective equipment. b. Notify local law enforcement officials. c. Obtain "universal" donor blood. d. Prepare the client for emergency surgery.

ANS: B The priority for any chest trauma client is airway, breathing, circulation. The nurse first ensures the client has a patent airway. Assessing respiratory rate and applying oxygen are next, followed by inserting IVs.

A client has been brought to the emergency department with a life-threatening chest injury. What action by the nurse takes priority? a. Assess the respiratory rate. b. Ensure a patent airway. c. Apply oxygen at 100%. d. Start two large-bore IV lines.

ANS: D Clients who are hyperkalemic (those with an elevated serum potassium level) may also be in renal failure. The client's serum creatinine should be reviewed to determine if it is greater than 1.8 mg/dL, at which time the health care provider should be notified before administering any supplemental potassium.

A client has been taking furosemide (Lasix) and valsartan (Diovan) for the past year. The hospital laboratory notifies the nurse that the client's serum potassium level is 6.2 mEq/L. What is the nurse's best action at this time? a. Assess the client's oxygen saturation level b. Ask the laboratory to retest the potassium level c. Give potassium as an IV infusion d. Check the client's serum creatinine

ANS: A Clients with sickle cell crisis often have severe pain that is managed with up to 48 hours of IV opioid analgesics. Even if the client is addicted and drug seeking, he or she is still in extreme pain. If the client can receive another dose of medication, the nurse should provide it. The other options are judgmental and do not address the client's pain. Giving placebos is unethical.

A client hospitalized with sickle cell crisis frequently asks for opioid pain medications, often shortly after receiving a dose. The nurses on the unit believe the client is drug seeking. When the client requests pain medication, what action by the nurse is best? a. Give the client pain medication if it is time for another dose. b. Instruct the client not to request pain medication too early. c. Request the provider leave a prescription for a placebo. d. Tell the client it is too early to have more pain medication.

ANS: A Natrecor should be given through a separate IV access because it is incompatible with many medications, especially heparin. A test bolus is not needed, nor is Lasix. Because the medication should be given through a separate IV, it is not necessary to prepare a piggyback line.

A client in severe heart failure has a heparin drip infusing. The health care provider prescribes nesiritide (Natrecor) to be given intravenously. Which intervention is essential before administration of this medication? a. Insert a separate IV access. b. Prepare a test bolus dose. c. Prepare the piggyback line. d. Administer furosemide (Lasix) first.

ANS: D Airway always is the priority. The other actions are important in this situation as well, but the nurse should stay with the client and ensure the airway remains patent (especially if vomiting occurs) while another person calls the provider (or Rapid Response Team) and facilitates getting an ECG done. Aspirin will probably be administered, depending on the provider's prescription and the client's current medications.

A client in the cardiac stepdown unit reports severe, crushing chest pain accompanied by nausea and vomiting. What action by the nurse takes priority? a. Notify the provider b. Call for an electrocardiogram (ECG) c. Administer an aspirin d. Maintain airway patency

ANS: A Broad-spectrum antibiotics must be initiated within 1 hour of establishing diagnosis. A blood transfusion is indicated for low red blood cell count or low hemoglobin and hematocrit; transfusion is not part of the sepsis resuscitation bundle. Cooling baths are not indicated because the client is hypothermic, nor is this part of the sepsis resuscitation bundle. NPO status is not indicated for this client, nor is it part of the sepsis resuscitation bundle.

A client is admitted to the hospital with two of the systemic inflammatory response syndrome variables: temperature of 95° F (35° C) and high white blood cell count. Which intervention from the sepsis resuscitation bundle does the nurse initiate? a. Broad-spectrum antibiotics b. Blood transfusion c. Cooling baths d. NPO status

ANS: C In heart failure, stimulation of the sympathetic nervous system represents the most immediate response. Adrenergic receptor stimulation causes an increase in heart rate and respiratory rate. Blood pressure will remain the same or will elevate slightly. Changes in creatinine occur when kidney damage has occurred, which is a later manifestation. Other later manifestations may include edema, increased respiratory rate, and lowered oxygen saturation readings.

A client is admitted with early-stage heart failure. Which assessment finding does the nurse expect? a. A decrease in blood pressure and urine output b. An increase in creatinine and extremity edema c. An increase in heart rate and respiratory rate d. A decrease in respirations and oxygen saturation

ANS: B, C, E Clients with left-sided heart failure will exhibit symptoms such as fatigue, dyspnea or breathlessness, and crackles on auscultation of breath sounds. Peripheral edema and ascites are associated with right-sided heart failure.

A client is diagnosed with left-sided heart failure. Which assessment findings will the nurse expect the client to have? Select all that apply. a. Peripheral edema b. Crackles in both lungs c. Breathlessness d. Ascites e. Tachypnea

ANS: A, D, E Having a familiar person nearby may provide comfort to the client. The nurse should remain with the client who is demonstrating physiologic deterioration. Offering genuine reassurance supports the client who is anxious. The health care provider should be notified, and increasing IV and oxygen rates may be needed, but these actions do not support the client's psychosocial integrity.

A client is exhibiting signs and symptoms of early shock. What is important for the nurse to do to support the psychosocial integrity of the client? (Select all that apply.) a. Ask the family members to stay with the client b. Call the health care provider c. Increase IV and oxygen rates d. Remain with client e. Reassure the client that everything is being done for him or her

ANS: A High glucose readings are common in shock, and best outcomes are the result of treating them and maintaining glucose readings in the normal range. Medications and IV solutions may raise blood glucose levels, but this is not the most accurate answer. The stress of the illness has not "made" the client diabetic.

A client is in shock and the nurse prepares to administer insulin for a blood glucose reading of 208 mg/dL. The spouse asks why the client needs insulin as the client is not a diabetic. What response by the nurse is best? a. "High glucose is common in shock and needs to be treated." b. "Some of the medications we are giving are to raise blood sugar." c. "The IV solution has lots of glucose, which raises blood sugar." d. "The stress of this illness has made your spouse a diabetic."

ANS: B This client's physiologic parameters did not exceed normal during and after activity, so it is safe for the client to continue using the bathroom. There is no indication that the client needs oxygen, a commode, or a bedpan.

A client is in the hospital after suffering a myocardial infarction and has bathroom privileges. The nurse assists the client to the bathroom and notes the client's O2 saturation to be 95%, pulse 88 beats/min, and respiratory rate 16 breaths/min after returning to bed. What action by the nurse is best? a. Administer oxygen at 2 L/min b. Allow continued bathroom privileges c. Obtain a bedside commode d. Suggest the client use a bedpan

ANS: A Normal cognitive function is a good indicator that the client is receiving the benefits of norepinephrine. The brain is very sensitive to changes in oxygenation and perfusion. Norepinephrine can cause chest pain as an adverse reaction, so the absence of chest pain does not indicate therapeutic effect. The IV site is normal. The urine output is normal, but only minimally so.

A client is receiving norepinephrine (Levophed) for shock. What assessment finding best indicates a therapeutic effect from this drug? a. Alert and oriented, answering questions b. Client denial of chest pain or chest pressure c. IV site without redness or swelling d. Urine output of 30 mL/hr for 2 hours

ANS: A The client at risk for septic shock should be instructed to clean his or her toothbrush daily, either by running it through the dishwasher or by rinsing it in laundry bleach. Clients should be instructed to bathe daily and wash the armpits, the groin, and the rectal area. The client should refrain from cleaning pet litter boxes. Clients recovering from septic shock are not at higher risk for bleeding disorders.

A client recovering from septic shock is preparing for discharge home. What priority information does the nurse include in the teaching plan for this client? a. "Clean your toothbrush with laundry bleach daily." b. "Bathe every other day with antimicrobial soap." c. "Wash your hands after changing pet litter boxes." d. "Use an electric razor when you shave your face."

ANS: C The hypodynamic phase of septic shock is characterized by a rapid decrease in cardiac output, systolic blood pressure, and pulse pressure. The nurse must initiate drug therapy to maintain blood pressure and cardiac output. Accurate urinary output and blood cultures are important to the treatment but are not the priority when a client's pulse pressure is decreasing rapidly. The family should be updated appropriately.

A client was admitted 2 days ago with early stages of septic shock. Today the nurse notes that the client's systolic blood pressure, pulse pressure, and cardiac output are decreasing rapidly. Which intervention does the nurse do first? a. Insert a Foley catheter to monitor urine output closely. b. Ask the client's family to come to the hospital because death is near. c. Initiate the prescribed dobutamine (Dobutrex) intravenous drip. d. Obtain blood cultures before administering the next dose of antibiotics.

ANS: C IV therapy for fluid resuscitation is the primary intervention for hypovolemic shock. A dopamine hydrochloride drip is a secondary treatment if the client does not respond to fluids. Aminoglycosides and heparin are given to clients with septic shock

The nurse is caring for a client who has hypovolemic shock. After administering oxygen, what is the priority intervention for this client? a. Administer an aminoglycoside. b. Initiate a dopamine hydrochloride (Intropin) drip. c. Administer crystalloid fluids. d. Initiate an intravenous heparin drip.

ANS: A When a local infection becomes systemic, the client develops a high-grade temperature, decreased urine output, and increased respiratory rate. Because of tachycardia and low blood pressure, the client may exhibit orthostatic hypotension. This is a subtle sign of systemic infection that requires further evaluation by the health care provider. The other signs are not manifestations of complications. Warmth and redness are expected with local infection.

A client who has a local infection of the right forearm is being discharged. The nurse teaches the client to seek immediate medical attention if which complication occurs? a. Dizziness on changing position b. Increased urine output c. Warmth and redness at site d. Low-grade temperature

ANS: B This client is at risk for depression because of the diagnosis of heart failure, and further assessment should be done. Calling the family to help distract the client does not address the core issue. Sedation is inappropriate in this situation because it ignores the client's feelings. Telling the client that things will get better may give the client false hope, and ignores the client's feelings.

A client who has been admitted for the third time this year for heart failure says, "This isn't worth it anymore. I just want it all to end." What is the nurse's best response? a. Calls the family to lift the client's spirits b. Considers further assessment for depression c. Sedates the client to decrease myocardial oxygen demand d. Tells the client that things will get better

ANS: A Blood cultures should be obtained before IV antibiotics are started. If hypotension occurs, fluid resuscitation is used first. CVP monitoring and vasopressor therapy are started if hypotension persists.

A client who has septic shock is admitted to the hospital. What priority intervention does the nurse implement first? a. Obtain two sets of blood cultures. b. Administer the prescribed IV vancomycin (Vancocin). c. Obtain central venous pressure (CVP) measurements. d. Administer the prescribed IV norepinephrine (Levophed).

ANS: D Clients with heart failure are instructed to weigh themselves daily to detect worsening heart failure early, and thus avoid complications. Other signs of worsening heart failure include increasing dyspnea, exercise intolerance, cold symptoms, and nocturia.

A client with a history of heart failure is being discharged. Which priority instruction will assist the client in the prevention of complications associated with heart failure? a. "Avoid drinking more than 3 quarts of liquids each day." b. "Eat six small meals daily instead of three larger meals." c. "When you feel short of breath, take an additional diuretic." d. "Weigh yourself daily while wearing the same amount of clothing."

ANS: A The client with a history of myocardial infarction is at risk for developing heart failure. The onset of nocturnal cough is an early manifestation of heart failure, and the client needs to be evaluated as soon as possible.

A client with a history of myocardial infarction calls the clinic to report the onset of a cough that is troublesome only at night. What direction does the nurse give to the client? a. "Please come into the clinic for an evaluation." b. "Increase your fluid intake during waking hours." c. "Use an over-the-counter cough suppressant." d. "Sleep on two pillows to facilitate postnasal drainage."

ANS: A The nurse should administer the medication. Generally, the health care provider will maintain the client's blood pressure between 90 and 110 mm Hg.

A client with heart failure is due to receive enalapril (Vasotec) and has a blood pressure of 98/50 mm Hg. What is the nurse's best action? a. Administer the Vasotec. b. Recheck the blood pressure. c. Hold the Vasotec. d. Notify the health care provider.

ANS: A Placing a client in a high Fowler's position, especially with pillows under each arm, can maximize chest expansion and improve oxygenation. The nurse next should auscultate the client's heart and lungs. The client may or may not need fluid restriction to help manage heart failure, and suctioning is not needed.

A client with heart failure is experiencing acute shortness of breath. What is the nurse's priority action? a. Place the client in a high Fowler's position. b. Perform nasotracheal suctioning of the client. c. Auscultate the client's heart and lung sounds. d. Place the client on a 1000 mL fluid restriction.

ANS: A Angiotensin-converting enzyme (ACE) inhibitors inhibit the excretion of potassium. Hyperkalemia can be a life-threatening side effect, and clients should be taught to limit potassium intake. Salt substitutes are composed of potassium chloride.

A client with heart failure is prescribed enalapril (Vasotec). What is the nurse's priority teaching for this client? a. "Avoid using salt substitutes." b. "Take your medication with food." c. "Avoid using aspirin-containing products." d. "Check your pulse daily."

ANS: D A diuretic such as bumetanide will decrease blood volume in a client who is already hypovolemic; this order should be questioned because this is not an appropriate action to expand the client's blood volume. The other orders are appropriate for improving blood pressure in shock, and do not need to be questioned.

A client with hypovolemic shock has these vital signs: temperature 97.9° F; pulse 122 beats/min; blood pressure 86/48 mm Hg; respirations 24 breaths/min; urine output 20 mL for last 2 hours; skin cool and clammy. Which medication order for this client does the nurse question? a. Dopamine (Intropin) 12 mcg/kg/min b. Dobutamine (Dobutrex) 5 mcg/kg/min c. Plasmanate 1 unit d. Bumetanide (Bumex) 1 mg IV

ANS: B Dopamine improves blood flow by increasing peripheral resistance, which increases blood pressure—a positive response in this case. Urine output less than 30 mL/hr or 0.5 mL/kg/hr and elevations in serum creatinine indicate poor tissue perfusion to the kidney and are a negative consequence of shock, not a positive response. Although a blood glucose of 245 mg/dL is an abnormal finding, dopamine increases blood pressure and myocardial contractility, not glucose levels.

A client with septic shock has been started on dopamine (Intropin) at 12 mcg/kg/min. Which response indicates a positive outcome? a. Hourly urine output 10 to 12 mL/hr b. Blood pressure 90/60 mm Hg and mean arterial pressure 70 mm Hg c. Blood glucose 245 mg/dL d. Serum creatinine 3.6 mg/dL

ANS: B Clients with polycythemia vera often have clotting abnormalities due to the hyperviscous blood with sluggish flow. The client reporting shortness of breath may have a pulmonary embolism and should be seen first. The client with a swollen calf may have a deep vein thrombosis and should be seen next. High blood pressure and gout symptoms are common findings with this disorder.

A nurse in a hematology clinic is working with four clients who have polycythemia vera. Which client should the nurse see first? a. Client with a blood pressure of 180/98 mm Hg b. Client who reports shortness of breath c. Client who reports calf tenderness and swelling d. Client with a swollen and painful left great toe

ANS: D Although individuals with SCD often have elevated white blood cell (WBC) counts, this extreme elevation could indicate leukemia, a complication of taking hydroxyurea. The nurse should report this finding immediately. Alternatively, it could indicate infection, a serious problem for clients with SCD. Hematocrit and hemoglobin levels are normally low in people with SCD. The potassium level, while slightly low, is not as worrisome as the WBCs.

A client with sickle cell disease (SCD) takes hydroxyurea (Droxia). The client presents to the clinic reporting an increase in fatigue. What laboratory result should the nurse report immediately? a. Hematocrit: 25% b. Hemoglobin: 9.2 mg/dL c. Potassium: 3.2 mEq/L d. White blood cell count: 38,000/mm3

ANS: A Thrombosis is the most likely complication. The patient with polycythemia may experience angina, heart failure, intermittent claudication, and thrombophlebitis, which may be complicated by embolization. The most common and serious acute complication is stroke due to thrombosis.

A complication of the hyperviscosity of polycythemia is a. Thrombosis. b. Cardiomyopathy. c. Pulmonary edema. d. Disseminated intravascular coagulation (DIC).

ANS: D Monitoring hydration status is important during an acute exacerbation because the patient is at risk for fluid overload or underhydration. Aspirin therapy is used to decrease risk for thrombosis. The patient should be encouraged to ambulate to prevent deep vein thrombosis (DVT). Iron is contraindicated in patients with polycythemia vera.

A critical action by the nurse caring for a patient with an acute exacerbation of polycythemia vera is to a. place the patient on bed rest. b. administer iron supplements. c. avoid use of aspirin products. d. monitor fluid intake and output.

ANS: B Cholinergic antagonist drugs cause bronchodilation by inhibiting the parasympathetic nervous system. This allows the sympathetic nervous system to dominate and release norepinephrine that actives beta2 receptors. Bronchodilators relax bronchiolar smooth muscles by binding to and activating pulmonary beta2 receptors. Corticosteroids disrupt the production of pathways of inflammatory mediators. Cromones stabilize the membranes of mast cells and prevent the release of inflammatory mediators.

A nurse administers medications to a client who has asthma. Which medication classification is paired correctly with its physiologic response to the medication? a. Bronchodilator - Stabilizes the membranes of mast cells and prevents the release of inflammatory mediators b. Cholinergic antagonist - Causes bronchodilation by inhibiting the parasympathetic nervous system c. Corticosteroid - Relaxes bronchiolar smooth muscles by binding to and activating pulmonary beta2 receptors d. Cromone - Disrupts the production of pathways of inflammatory mediators

ANS: C, E Suprasternal retraction caused by inhalation usually indicates that the client is using accessory muscles and is having difficulty moving air into the respiratory passages because of airway narrowing. Wheezing indicates a narrowed airway; a decreased pulse oxygen saturation also supports this finding. The asthma is not responding to the medication, and intervention is needed. Administration of a rescue inhaler is indicated, probably along with administration of oxygen. The nurse would not do a peak flow reading at this time, nor would a code be called. Midline trachea is a normal and expected finding.

A nurse assesses a client with asthma and notes bilateral wheezing, decreased pulse oxygen saturation, and suprasternal retraction on inhalation. Which actions should the nurse take? (Select all that apply.) a. Administer prescribed salmeterol (Serevent) inhaler. b. Assess the client for a tracheal deviation. c. Administer oxygen to keep saturations greater than 94%. d. Perform peak expiratory flow readings. e. Administer prescribed albuterol (Proventil) inhaler.

ANS: B, C, E Difficulty sleeping could indicate worsening breathlessness, as could taking longer to perform activities of daily living. Weight loss could mean increased dyspnea as the client becomes too fatigued to eat. The color of the client's sputum would not assist in determining activity tolerance. Asking whether the client walks upstairs every day is not as pertinent as determining if the client becomes short of breath on walking upstairs, or if the client goes upstairs less often than previously.

A nurse assesses a client with chronic obstructive pulmonary disease. Which questions should the nurse ask to determine the client's activity tolerance? (Select all that apply.) a. "What color is your sputum?" b. "Do you have any difficulty sleeping?" c. "How long does it take to perform your morning routine?" d. "Do you walk upstairs every day?" e. "Have you lost any weight lately?"

ANS: D Tachycardia can indicate hypoxemia as the body tries to circulate the oxygen that is available. A barrel chest is not an emergency finding. Likewise, a pulse oximetry level of 92% is not considered an acute finding. The expiratory phase is expected to be longer than the inspiratory phase in someone with airflow limitation.

A nurse assesses several clients who have a history of asthma. Which client should the nurse assess first? a. A 66-year-old client with a barrel chest and clubbed fingernails b. A 48-year-old client with an oxygen saturation level of 92% at rest c. A 35-year-old client who has a longer expiratory phase than inspiratory phase d. A 27-year-old client with a heart rate of 120 beats/min

ANS: C The gene for AAT is a recessive gene. Clients with only one allele produce enough AAT to prevent chronic obstructive pulmonary disease (COPD) unless the client smokes. A client with two alleles is at high risk for COPD even if not exposed to smoke or other irritants. The client is a carrier, and children may or may not be at high risk depending on the partner's AAT levels. Contacting a genetic counselor may be helpful but does not address the client's current question.

A nurse cares for a client who tests positive for alpha1-antitrypsin (AAT) deficiency. The client asks, "What does this mean?" How should the nurse respond? a. "Your children will be at high risk for the development of chronic obstructive pulmonary disease." b. "I will contact a genetic counselor to discuss your condition." c. "Your risk for chronic obstructive pulmonary disease is higher, especially if you smoke." d. "This is a recessive gene and should have no impact on your health."

ANS: D Placing the infant in a knee-chest position will decrease venous return so that smaller amounts of highly saturated blood reach the heart. This should be done after calming the infant. Administering oxygen is indicated after placing the infant in a knee-chest position. Administering morphine sulfate calms the infant. It may be indicated some time after the infant has been calmed. Calming the crying infant is the first response. An infant with unrepaired tetralogy of Fallot who is crying and agitated may eventually lose consciousness.

A nurse is assigned to care for an infant with an unrepaired tetralogy of Fallot. What should the nurse do first when the baby is crying and becomes severely cyanotic? a. Place the infant in a knee-chest position. b. Administer oxygen. c. Administer morphine sulfate. d. Calm the infant.

ANS: A Smoking increases pulmonary hypertension, resulting in cor pulmonale, or right-sided heart failure. Increased pressures in the lungs make it more difficult for blood to flow through the lungs. Blood backs up into the right side of the heart and then into the peripheral venous system, creating distended neck veins and dependent edema. Inflammation in bronchi and bronchioles creates an airway obstruction which manifests as wheezes. Thick mucus in the lungs has no impact on distended neck veins and edema. Left ventricular hypertrophy is associated with left heart failure and is not caused by a 40-year smoking history.

A nurse cares for a client with a 40-year smoking history who is experiencing distended neck veins and dependent edema. Which physiologic process should the nurse correlate with this client's history and clinical manifestations? a. Increased pulmonary pressure creating a higher workload on the right side of the heart b. Exposure to irritants resulting in increased inflammation of the bronchi and bronchioles c. Increased number and size of mucus glands producing large amounts of thick mucus d. Left ventricular hypertrophy creating a decrease in cardiac output

ANS: B Aspirin and other nonsteroidal anti-inflammatory drugs (NSAIDs) can trigger asthma in some people. This results from increased production of leukotriene when aspirin or NSAIDs suppress other inflammatory pathways and is a high priority given the client's history. Reviewing pulmonary function test results will not address the immediate problem of frequent asthma attacks. This is a good intervention for reviewing response to bronchodilators. Questioning the client about the use of bronchodilators will address interventions for the attacks but not their cause. Reviewing arterial blood gas results would not be of use in a client between attacks because many clients are asymptomatic when not having attacks

A nurse cares for a client with arthritis who reports frequent asthma attacks. Which action should the nurse take first? a. Review the client's pulmonary function test results. b. Ask about medications the client is currently taking. c. Assess how frequently the client uses a bronchodilator. d. Consult the provider and request arterial blood gases.

ANS: C Clients with severe COPD may not be able to perform daily activities, including bathing and eating, because of excessive shortness of breath. The nurse should ask the client if shortness of breath is interfering with basic activities. Although the nurse should know about the client's support systems, current knowledge, and medications, these questions do not address the client's appearance.

A nurse cares for a client with chronic obstructive pulmonary disease (COPD) who appears thin and disheveled. Which question should the nurse ask first? a. "Do you have a strong support system?" b. "What do you understand about your disease?" c. "Do you experience shortness of breath with basic activities?" d. "What medications are you prescribed to take each day?"

ANS: C Many clients with moderate to severe COPD become socially isolated because they are embarrassed by frequent coughing and mucus production. They also can experience fatigue, which limits their activities. The nurse needs to encourage the client to verbalize thoughts and feelings so that appropriate interventions can be selected. Joining a support group would not decrease feelings of social isolation if the client does not verbalize feelings. Antianxiety agents will not help the client with social isolation. Encouraging a client to participate in activities without verbalizing concerns also would not be an effective strategy for decreasing social isolation.

A nurse cares for a client with chronic obstructive pulmonary disease (COPD). The client states that he no longer enjoys going out with his friends. How should the nurse respond? a. "There are a variety of support groups for people who have COPD." b. "I will ask your provider to prescribe you with an antianxiety agent." c. "Share any thoughts and feelings that cause you to limit social activities." d. "Friends can be a good support system for clients with chronic disorders."

ANS: C Cystic fibrosis is an autosomal recessive disorder in which both gene alleles must be mutated for the disorder to be expressed. The nurse should encourage both the client and partner to be tested for the abnormal gene. The other statements are not true.

A nurse cares for a female client who has a family history of cystic fibrosis. The client asks, "Will my children have cystic fibrosis?" How should the nurse respond? a. "Since many of your family members are carriers, your children will also be carriers of the gene." b. "Cystic fibrosis is an autosomal recessive disorder. If you are a carrier, your children will have the disorder." c. "Since you have a family history of cystic fibrosis, I would encourage you and your partner to be tested." d. "Cystic fibrosis is caused by a protein that controls the movement of chloride. Adjusting your diet will decrease the spread of this disorder."

ANS: A An elevated creatinine indicates kidney damage, which occurs in SCD. A hematocrit level of 30% is an expected finding, as is a slightly elevated white blood cell count. A sodium level of 147 mEq/L, although slightly high, is not concerning.

A nurse caring for a client with sickle cell disease (SCD) reviews the client's laboratory work. Which finding should the nurse report to the provider? a. Creatinine: 2.9 mg/dL b. Hematocrit: 30% c. Sodium: 147 mEq/L d. White blood cell count: 12,000/mm3

ANS: D Oxygen should be administered to a client who is hypoxic even if the client has COPD and is a carbon dioxide retainer. The other interventions do not address the client's hypoxia, which is the priority.

A nurse evaluates the following arterial blood gas and vital sign results for a client with chronic obstructive pulmonary disease (COPD): Arterial Blood Gas Results Vital Signs pH = 7.32 PaCO2 = 62 mm Hg PaO2 = 46 mm Hg HCO3- = 28 mEq/L Heart rate = 110 beats/min Respiratory rate = 12 breaths/min Blood pressure = 145/65 mm Hg Oxygen saturation = 76% Which action should the nurse take first? a. Administer a short-acting beta2 agonist inhaler. b. Document the findings as normal for a client with COPD. c. Teach the client diaphragmatic breathing techniques. d. Initiate oxygenation therapy to increase saturation to 92%.

ANS: C Up to the age of 6 years, children breathe primarily with their diaphragm. The intercostal muscles assist by increasing the chest diameter. When distress occurs, the intercostal muscles between the rib cage work with extra effort to move air through narrow airways. This causes retractions.

A nurse explains why a 4-year-old presenting with respiratory distress has retractions. Which statement by the parent indicates that the teaching was understood? a. "When distress occurs, children swallow air, leading to expansion of the rib cage and retractions." b. "Retractions occur in all children, because their ribs are soft and pliable. They are not related to respiratory distress." c. "Children breathe primarily with their diaphragm, but when distress occurs, the muscles between the rib cage work with extra effort to move air through narrow airways." d. "Children breathe primarily with the muscles between the ribs, so when distress occurs, the extra work of breathing causes retractions."

ANS: B The patient is experiencing cardiac distress for reasons unknown. The nurse should first secure the safety of the patient and decrease the workload on the patient's heart by putting him in a resting position; this will increase cardiac output by decreasing after load. Once the patient is stable, the nurse can obtain oxygen to put on the patient. Next, the nurse can begin to monitor the patient's oxygen and cardiac status. If necessary, the emergency team may be activated to defibrillate.

A nurse is assisting a patient with ambulation. The patient becomes short of breath and begins to complain of sharp chest pain. Which action by the nurse is the first priority? a. Call for the emergency response team to bring the defibrillator. b. Have the patient sit down in the nearest chair. c. Return the patient to the room and apply 100% oxygen. d. Ask a coworker to get the ECG machine STAT.

ANS: B Urine output changes are a sensitive early indicator of shock. The nurse should delegate emptying the urinary catheter and measuring output to the UAP as a baseline for hourly urine output measurements. The UAP cannot assess for pain. Repositioning may or may not be effective for decreasing restlessness, but does not take priority over physical assessments. Reassurance is a therapeutic nursing action, but the nurse needs to do more in this situation.

A nurse is caring for a client after surgery who is restless and apprehensive. The unlicensed assistive personnel (UAP) reports the vital signs and the nurse sees they are only slightly different from previous readings. What action does the nurse delegate next to the UAP? a. Assess the client for pain or discomfort. b. Measure urine output from the catheter. c. Reposition the client to the unaffected side. d. Stay with the client and reassure him or her.

ANS: B Signs of the earliest stage of shock are subtle and may manifest in slight increases in heart rate, respiratory rate, or blood pressure. Even though these readings are not out of the normal range, the nurse should conduct a thorough assessment of the client, focusing on indicators of perfusion. The client may need pain medication, but this is not the priority at this time. Documentation should be done thoroughly but is not the priority either. The nurse should not increase the rate of the IV infusion without an order.

A nurse is caring for a client after surgery. The client's respiratory rate has increased from 12 to 18 breaths/min and the pulse rate increased from 86 to 98 beats/min since they were last assessed 4 hours ago. What action by the nurse is best? a. Ask if the client needs pain medication. b. Assess the client's tissue perfusion further. c. Document the findings in the client's chart. d. Increase the rate of the client's IV infusion.

ANS: A, B, D The nurse can delegate assisting the client to get up in the chair or ambulate to the bathroom, applying TEDs or sequential compression devices, and taking/recording vital signs. The spirometer should be used every hour the day after surgery. Assessing pain using a 0-to-10 scale is a nursing assessment, although if the client reports pain, the UAP should inform the nurse so a more detailed assessment is done.

A nurse is caring for a client who had coronary artery bypass grafting yesterday. What actions does the nurse delegate to the unlicensed assistive personnel (UAP)? (Select all that apply.) a. Take and record a full set of vitals per hospital protocol. b. Assist the client to the chair for meals and to the bathroom c. Have the client rate pain on a 0-10 scale and report to the nurse d. Ensure the client wears TED hose or sequential compression devices

ANS: B A lactate level of 6 mmol/L is high and is indicative of possible shock. A creatinine level of 0.9 mg/dL is normal. A sodium level of 150 mEq/L is high, but that is not related directly to shock. A white blood cell count of 11,000/mm3 is slightly high but is not as critical as the lactate level.

A nurse is caring for several clients at risk for shock. Which laboratory value requires the nurse to communicate with the health care provider? a. Creatinine: 0.9 mg/dL b. Lactate: 6 mmol/L c. Sodium: 150 mEq/L d. White blood cell count: 11,000/mm3

ANS: C, D, E The parents should be instructed to notify the physician after their infant's cardiac surgery fora temperature above 37.7o C; new, frequent coughing; and any episodes of the infant turning blue or bluer than normal. A respiratory rate of 36 at rest for an infant is within normal expectations, and it is expected that the appetite will increase slowly.

A nurse is conducting discharge teaching to parents about the care of their infant after cardiac surgery. The nurse instructs the parents to notify the physician if what condition occurs? Select all that apply. a. Respiratory rate of 36 at rest b. Appetite slowly increasing c. Temperature above 37.7° C (100° F) d. New, frequent coughing e. Turning blue or bluer than normal

ANS: C Clients with CF often are malnourished due to vitamin deficiency and pancreatic malfunction. Maintaining nutrition is essential. Daily antibiotics and daily exercise are not essential actions. Genetic screening would not help the client manage CF better.

A nurse is teaching a client who has cystic fibrosis (CF). Which statement should the nurse include in this client's teaching? a. "Take an antibiotic each day." b. "Contact your provider to obtain genetic screening." c. "Eat a well-balanced, nutritious diet." d. "Plan to exercise for 30 minutes every day."

ANS: B Weight gain can result from fluid accumulation in the interstitial spaces. This is known as edema. The nurse should note whether the client feels that his or her shoes or rings are tight, and should observe, when present, an indentation around the leg where the socks end. The other answers do not describe edema.

A nurse obtains the health history of a client who is newly admitted to the medical unit. Which statement by the client should alert the nurse to the presence of edema? a. "I wake up to go to the bathroom at night." b. "My shoes fit tighter by the end of the day." c. "I seem to be feeling more anxious lately." d. "I drink at least eight glasses of water a day."

ANS: A, B, D Interventions to decrease thick tenacious secretions include maintaining adequate hydration and providing humidified oxygen. These actions will help to thin secretions, making them easier to remove by coughing. The use of a vibrating positive expiratory pressure device can also help clients remove thick secretions. Although suctioning may assist with the removal of secretions, frequent suctioning can cause airway trauma and does not support the client's ability to successfully remove secretions through normal coughing. Diaphragmatic breathing is not used to improve the removal of thick secretions.

A nurse plans care for a client who has chronic obstructive pulmonary disease and thick, tenacious secretions. Which interventions should the nurse include in this client's plan of care? (Select all that apply.) a. Ask the client to drink 2 liters of fluids daily. b. Add humidity to the prescribed oxygen. c. Suction the client every 2 to 3 hours. d. Use a vibrating positive expiratory pressure device. e. Encourage diaphragmatic breathing.

ANS: A, B, C Fluids can make a client feel bloated and should be avoided with meals. Resting before the meal will help a client with dyspnea. Six small meals a day also will help to decrease bloating. Fibrous foods can produce gas, which can cause abdominal bloating and can increase shortness of breath. The client should increase calorie and protein intake to prevent malnourishment. The client should not increase carbohydrate intake as this will increase carbon dioxide production and increase the client's risk of for acidosis.

A nurse teaches a client who has chronic obstructive pulmonary disease. Which statements related to nutrition should the nurse include in this client's teaching? (Select all that apply.) a. "Avoid drinking fluids just before and during meals." b. "Rest before meals if you have dyspnea." c. "Have about six small meals a day." d. "Eat high-fiber foods to promote gastric emptying." e. "Increase carbohydrate intake for energy."

ANS: A, C, D, E Several factors cause red blood cells to sickle in SCD, including dehydration, extreme stress, high altitudes, and pregnancy. Strenuous exercise can also cause sickling, but not unless it is very vigorous.

A nurse working with clients with sickle cell disease (SCD) teaches about self-management to prevent exacerbations and sickle cell crises. What factors should clients be taught to avoid? (Select all that apply.) a. Dehydration b. Exercise c. Extreme stress d. High altitudes e. Pregnancy

ANS: B Clients are often in denial after a coronary event. The client who seems to be in denial but is compliant with treatment may be using a healthy form of coping that allows time to process the event and start to use problem-focused coping. The student should not discourage this type of denial and coping, but rather continue providing education in a positive manner. Emphasizing complications may make the client defensive and more anxious. Telling the client that denial is normal is placing too much attention on the process. Forcing the client to verbalize understanding of the illness is also potentially threatening to the client.

A nursing student is caring for a client who had a myocardial infarction. The student is confused because the client states nothing is wrong and yet listens attentively while the student provides education on lifestyle changes and healthy menu choices. What response by the experienced nurse is best? a. "You need to make sure the client understands this illness." b. "Continue to educate the client on possible healthy changes." c. "Emphasize complications that can occur with noncompliance." d. "Tell the client that denial is normal and will soon go away."

ANS: D Hypoxia occurs because of decreased circulating blood volume, which leads to decreased oxygen to muscles, causing fatigue, decreased activity tolerance, and a feeling of shortness of breath.

A patient has been diagnosed with severe iron deficiency anemia. During physical assessment for which of the following symptoms would the nurse assess to determine the patient's oxygen status? a. Increased breathlessness but increased activity tolerance b. Decreased breathlessness and decreased activity tolerance c. Increased activity tolerance and decreased breathlessness d. Decreased activity tolerance and increased breathlessness

ANS: D Hypoxia is the drive to breathe in a patient with chronic obstructive pulmonary disease who has become used to acidic pH and elevated CO2 levels. Turning up to 6 L/min increases the oxygen level, which turns off the drive to breathe.

A patient has been newly diagnosed with emphysema. In discussing his condition with the nurse, which of his statements would indicate a need for further education? a. "I'll make sure that I rest between activities so I don't get so short of breath." b. "I'll rest for 30 minutes before I eat my meal." c. "If I have trouble breathing at night, I'll use two to three pillows to prop up." d. "If I get short of breath, I'll turn up my oxygen level to 6 L/min."

ANS: B Carbon monoxide strongly binds to hemoglobin, making it unavailable for oxygen binding and transport.

A patient is admitted to the emergency department with suspected carbon monoxide poisoning. Even though the patient's color is ruddy, not cyanotic, the nurse understands that the patient is at a risk for decreased oxygen-carrying capacity of blood because carbon monoxide does which of the following: a. Stimulates hyperventilation, causing respiratory alkalosis b. Forms a strong bond with hemoglobin, creating a functional anemia. c. Stimulates hypoventilation, causing respiratory acidosis d. Causes alveoli to overinflate, leading to atelectasis

ANS: D Decreased effective contraction of left side of heart leads to back up of fluid in the lungs, increasing hydrostatic pressure and causing pulmonary edema, resulting in crackles in lung bases.

A patient is admitted with the diagnosis of severe left-sided heart failure. The nurse expects to auscultate which adventitious lung sounds? a. Sonorous wheezes in the left lower lung b. Rhonchi midsternum c. Crackles only in apex of lungs d. Inspiratory crackles in lung bases

ANS: A When the lung collapses, the thoracic space fills with air on each inspiration, and the atmospheric air irritates the parietal pleura, causing pain.

A patient was admitted after a motor vehicle accident with multiple fractured ribs. Respiratory assessment includes signs/symptoms of secondary pneumothorax, which includes which of the following? a. Sharp pleuritic pain that worsens on inspiration b. Crackles over lung bases of affected lung c. Tracheal deviation toward the affected lung d. Increased diaphragmatic excursion on side of rib fractures

ANS: D Effects of nicotine on blood vessels and lung tissue have been proven to increase pathological changes, leading to heart disease and lung cancer.

A patient who started smoking in adolescence and continues to smoke 40 years later comes to the clinic. The nurse understands that this patient has an increased risk for being diagnosed with which disorder: a. Alcoholism and hypertension b. Obesity and diabetes c. Stress-related illnesses d. Cardiopulmonary disease and lung cancer

ANS: A Clubbing of the nail bed is a frequent symptom of COPD and can make activities of daily living difficult. Taking a nap decreases fatigue but does not help the patient perform fine motor skills. Loss of mental status is not a normal finding with COPD. Low oxygen not low circulating blood volume is the problem in COPD.

A patient with COPD asks the nurse why he is having increased difficulty with his fine motor skills, such as buttoning his shirt. Which response by the nurse is most therapeutic? a. "Your body isn't receiving enough oxygen to send down to your fingers; this causes them to club and makes dexterity difficult." b. "Your disease process makes even the smallest tasks seem exhausting. Try taking a nap before getting dressed." c. "Often patients with your disease lose mental status and forget how to perform daily tasks." d. "Your disease affects both your lungs and your heart, and not enough blood is being pumped. So you are losing sensory feedback in your extremities."

ANS: B The client needs IV pain relief, and it should be administered on a routine schedule (i.e., before the client has to request it). Morphine is not administered intramuscularly (IM) to clients with sickle cell disease (SCD). In fact, all IM injections are avoided because absorption is impaired by poor perfusion and sclerosed skin. Nonsteroidal anti-inflammatory drugs may be used for clients with SCD for pain relief once their pain is under control; however, in a crisis, this choice of analgesic is not strong enough. Moderate pain may be treated with oral opioids, but this client is in a sickle cell crisis; IV analgesics should be used until his or her condition stabilizes.

A recently admitted client who is in sickle cell crisis requests "something for pain." What does the nurse administer? a. IM morphine sulfate b. IV hydromorphone (Dilaudid) c. Oral ibuprofen (Motrin) d. Oral morphine sulfate (MS-Contin)

A nurse is preparing an IV solution for a patient who has hypernatremia. Which solutions are the best choices for this condition? Select all that apply. a. 5% dextrose in water (D5W) b. 0.9% NaCl (normal saline) c. Lactated Ringer's solution d. 0.33% NaCl (¹∕³-strength normal saline) e. 0.45% NaCl (½-strength normal saline) f. 10% dextrose in water (D10W)

A, D, E 5% dextrose in water (D5W), 0.33% NaCl (¹∕³-strength normal saline), and 0.45% NaCl (½-strength normal saline) are used to treat hypernatremia. 0.9% NaCl (normal saline) is used to treat hypovolemia, metabolic alkalosis, hyponatremia, and hypochloremia. Lactated Ringer's solution is used in the treatment of hypovolemia, burns, and fluid lost from gastrointestinal sources. 10% dextrose in water (D10W) is used in peripheral parenteral nutrition (PPN).

A 68-year-old man is admitted to the hospital with dehydration. He has a history of atrial fibrillation, congestive heart failure (CHF), and hypertension. His current medications are digoxin (Lanoxin), chlorothiazide (Diuril), and potassium supplements. He tells a nurse that he has had flulike symptoms for the past week and has been unable to drink for the past 48 hours. The nurse starts the client's IV and receives laboratory results, which include a potassium level of 2.7 mEq/L. The physician orders an IV potassium supplement. How does the nurse administer this medication? a. Added to an IV, not to exceed 20 mEq/hr b. Added to an IV, not to exceed 30 mEq/hr c. Rapid IV push, a 25-mEq dose d Slow IV push, a 30-mEq dose

A. Added to an IV, not to exceed 20 mEq/hr The maximum recommended infusion rate of potassium is 5-10 mEq/hr. This rate is never to exceed 20 mEq/hr under any circumstances. Potassium should never be administered via IV push.

Laboratory results for a client with a large draining abdominal wound show a serum sodium decrease from 138 mEq/L to 131 mEq/L. What is the nurse's first action? a. Assess the client's respiratory status. b. Establish intravenous access. c. Notify the provider of laboratory results. d. Assess for orthostatic hypotension.

A. Assess the client's respiratory status. Hyponatremia may present with neuromuscular changes including muscle weakness of the legs, arms, and respiratory muscles. The nurse should assess the respiratory effectiveness of a client with hyponatremia as a priority. Obtaining assessment data is important when calling the provider in addition to reporting the laboratory result. Establishing IV access and assessing for orthostatic hypotension are important, but are lower-priority interventions.

The nurse manager of a medical-surgical unit is completing assignments for the day shift staff. The client with which electrolyte laboratory value is assigned to the LPN/LVN? a. Calcium level of 9.5 mg/dL b. Magnesium level of 4.1 mEq/L c. Potassium level of 6.0 mEq/L d. Sodium level of 120 mEq/L

A. Calcium level of 9.5 mg/dL Because a calcium level of 9.5 mg/dL is within normal limits, it is appropriate to assign this client to an LPN/LVN. A magnesium level of 4.1 mEq/L, potassium level of 6.0 mEq/L, and a sodium level of 120 mEq/L are abnormalities in electrolytes that can cause serious complications and will require assessments and/or interventions by the RN.

Which situation can cause a client to experience "insensible water loss"? Select all that apply. a. Diarrhea b. Dry, hot weather c. Fever d. Increased respiratory rate e. Nausea f. Mechanical ventilation

A. Diarrhea B. Dry, hot weather C. Fever D. Increased respiratory rate F. Mechanical ventilation Insensible water loss occurs through the intestinal tract as diarrhea. It can be caused and/or influenced by dry, hot weather. Insensible water loss occurs through the skin, lungs (increased rate of respirations), and intestinal tract. It is increased in clients who are mechanically ventilated, and it is increased by the presence of fever. Nausea with no accompanying vomiting would not cause insensible water loss.

How is isotonic dehydration defined? a. Fluids and electrolytes are lost from the body in equal amounts. b. Fluids are lost from the body in greater quantities than electrolytes. c. Electrolytes are lost from the body in greater quantities than fluids. d. Fluids and electrolytes are lost from the body, but only water is used to replace losses.

A. Fluids and electrolytes are lost from the body in equal amounts. Isotonic dehydration is when fluids and electrolytes are lost in equal quantities. Hypertonic dehydration occurs when fluids are lost in greater quantities than electrolytes. Hypotonic dehydration occurs when electrolytes are lost in greater quantities than fluids, or when fluid and electrolyte losses are replaced by water only.

A 77-year-old woman is brought to the emergency department by her family after she has had diarrhea for 3 days. The family tells the nurse that she has not been eating or drinking well, but that she has been taking her diuretics for congestive heart failure (CHF). Laboratory results include a potassium level of 7.0 mEq/L. Which medication does the nurse anticipate administering? a. Insulin (regular insulin) and dextrose (D20W) b. Loperamide (Imodium) c. Sodium polystyrene sulfonate (Kayexalate) d. Supplemental potassium

A. Insulin (regular insulin) and dextrose (D20W) If potassium levels are high, a combination of 20 units of regular insulin in 100 mL of 20% dextrose may be prescribed to promote movement of potassium from the blood into the intracellular fluid (ICF). Imodium is used in the treatment of diarrhea. Kayexalate is used for hyperkalemia, but not when the potassium level is this high (7.0). Additional potassium would make the client's condition more critical.

ANS: A The low white blood cell count indicates that the patient is at high risk for infection and needs immediate actions to diagnose and treat the cause of the leucopenia. The other information may require further assessment or treatment, but does not place the patient at immediate risk for complications.

The nurse has obtained the health history, physical assessment data, and laboratory results shown in the accompanying figure for a patient admitted with aplastic anemia. Which information is most important to communicate to the health care provider? a. Neutropenia b. Increasing fatigue c. Thrombocytopenia d. Frequent constipation

A client has had a total urine output of 200 mL in the past 24 hours. Which priority action does the nurse take? a. Notify the primary health care provider. b. Encourage the client to drink more fluids. c. Take no action because this is a normal urinary output. d. Insert an indwelling urinary catheter.

A. Notify the primary health care provider. The minimum amount of urine per day needed to excrete toxic waste products is 400 to 600 mL. With a urine output of 200 mL in 24 hours, a toxic buildup of nitrogen and lethal electrolyte imbalances can occur. The primary health care provider should be notified because additional tests or orders might be necessary. Encouraging the client to drink more fluids may be necessary once the underlying cause of the decreased urine output (oliguria) is determined. Inserting an indwelling urinary catheter may be indicated, but not before speaking with the provider.

A nurse is preparing a presentation for a group of older adults at a local senior center about the importance of fluid intake. As part of the presentation, the nurse plans to discuss how the intake and output of fluids is typically balanced each day. When describing the normal daily output of fluids, which component would the nurse identify as accounting for the smallest amount of fluid output? a. Perspiration b. Feces c. Exhaled air d. Urine

A. Perspiration Normal urine output for 24 hours is approximately 1,500 mL if intake is normal. Loss of fluid through the skin as perspiration accounts for an average daily loss of 100 to 200 mL of fluid. In addition to perspiration, insensible fluid loss through the skin amounts to about 300 to 400 mL per day. Loss of fluid through the gastrointestinal system in the form of feces is usually minimal, approximately 200 mL per day. Loss of water through respiration is approximately 300 mL per day.

A client with diabetes is prescribed insulin glargine once daily and regular insulin four times daily. One dose of regular insulin is scheduled at the same time as the glargine. How does the nurse instruct the client to administer the two doses of insulin? a. "Draw up and inject the insulin glargine first, then draw up and inject the regular insulin." b. "Draw up and inject the insulin glargine first, wait 20 minutes, then draw up and inject the regular insulin." c. "First draw up the dose of regular insulin, then draw up the dose of insulin glargine in the same syringe, mix, and inject the two insulins together." d. "First draw up the dose of insulin glargine, then draw up the dose of regular insulin in the same syringe, mix, and inject the two insulins together."

ANS: A Insulin glargine must not be diluted or mixed with any other insulin or solution. Mixing results in an unpredictable alteration in the onset of action and time to peak action. The correct instruction is to draw up and inject first the glargine, then the regular insulin right afterward.

A client with diabetes is visually impaired and wants to know whether syringes can be prefilled and stored for later use. Which is the nurse's best response? a. "Yes. Prefilled syringes can be stored for 3 weeks in the refrigerator in a vertical position with the needle pointing up." b. "Yes. Prefilled syringes can be stored for up to 3 weeks in the refrigerator, placed in a horizontal position." c. "Insulin reacts with plastic, so prefilled syringes are okay, but they must be made of glass." d. "No. Insulin cannot be stored for any length of time outside of the container."

ANS: A Insulin is relatively stable when stored in a cool, dry place away from light. When refrigerated, prefilled syringes are stable for up to 3 weeks. They should be stored in the vertical position with the needle pointing up to prevent suspended insulin particles from clogging the needle. The other answers are inaccurate.

What is the best response for the nurse to give a parent about contacting the physician regarding an infant with diarrhea? a. "Call your pediatrician if the infant has not had a wet diaper for 6 hours." b. "The pediatrician should be contacted if the infant has two loose stools in an 8-hour period." c. "Call the doctor immediately if the infant has a temperature greater than 100° F." d. "Notify the pediatrician if the infant naps more than 2 hours."

ANS: A No urine output in 6 hours needs to be reported because it indicates dehydration. Two loose stools in 8 hours is not a serious concern. If blood is obvious in the stool or the frequency increases to one bowel movement every hour for more than 8 hours, the physician should be notified. A fever greater than 101° F should be reported to the infant's physician. It is normal for the infant who is not ill to nap for 2 hours. The infant who is ill may nap longer than the typical amount.

A new nurse graduate is caring for a postoperative client with the following arterial blood gas values: pH, 7.30; PaCO2, 60 mm Hg; PaO2, 80 mm Hg; bicarbonate, 24 mEq/L; and O2 saturation, 96%. Which of these actions by the new graduate is indicated? a. Encourage the client to use the incentive spirometer and cough. b. Administer oxygen by nasal cannula. c. Request a prescription for sodium bicarbonate from the health care provider

ANS: A Respiratory acidosis is caused by CO2 retention and impaired chest expansion secondary to anesthesia. The nurse takes steps to promote CO2 elimination, including maintaining a patent airway and expanding the lungs through breathing techniques. O2 is not indicated because PaO2 and oxygen saturation are within the normal range. Sodium bicarbonate is not indicated because the bicarbonate level is in the normal range; promoting excretion of respiratory acids is the priority in respiratory acidosis. Postanesthesia, the client will need interventions related to promoting CO2 elimination or the client may progress to a state of somnolence and unresponsiveness.

A morbidly obese client has chosen gastric bypass surgery to promote weight loss. The nurse plans to teach the client about the need to perform monitoring to detect what disturbance consistent with rapid weight loss associated with this procedure? a. Ketosis b. Hypoxemia c. Urinary retention d. Insufficient ventilation

ANS: A Starvation, fasting, or following a strict calorie-reduced diet with rapid weight loss contributes to ketone formation and metabolic acidosis. Weight loss should decrease the work of breathing and improve hypoxemia, if present, as well as hypoventilation. Urinary retention does not occur in acid-base imbalance.

The nurse has been reviewing options for insulin therapy with several clients. For which client does the nurse choose to recommend the pen-type injector insulin delivery system? a. Older adult client who lives at home alone but has periods of confusion b. Client on an intensive regimen with frequent, small insulin doses c. Client from the low-vision clinic who has trouble seeing the syringe d. "Brittle" client who has frequent episodes of hypoglycemia

ANS: B The pen-type injector allows greater accuracy with small doses, especially doses lower than 5 units. It is not recommended for those who have visual or neurologic impairments. The client with frequent hypoglycemia would not derive special benefit from using the pen.

The parents of a child with acid-base imbalance ask the nurse about mechanisms that regulate acid-base balance. Which statement by the nurse accurately explains the mechanisms regulating acid-base balance in children? a. The respiratory, renal, and chemical-buffering systems b. The kidneys balance acid; the lungs balance base c. The cardiovascular and integumentary systems d. The skin, kidney, and endocrine systems

ANS: A The acid-base system is regulated by chemical buffering, respiratory control of carbon dioxide, and renal regulation of bicarbonate and secretion of hydrogen ions. Both the kidneys and the lungs, along with the buffering system, contribute to acid-base balance. Neither system regulates acid or base balances exclusively. The cardiovascular and integumentary systems are not part of acid-base regulation in the body. Chemical buffers, the lungs, and the kidneys work together to keep the blood pH within normal range.

Which action is the primary concern in the treatment plan for a child with persistent vomiting? a. Detecting the cause of vomiting b. Preventing metabolic acidosis c. Positioning the child to prevent further vomiting d. Recording intake and output

ANS: A The primary focus of managing vomiting is detection of the cause and then treatment of the cause. Metabolic alkalosis results from persistent vomiting. Prevention of complications is the secondary focus of treatment. The child with persistent vomiting should be positioned upright or side-lying to prevent aspiration. Recording intake and output is a nursing intervention, but it is not the primary focus of treatment.

What priority intervention will the nurse employ to prevent injury to the patient with bone cancer? a. Using a lift sheet when repositioning the patient b. Positioning the patient so the heels do not touch the mattress c. Providing small, frequent meals rich in calcium and phosphorus d. Applying pressure for a full 5 minutes after intramuscular injections

ANS: A The resultant bone destruction from bone cancer can cause pathological fractures by grasping or pulling on a patient by the extremities or trunk of the body during re-positioning. Use of a lift sheet evenly distributes the patient's weight, lessening the chance of fractures occurring. While safety risks exist, the priority for bone cancer is reducing risk of fractures.

A client who has been taking pioglitazone (Actos) for 6 months reports to the nurse that his urine has become darker since starting the medication. Which is the nurse's first action? a. Review results of liver enzyme studies. b. Document the report in the client's chart. c. Instruct the client to increase water intake. d. Test a sample of urine for occult blood.

ANS: A Thiazolidinediones (including pioglitazone) can affect liver function; liver function should be assessed at the start of therapy and at regular intervals while the client continues to take these drugs. Dark urine is one indicator of liver impairment because bilirubin is increased in the blood and is excreted in the urine. The nurse should check the client's most recent liver function studies. Documentation should be done after all assessments have been completed. The client does not need to be told to increase water intake, and the nurse does not need to check the urine for occult blood.

Which information must the organ transplant nurse emphasize before a client is discharged? a. "Taking immunosuppressant medications increases your risk for cancer and the need for screenings." b. "You are at increased risk for cancer when you reach 60 years of age." c. "Immunosuppressant medications will decrease your risk for developing cancers." d. "After 6 months, you may stop immunosuppressant medications, and your risk for cancer will be the same as that of the general population."

ANS: A Use of immunosuppressant medications to prevent organ rejection increases the risk for cancer. Advanced age is a risk factor for all people, not just for organ transplant recipients. Immunosuppressant medications must be taken for the life of the organ; the risk for developing cancer remains.

A nurse evaluates a client's arterial blood gas values (ABGs): pH 7.30, PaO2 86 mm Hg, PaCO2 55 mm Hg, and HCO3- 22 mEq/L. Which intervention should the nurse implement first? a. Assess the airway. b. Administer prescribed bronchodilators. c. Provide oxygen. d. Administer prescribed mucolytics.

ANS: A All interventions are important for clients with respiratory acidosis; this is indicated by the ABGs. However, the priority is assessing and maintaining an airway. Without a patent airway, other interventions will not be helpful.

While collecting a health history on a patient admitted for colon cancer, which of the following questions would be a priority to ask this patient? a. "Have you noticed any blood in your stool?" b. "Have you been experiencing nausea?" c. "Do you have back pain?" d. "Have you noticed any swelling in your abdomen?"

ANS: A Early colon cancer is often asymptomatic, with occult or frank blood in the stool being an assessment finding in a patient diagnosed with colon cancer. If pain is present, it is usually lower abdominal cramping. Constipation and diarrhea are more frequent findings than nausea or ascites.

Which activity performed by the community health nurse best reflects primary prevention of cancer? a. Assisting women to obtain free mammograms b. Teaching a class on cancer prevention c. Encouraging long-term smokers to get a chest x-ray d. Encouraging sexually active women to get annual Papanicolaou (Pap) smears

ANS: B Primary prevention involves avoiding exposure to known causes of cancer; education assists clients with this strategy. Mammography is part of a secondary level of prevention, defined as screening for early detection. Chest x-ray is a method of detecting a cancer that is present—secondary prevention and early detection. A Pap smear is a means of detecting cervical cancer early—secondary prevention.

A toddler is hospitalized with severe dehydration. The nurse should assess the child for which possible complication? A. Hypertension B. Hypokalemia C. A rapid, bounding pulse D. Decreased specific gravity

ANS: B The child needs to be monitored for hypotension. Hypokalemia is a concern in severe dehydration. A rapid, thready pulse would be seen in severe dehydration. The urine would be concentrated, so the specific gravity would increase.

The nurse is assessing a client for bladder cancer. About which sign or symptom does the nurse ask the client? a. Unexplained fevers b. Presence of blood in the urine c. History of urinary tract infections d. Change in the size of urine stream

ANS: B The nurse asks the client about presence of blood in the urine which is often found in clients with bladder cancer. Unexplained fevers may be assessed in the client with leukemia. During an assessment for prostate cancer, the nurse would ask the client about a history of urinary tract infections and a change in the size of urine stream because enlargement of the prostate affects the urinary bladder, thereby affecting the size of the urine stream. The client experiences urine retention and has repeated urinary tract infections.

Which question is most important for the nurse to ask the client who has a serum potassium level of 2.9 mEq/L? a. "Do you use sugar substitutes?" b. "Do you use diuretics or laxatives?" c. "Have you had any muscle twitches or cramps, especially at night?" d. "Have you or any member of your family ever been diagnosed with lung disease?"

ANS: B The serum potassium level is low, and the client has hypokalemia. Misuse or overuse of diuretics, especially high-ceiling (loop) and thiazide diuretics, and laxatives are common causes of hypokalemia among older adults and clients with eating disorders. Sugar substitutes do not change serum potassium levels. Muscle cramps and twitching may occur with hyperkalemia and hypocalcemia but not with hypokalemia. Lung disease is not associated with hypokalemia.

When teaching women about the risk of breast cancer, which risk factor does the nurse know is the most common for the development of the disease? a. Having an aunt with breast cancer b. Being an older adult c. Being a Euro-American d. Consuming a low-fat diet

ANS: B There is no single-known cause for breast cancer. Being an older woman or man is the primary risk factor, although some people are at higher risk than others. Having a first-degree relative (mother, sister, or daughter) with breast cancer can increase the risk; an aunt is not considered a first-degree relative. Although Euro-American women older than 40 years are at a more increased risk than other racial/ethnic groups, the greater risk is being an older adult. Consuming a high-fat diet is considered a risk factor.

The nurse suspects metastasis from left breast cancer to the thoracic spine when the client has which symptom? a. Vomiting b. Back pain c. Frequent urination d. Cyanosis of the toes

ANS: B Typical sites of breast cancer metastasis include bone, manifested by back pain, lung, liver, and brain. Signs of metastasis to the spine may include numbness, pain, paresthesias and tingling, and loss of bowel and bladder control. Although frequent urination may be a sign of bladder cancer, incontinence is more indicative of spinal metastasis. Cyanosis of the toes indicates decreased tissue perfusion, often related to atherosclerotic disease.

How is the migratory feature of cancer cells explained? a. They have a large nuclear-to-cytoplasmic ratio. b. They lose their specific functions. c. They do not make fibronectin. d. They have a short generation time.

ANS: C Fibronectin is a protein that protrudes from the membrane of normal cells, allowing them to bind tightly together. Cancer cells do not make fibronectin; hence they adhere loosely to each other and break off from the main tumor. These cells easily slip through the walls of the blood vessels and migrate to other body sites. Cancer cells have a large nuclear-to-cytoplasmic ratio; they lose their specific functions and have a short generation time. But it is the absence of fibronectin that makes these cells migratory.

What assessment should the nurse make before initiating an intravenous (IV) infusion of dextrose 5% in 0.9% normal saline solution with 10 mEq of potassium chloride for a child hospitalized with dehydration? a. Fluid intake b. Number of stools c. Urine output d. Capillary refill

ANS: C Fluid intake does not give information about renal function. Stool count sheds light on intestinal function. Renal function is the concern before potassium chloride is added to an IV solution. Potassium chloride should never be added to an IV solution in the presence of oliguria or anuria (urine output less than 0.5 mL/kg/hr). Assessment of capillary refill does not provide data about renal function.

A patient has just been diagnosed with diabetes mellitus. His doctor has requested glucagon for emergency use at home. The nurse instructs the patient that the purpose of this drug is to treat: a. Hyperglycemia from insufficient insulin injection. b. Hyperglycemia from eating a large meal. c. Hypoglycemia from insulin overdose. d. Lipohypertrophy from inadequate insulin absorption.

ANS: C Glucagon is for emergency use for insulin overdose. The patient will usually arouse within 20 minutes if unconscious. The family should also be instructed how to use the glucagon injection as well.

The nurse is aware that an abdominal mass found in a 10-month-old infant corresponds with which childhood cancer? a. Osteogenic sarcoma b. Rhabdomyosarcoma c. Neuroblastoma d. Non-Hodgkin lymphoma

ANS: C Neuroblastoma is found exclusively in infants and children. In most cases of neuroblastoma, a primary abdominal mass and protuberant, firm abdomen are present. Osteogenic sarcoma is a bone tumor. Bone tumors typically affect older children. Rhabdomyosarcoma is a malignancy of muscle, or striated tissue. It occurs most often in the periorbital area, in the head and neck in younger children, or in the trunk and extremities in older children. Non-Hodgkin lymphoma is a neoplasm of lymphoid cells. Painless, enlarged lymph nodes are found in the cervical or axillary region. Abdominal signs and symptoms do not include a mass.

A client who has just had a mastectomy is crying. When the nurse asks about her crying, the client responds, "I know I shouldn't cry because this surgery may well save my life." What is the nurse's best response? a. "It is all right to cry. Mourning this loss will help make you stronger." b. "I know this is hard, but your chances of survival are better now." c. "I can arrange for someone who had a mastectomy to come visit if you like." d. "How have you coped with difficult situations in the past?"

ANS: C Often, cancer surgery involves the loss of a body part or a decrease in function. Mourning or grieving for a body image alteration is a healthy part of adapting or adjusting to a new image. Visiting with someone who has experienced the same situation as the client is very helpful in showing the client that many aspects of life can be the same afterward. If the opportunity to arrange this type of visit is available, this would be the nurse's best response. The other options do not provide any assistance to the client in coping with her new body image and grieving for her loss.

A 52-year-old client relates to the nurse that she has never had a mammogram because she is terrified that she will have cancer. Which response by the nurse is therapeutic? a. "Don't worry, most lumps are discovered by women during breast self-examination." b. "Does anyone in your family have breast cancer?" c. "Finding a cancer in the early stages increases the chance for cure." d. "Have you noticed a lump or thickening in your breast?"

ANS: C Providing truthful information addresses the client's concerns. Mammography can detect lumps smaller than those discovered by palpation. Asking about family history or symptoms is not therapeutic because it does not address the client's fear of cancer.

Which acid-base imbalance does the nurse anticipate the client with morbid obesity may develop? a. Metabolic acidosis b. Metabolic alkalosis c. Respiratory acidosis d. Respiratory alkalosis

ANS: C Respiratory acidosis is related to CO2 retention secondary to respiratory depression, inadequate chest expansion, airway obstruction, and reduced alveolar-capillary diffusion, which are common in morbidly obese clients who experience inadequate chest expansion owing to their size and work of breathing. Metabolic acidosis is related to overproduction of hydrogen ions, underelimination of hydrogen ions, underproduction of bicarbonate ions, and overelimination of bicarbonate ions. Metabolic alkalosis is related to loss of bicarbonate or buffers (i.e., vomiting or nasogastric suction). Respiratory alkalosis usually is caused by excessive loss of CO2 through hyperventilation secondary to fever, central nervous system lesions, and salicylates.

The nurse is caring for a client with an oxygen saturation of 88% and accessory muscle use. The nurse provides oxygen and anticipates which of these health care provider orders? a. Administration of IV sodium bicarbonate b. Computed tomography of the chest, STAT c. Intubation and mechanical ventilation d. Administration of concentrated potassium chloride solution

ANS: C Support with mechanical ventilation may be needed for clients who cannot keep their oxygen saturation at 90% or who have respiratory muscle fatigue. Sodium bicarbonate is used to treat metabolic acidosis; this client displays hypoxemia. Although the underlying reason for this client's hypoxemia may eventually require a diagnostic study, the priority is to restore oxygenation. No indication suggests that this client has hypokalemia; signs of hypoxemia and work of breathing are present, requiring correction with intubation and mechanical ventilation.

What should the nurse teach parents about oral hygiene for the child receiving chemotherapy? a. Brush the teeth briskly to remove bacteria. b. Use a mouthwash that contains alcohol. c. Inspect the child's mouth daily for ulcers. d. Perform oral hygiene twice a day.

ANS: C The child's mouth is inspected regularly for ulcers. At the first sign of ulceration, an antifungal drug is initiated. The teeth should be brushed with a soft-bristled toothbrush. Excessive force with brushing should be avoided because delicate tissue could be broken, causing infection or bleeding. Mouthwashes containing alcohol may be drying to oral mucosa, thus breaking down the protective barrier of the skin. Oral hygiene should be performed four times a day.

Why are infants at greater risk for fluid and electrolyte imbalances than older children? A. Their metabolic rate is lower. B. They have a decreased surface area. C. Their kidney functioning is immature. D. Their daily exchange of extracellular fluid is decreased.

ANS: C The infant has a higher metabolic rate. The infant has a proportionately greater body surface area, which allows for greater insensible water loss. The infant's kidneys are unable to concentrate or dilute urine, conserve or excrete sodium, and acidify urine. There is an increased amount of extracellular fluid in the infant. Forty percent of a neonate's body fluid is extracellular fluid, compared with 20% in an adult. Fluid is lost from the extracellular space first.

Which pathologic description of a client's tumor does the nurse interpret as being the "most malignant" or "high grade" cancer? a. poorly differentiated; mitotic index = 20%, euploid b. Moderately differentiated; mitotic index = 50%, euploid c. Undifferentiated; mitotic index = 50%, aneuploid d. Highly differentiated; mitotic index = 10%, aneuploid

ANS: C Tumors that closely resemble normal cells are "less malignant," and those that have few normal cell features are "more malignant." Thus, those that are euploid are less malignant and those that are aneuploid, with abnormal numbers or structures of chromosomes, are more malignant. Less malignant cells are highly differentiated, and more malignant cells are poorly or undifferentiated. Cells that divide faster (have a higher mitotic index) are more malignant.

What is a known cause of skin cancer? a. Intake of nitrites b. Cigarette smoke c. Tanning beds d. Low-fiber diet

ANS: C Ultraviolet radiation from tanning beds, cosmic radiation, germicidal lights, excessive exposure to the sun, and injuries from burns are known to cause skin cancer. Radiation mutates the genes and can cause cancer among non-dividing cells as well. Intake of nitrites from processed foods such as lunch meats, sausages and bacon increases the risk of cancer. Cigarette smoke is known to cause lung cancer. Although dietary factors like a low-fiber diet are suspected to alter cancer risk, their exact contribution is not clear.

The nurse interprets which arterial blood gas values as partially compensated metabolic acidosis? a. pH 7.28, HCO3- 19 mEq/L, PCO2 45 mm Hg, PO2 96 mm Hg b. pH 7.45, HCO3- 22 mEq/L, PCO2 40 mm Hg, PO2 98 mm Hg c. pH 7.32, HCO3- 17 mEq/L, PCO2 25 mm Hg, PO2 98 mm Hg d. pH 7.48, HCO3- 28 mEq/L, PCO2 45 mm Hg, PO2 92 mm Hg

ANS: C The pH is lower than normal, indicating mild acidosis. The acidosis is metabolic in origin, as indicated by the normal arterial oxygen partial pressure and the low bicarbonate level. The decreased carbon dioxide level indicates an increased respiratory rate, causing the carbon dioxide to be blown off and bringing the pH closer to normal (but not completely normal). Thus, the metabolic acidosis is only partially compensated for by the respiratory effort.

The home health nurse is caring for a client who has a history of a kidney transplant and takes cyclosporine (Sandimmune) and prednisone (Deltasone) to prevent rejection. Which assessment finding is most important to communicate to the transplant team? a. Temperature of 96.6° F b. Reports of joint pain c. Pink and dry oral mucosa d. Palpable lump in the client's axilla

ANS: D Clients taking immunosuppressive drugs to prevent rejection are at increased risk for the development of cancer; any lump should be reported to the physician. Fever should be reported to the physician, but this client's temperature is normal. It is not necessary to report joint pain to the transplant team; it is not a sign of rejection and is not a complication of transplant. A pink and dry oral mucosa may be a sign of dehydration, but it is not necessary to report this to the transplant team.

A client is learning to inject insulin. Which action is important for the nurse to teach the client? a. "Do not use needles more than twice before discarding." b. "Massage the site for 1 full minute after injection." c. "Try to make the injection deep enough to enter muscle." d. "Keep the vial you are using in the pantry or the bedroom drawer."

ANS: D Cold insulin directly from the refrigerator is the most common cause of irritation (not infection) at the insulin injection site. Insulin in active use can be stored at room temperature. However, the bathroom is not the best place to store any medication because of increased heat and humidity. Needles should be used only once. Massage will not prevent or treat irritation from cold insulin. Insulin is given by subcutaneous, not intramuscular, injection.

A nurse is teaching parents about diarrhea. Which statement by the parents indicates understanding of the teaching? a. Diarrhea results from a fluid deficit in the small intestine. b. Organisms destroy intestinal mucosal cells, resulting in an increased intestinal surface area. c. Malabsorption results in metabolic alkalosis. d. Increased motility results in impaired absorption of fluid and nutrients.

ANS: D Diarrhea results from fluid excess in the small intestine. Destroyed intestinal mucosal cells result in decreased intestinal surface area. Loss of electrolytes in the stool from diarrhea results in metabolic acidosis. Increased motility and rapid emptying of the intestines result in impaired absorption of nutrients and water. Electrolytes are drawn from the extracellular space into stool, and dehydration results.

What is the priority nursing intervention for a 6-month-old infant hospitalized with diarrhea and dehydration? a. Estimating insensible fluid loss b. Collecting urine for culture and sensitivity c. Palpating the posterior fontanel d. Measuring the infant's weight

ANS: D Infants have a greater total body surface area and therefore a greater potential for fluid loss through the skin. It is not possible to measure insensible fluid loss. Urine for culture and sensitivity is not usually part of the treatment plan for the infant who is dehydrated from diarrhea. The posterior fontanel closes by 2 months of age. The anterior fontanel can be palpated during an assessment of an infant with dehydration. Weight is a crucial indicator of fluid status. It is an important criterion for assessing hydration status and response to fluid replacement.

A client is receiving IV insulin for hyperglycemia. Which laboratory value requires immediate intervention by the nurse? a. Serum chloride level of 98 mmol/L b. Serum calcium level of 8.8 mg/dL c. Serum sodium level of 132 mmol/L d. Serum potassium level of 2.5 mmol/L

ANS: D Insulin activates the sodium-potassium ATPase pump, increasing the movement of potassium from the extracellular fluid into the intracellular fluid, resulting in hypokalemia. In hyperglycemia, hypokalemia can also result from excessive urine loss of potassium. The chloride level is normal. The calcium and sodium levels are slightly low, but this would not be related to hyperglycemia and insulin administration.

Which assessment finding indicates to the nurse that fluid resuscitation therapy for the client with isotonic dehydration is effective? a. Respiratory rate has changed from 16 to 18 breaths/min b. Urine specific gravity has increased from 1.040 to 1.050 c. Neck veins are flat when the client moves to a sitting position d. Pulse pressure has changed from 22 mm Hg to 32 mm Hg

ANS: D Isotonic dehydration manifests as hypovolemia and shock. The increasing pulse pressure (difference between the diastolic and systolic blood pressures) is an indication that the fluid volume deficit is being corrected.

Which type of cancer has been associated with Down syndrome? a. Breast cancer b. Colorectal cancer c. Malignant melanoma d. Leukemia

ANS: D Leukemia is associated with Down syndrome and Turner syndrome. Breast cancer is often found clustered in families, not in association with Down syndrome. Colorectal cancer is associated with familial polyposis. Malignant melanoma is associated with familial clustering and sun exposure.

The nurse monitors for which acid-base problem in a client who is taking furosemide (Lasix) for hypertension? a. Acid excess secondary to respiratory acidosis b. Acid deficit secondary to respiratory alkalosis c. Acid excess secondary to metabolic acidosis d. Acid deficit secondary to metabolic alkalosis

ANS: D Many diuretics, especially loop diuretics, increase the excretion of hydrogen ions, leading to excess acid loss through the renal system. This situation is an acid deficit of metabolic origin.

A 65-year-old client tells the nurse she does not have mammograms because there is no history of breast cancer in her family. What is the nurse's best response? a. "You are correct. Breast cancer is an inherited type of malignancy and your family history indicates a low risk for you." b. "Performing breast self-examination monthly at home is sufficient screening for someone with your family history." c. "Because your breasts are no longer as dense as they were when you were younger, your risk for breast cancer is now decreased." d. "Breast cancer can be found more frequently in families; however, the risk for general, nonfamilial breast cancer increases with age."

ANS: D Only a small percentage of cancers, including breast cancers, are hereditary or familial. The far more critically important risk factor for breast cancer in women is advancing age. Although performance of monthly self-breast examination is good, for a woman of this age, it should be done in conjunction with a yearly mammogram.

The nurse is giving a group presentation on cancer prevention and recognition. Which statement by an older adult client indicates understanding of the nurse's instructions? a. "Cigarette smoking always causes lung cancer." b. "Taking multivitamins will prevent me from developing cancer." c. "If I have only one shot of whiskey a day, I probably will not develop cancer." d. "I need to report the pain going down my legs to my health care provider."

ANS: D Pain in the back of the legs could indicate prostate cancer in an older man. Cigarette smoking is implicated in causing lung cancer and other types of cancer, but it does not always cause cancer. Investigation is ongoing about the efficacy of vitamins A and C in cancer prevention. Limiting alcohol to one drink per day is only one preventive measure.

The nurse is caring for a client with hypoxemia and metabolic acidosis. Which task can be delegated to the nursing assistant who is helping with the client's care? a. Assess the client's respiratory pattern. b. Increase the IV normal saline to 120 mL/hr c. Titrate O2 to maintain an O2 saturation of 95% to 100% d. Apply the pulse oximeter for continuous readings

ANS: D Placing a peripheral pulse oximeter is a standardized nursing skill that is within the scope of practice for unlicensed personnel. Assessment and intravenous therapy are skills performed by the professional nurse. Titration of O2 requires assessment and intervention beyond the scope of practice of an unlicensed individual.

A child has a 2-day history of vomiting and diarrhea. He has hypoactive bowel sounds and an irregular pulse. Electrolyte values are sodium, 139 mEq/L; potassium, 3.3 mEq/L; and calcium, 9.5 mg/dL. This child is likely to have which of the following electrolyte imbalances? a. Hyponatremia b. Hypocalcemia c. Hyperkalemia d. Hypokalemia

ANS: D The normal serum sodium level is 135 to 145 mEq/L. A level of 139 mEq/L is within normal limits. A serum calcium level less than 8.5 mg/dL is considered hypocalcemia. A serum potassium level greater than 5 mEq/L is considered hyperkalemia. A serum potassium level less than 3.5 mEq/L is considered hypokalemia. Clinical manifestations of hypokalemia include muscle weakness, decreased bowel sounds, cardiac irregularities, hypotension, and fatigue.

ANS: C Long-acting beta2 agonist medications will help prevent an acute asthma attack because they are long acting. The client will take this medication every day for best effect. The client does not have to always keep this medication with him or her because it is not used as a rescue medication. This is not the medication the client will use during an acute asthma attack because it does not have an immediate onset of action. The client will not be weaned off this medication because this is likely to be one of his or her daily medications.

After teaching a client who is prescribed a long-acting beta2 agonist medication, a nurse assesses the client's understanding. Which statement indicates the client comprehends the teaching? a. "I will carry this medication with me at all times in case I need it." b. "I will take this medication when I start to experience an asthma attack." c. "I will take this medication every morning to help prevent an acute attack." d. "I will be weaned off this medication when I no longer need it."

ANS: D, E, F In an older adult receiving a transfusion, hypertension is a sign of overload, low blood pressure is a sign of a transfusion reaction, and a rapid and bounding pulse is a sign of fluid overload. In this scenario, 2 units, or about a liter of fluid, could be problematic. Capillary refill time that is less than 3 seconds is considered to be normal and would not pose a problem. Increased (not decreased) pallor and cyanosis are signs of a transfusion reaction, while swollen (not flattened) superficial veins are present in fluid overload in older adult clients receiving transfusions.

An 82-year-old client with anemia is requested to receive 2 units of whole blood. Which assessment findings cause the nurse to discontinue the transfusion because it is unsafe for the client? (Select all that apply.) a. Capillary refill less than 3 seconds b. Decreased pallor c. Flattened superficial veins d. Hypertension e. Hypotension f. Rapid, bounding pulse

ANS: A Because sickle cell disease is commonly diagnosed during childhood, the pediatric nurse will be familiar with the disease and with red blood cell transfusion; therefore, he or she should be assigned to the client with sickle cell disease. Aplastic anemia, folic acid deficiency, and polycythemia vera are problems more commonly seen in adult clients who should be cared for by nurses who are more experienced in caring for adults.

An RN from pediatrics has "floated" to the medical-surgical unit. Which client is assigned to the float nurse? a. A 42-year-old with sickle cell disease receiving a transfusion of red blood cells b. A 50-year-old with pancytopenia needing assessment of risk factors for aplastic anemia c. A 55-year-old with folic acid deficiency anemia caused by alcohol abuse who needs counseling d. A 60-year-year with newly diagnosed polycythemia vera who needs teaching about the disease

ANS: B Nursing care for patients with anemia should alternate periods of rest and activity to encourage activity without causing undue fatigue. There is no indication that the patient has a bleeding disorder, so a diet high in vitamin K or teaching about how to avoid injury is not needed. Protective isolation might be used for a patient with aplastic anemia, but it is not indicated for hemolytic anemia.

An appropriate nursing intervention for a hospitalized patient with severe hemolytic anemia is to a. provide a diet high in vitamin K. b. alternate periods of rest and activity. c. teach the patient how to avoid injury. d. place the patient on protective isolation.

ANS: A Dyspnea on exertion is an early manifestation of heart failure and is associated with an activity such as stair climbing. The other findings are not specific to early occurrence of heart failure.

An emergency room nurse obtains the health history of a client. Which statement by the client should alert the nurse to the occurrence of heart failure? a. "I get short of breath when I climb stairs." b. "I see halos floating around my head." c. "I have trouble remembering things." d. "I have lost weight over the past month."

ANS: A Echocardiography is considered the best tool for the diagnosis of heart failure. A chest x-ray probably will be done, and if the client has dyspnea, an arterial blood gas will be drawn, but the echocardiogram is the priority. T4 and TSH might be ordered to assess for a contributing cause of heart failure.

An older adult client is admitted with fluid volume excess. Which diagnostic study does the nurse facilitate as a priority? a. Echocardiography b. Chest x-ray c. T4 and thyroid-stimulating hormone (TSH) d. Arterial blood gas

A nurse is assessing infants in the NICU for fluid balance status. Which nursing action would the nurse depend on as the most reliable indicator of a patient's fluid balance status? a. Recording intake and output b. Testing skin turgor c. Reviewing the complete blood count d. Measuring weight daily

D. Measuring weight daily Daily weight is the most reliable indicator of a person's fluid balance status. Intake and output are not always as accurate and may involve a subjective component. Measurement of skin turgor is subjective, and the complete blood count does not necessarily reflect fluid balance

ANS: B Intravenous nitroglycerin and furosemide will decrease the client's blood pressure, so it is important to monitor closely for hypotension. Intravenous medications are not administered under the tongue. Although the client may need an indwelling urinary catheter to monitor output, it is not the priority. The client's glucose levels should not be affected by these medications.

The client with heart failure has been prescribed intravenous nitroglycerin and furosemide (Lasix) for pulmonary edema. Which is the priority nursing intervention? a. Insert an indwelling urinary catheter. b. Monitor the client's blood pressure. c. Place the nitroglycerin under the client's tongue. d. Monitor the client's serum glucose level.

ANS: A Prolonged INR indicates that blood takes longer than normal to clot; this client is at risk for bleeding. PTT of 12.5 seconds and a platelet value of 170,000/mm3 are both normal and pose no risk for bleeding. Although a hemoglobin of 8.2 g/dL is low, the client could have severe iron deficiency or could have received medication affecting the bone marrow.

The client with which laboratory result is at risk for hemorrhagic shock? a. International normalized ratio (INR) 7.9 b. Partial thromboplastin time (PTT) 12.5 seconds c. Platelets 170,000/mm3 d. Hemoglobin 8.2 g/dL

ANS: A Esophageal varices are caused by portal hypertension; the portal vessels are under high pressure and are prone to rupture, causing massive upper gastrointestinal tract bleeding and hypovolemic shock. As the kidneys fail, fluid is typically retained, causing fluid volume excess, not hypovolemia. Nonsteroidal anti-inflammatory drugs such as naproxen and ibuprofen, not acetaminophen, predispose the client to gastrointestinal bleeding and hypovolemia. Although a kidney stone may cause hematuria, there is not generally massive blood loss or hypovolemia.

The client with which problem is at highest risk for hypovolemic shock? a. Esophageal varices b. Kidney failure c. Arthritis and daily acetaminophen use d. Kidney stone

ANS: D During severe sepsis, interventions should focus on decreasing hypoxia, maintaining acid-base balance, keeping blood glucose levels as normal as possible, maintaining organ perfusion, minimizing adrenal insufficiency, and decreasing microemboli. Treatment should include administration of low-dose corticosteroids, insulin drip with blood glucose checks every 1 to 2 hours, hourly intake and output monitoring, and an increase in ventilator rate and tidal volume.

The intensive care nurse is caring for an intubated client who has severe sepsis that led to acute respiratory distress. Which nursing intervention is most appropriate during this stage of sepsis? a. Check blood glucose levels every 4 hours. b. Monitor intake and urinary output twice each shift. c. Decrease ventilator rate and tidal volume. d. Administer prescribed low-dose corticosteroids.

ANS: A Any problem that impairs oxygen delivery to tissues and organs can start the syndrome of shock and lead to a life-threatening emergency. Shock represents the "whole-body response," affecting all organs in a predictable sequence. Compensation mechanisms attempt to maintain homeostasis and deliver necessary oxygen to organs but eventually will fail without reversal of the cause of shock, resulting in death.

The intensive care nurse is educating the spouse of a client who is being treated for shock. The spouse states, "The doctor said she has shock. What is that?" What is the nurse's best response? a. "Shock occurs when oxygen to the body's tissues and organs is impaired." b. "Shock is a serious condition, but it is not a life-threatening emergency." c. "Shock progresses slowly and can be stopped by the body's normal compensation." d. "Shock is a condition that affects only specific body organs like the kidneys."

ANS: B Because the child is not having difficulty breathing, the nurse should teach the parents the signs of respiratory distress and tell them to come to the emergency department if they develop. Cool mist is recommended to provide relief.

The mother of a 20-month-old child tells the nurse that the child has a barking cough at night. The child's temperature is 37 °C (98.6 °F). Based on the nurse's knowledge of upper respiratory infections, this is a symptom of croup. What should the nurse instruct the mother to do? a. Control the fever with acetaminophen and call if the cough gets worse tonight. b. Try a cool-mist vaporizer at night and watch for signs of difficulty breathing. c. Try over-the-counter cough medicine and come to the clinic tomorrow if there is no improvement. d. Take the child to the hospital in case epiglottitis occurs.

ANS: B, C, E Digoxin toxicity may cause bradycardia. Fatigue and anorexia are symptoms of digoxin toxicity. Hypokalemia causes increased sensitivity to the drug and toxicity, but it is not a symptom of toxicity. A serum digoxin level between 0.8 and 2.0 is considered normal and is not a symptom.

The nurse caring for a client with heart failure is concerned that digoxin toxicity has developed. For which signs and symptoms of digoxin toxicity does the nurse notify the provider? (Select all that apply.) a. Hypokalemia b. Sinus bradycardia c. Fatigue d. Serum digoxin level of 1.5 e. Anorexia

ANS: B With the neonate's first breath, gas exchange is transferred from the placenta to the lungs. The separation of the fetus from the umbilical cord does not contribute to the establishment of neonatal circulation. In the normal neonate, fetal shunts functionally close in response to pressure changes in the systemic and pulmonary circulations and to increased oxygen content. This process may take several days to complete. The fetal shunts normally close within several days of birth, but may take several days.

The nurse discovers a heart murmur in an infant 1 hour after birth. She is aware that fetal shunts are closed in the neonate at what point? a. When the umbilical cord is cut b. Within several days of birth c. Within a month after birth d. By the end of the first year of life

ANS: A Cutting out beef or hamburgers made at home is not necessary; however, fast-food hamburgers are to be avoided owing to higher sodium content. Bacon, canned foods, lunchmeats, and processed foods are high in sodium, which promotes fluid retention; these are to be avoided. The client correctly understands that adding salt to food should be avoided.

The nurse discusses the importance of restricting sodium in the diet for a client with heart failure. Which statement made by the client indicates that the client needs further teaching? a. "I should avoid eating hamburgers." b. "I must cut out bacon and canned foods." c. "I shouldn't put the salt shaker on the table anymore." d. "I should avoid lunchmeats but may cook my own turkey."

ANS: A This client has a falling systolic blood pressure, rising diastolic blood pressure, and narrowing pulse pressure, all of which may be indications of the progressive stage of shock. The nurse should assess this client first. The client with the unchanged oxygen saturation is stable at this point. Although the client with a change in pulse has a slower rate, it is not an indicator of shock since the pulse is still within the normal range; it may indicate the client's pain or anxiety has been relieved, or he or she is sleeping or relaxing. A urine output of 40 mL/hr is only slightly above the normal range, which is 30 mL/hr.

The nurse gets the hand-off report on four clients. Which client should the nurse assess first? a. Client with a blood pressure change of 128/74 to 110/88 mm Hg b. Client with oxygen saturation unchanged at 94% c. Client with a pulse change of 100 to 88 beats/min d. Client with urine output of 40 mL/hr for the last 2 hours

ANS: A Raising the head of the bed brings the diaphragm down and allows for better chest expansion, thus improving ventilation.

The nurse goes to assess a new patient and finds him lying supine in bed. The patient tells the nurse that he feels short of breath. Which nursing action should the nurse perform first? a. Raise the head of the bed to 45 degrees. b. Take his oxygen saturation with a pulse oximeter. c. Take his blood pressure and respiratory rate. d. Notify the health care provider of his shortness of breath.

ANS: C The client is displaying typical signs of acute pulmonary edema secondary to fluid-filled alveoli and pulmonary congestion; a diuretic will promote fluid loss. Although enalapril will promote vasodilation and decrease cardiac workload, the client is demonstrating signs of acute pulmonary edema secondary to intra-alveolar fluid. Heparin will prevent deep vein thrombosis secondary to immobility, but will not reduce fluid excess. Although all clients with congestive heart failure should have daily weights and I & O monitored, this is not a priority; removing fluid volume and treating dyspnea are matters of priority.

The nurse in the emergency department is caring for a client with acute heart failure who is experiencing severe dyspnea; pink, frothy sputum; and crackles throughout the lung fields. The nurse reviews the medical record, which contains the following information: Crackles in all fields S3 present Oliguria Ejection fraction 30% BNP 560 Sodium 130 mEq/L Diagnosis: heart failure Enalapril 10 mg orally daily Heparin 5000 units subcutaneously every 12 hours Furosemide 40 mg IV daily Strict I & O Which prescription does the nurse implement first? a. Enalapril b. Heparin c. Furosemide d. Intake and output (I&O)

ANS: C The proper order for correctly using an inhaler with a spacer is as follows. Insert the mouthpiece of the inhaler into the nonmouthpiece end of the spacer. Shake the whole unit vigorously three or four times. Place the mouthpiece into the mouth, over the tongue, and seal the lips tightly around it. Press down firmly on the canister of the inhaler to release one dose of medication into the spacer. Breathe in slowly and deeply. Remove the mouthpiece from the mouth, and, keeping the lips closed, hold the breath for at least 10 seconds. Then breathe out slowly. Wait at least 1 minute between puffs.

The nurse instructs a client on how to correctly use an inhaler with a spacer. In which order should these steps occur? 1. "Press down firmly on the canister to release one dose of medication." 2. "Breathe in slowly and deeply." 3. "Shake the whole unit vigorously three or four times." 4. "Insert the mouthpiece of the inhaler into the nonmouthpiece end of the spacer." 5. "Place the mouthpiece into your mouth, over the tongue, and seal your lips tightly around the mouthpiece." 6. "Remove the mouthpiece from your mouth, keep your lips closed, and hold your breath for at least 10 seconds." a. 2, 3, 4, 5, 6, 1 b. 3, 4, 5, 1, 6, 2 c. 4, 3, 5, 1, 2, 6 d. 5, 3, 6, 1, 2, 4

ANS: A The proper order for obtaining a peak expiratory flow rate is as follows. Make sure the device reads zero or is at base level. The client should stand up (unless he or she has a physical disability). The client should take as deep a breath as possible, place the meter in the mouth, and close the lips around the mouthpiece. The client should blow out as hard and as fast as possible for 1 to 2 seconds. The value obtained should be written down. The process should be repeated two more times, and the highest of the three numbers should be recorded in the client's chart.

The nurse instructs a client on the steps needed to obtain a peak expiratory flow rate. In which order should these steps occur? 1. "Take as deep a breath as possible." 2. "Stand up (unless you have a physical disability)." 3. "Place the meter in your mouth, and close your lips around the mouthpiece." 4. "Make sure the device reads zero or is at base level." 5. "Blow out as hard and as fast as possible for 1 to 2 seconds." 6. "Write down the value obtained." 7. "Repeat the process two additional times, and record the highest number in your chart." a. 4, 2, 1, 3, 5, 6, 7 b. 3, 4, 1, 2, 5, 7, 6 c. 2, 1, 3, 4, 5, 6, 7 d. 1, 3, 2, 5, 6, 7, 4

ANS: B Administration of the first dose of angiotensin-converting enzyme (ACE) inhibitors is often associated with hypotension, usually termed first-dose effect. The nurse should instruct the client to seek assistance before arising from bed to prevent injury from postural hypotension.

The nurse is administering captopril (Capoten) to a client with heart failure. What is the priority intervention for this client? a. Administer this medication before meals to aid absorption. b. Instruct the client to ask for assistance when arising from bed. c. Give the medication with milk to prevent stomach upset. d. Monitor the potassium level and check for symptoms of hypokalemia.

ANS: A Glucocorticoids (corticosteroids) decrease inflammation and prevent bronchospasm in the patient with asthma. The glucocorticoids are used to prevent problems. Anticholinergics decrease the allergic response and decrease sneezing and rhinorrhea. Antitussives are used to decrease cough, and mucolytics assist in the removal of mucus. Sympathomimetic agents (beta2 agonist) are used to relieve bronchospasm in an acute episode.

The nurse is administering oral glucocorticoids to a patient with asthma. What assessment finding would the nurse identify as a therapeutic response to this medication? a. No observable respiratory difficulty or shortness of breath over the last 24 hours. b. A decrease in the amount of nasal drainage and sneezing. c. No sputum production, and a decrease in coughing episodes. d. Relief of an acute asthmatic attack.

ANS: B The client receiving sodium nitroprusside should have his or her blood pressure assessed every 15 minutes. Higher doses can cause systemic vasodilation and can increase shock. The nurse should monitor the client's pain, urinary output, and extremities, but these assessments do not directly relate to the nitroprusside infusion.

The nurse is administering prescribed sodium nitroprusside (Nipride) intravenously to a client who has shock. Which nursing intervention is a priority when administering this medication? a. Ask if the client has chest pain every 30 minutes. b. Assess the client's blood pressure every 15 minutes. c. Monitor the client's urinary output every hour. d. Observe the client's extremities every 4 hours.

ANS: B Signs of systemic congestion occur with right-sided heart failure. Fluid is retained, pressure builds in the venous system, and peripheral edema develops. Left-sided heart failure symptoms include respiratory symptoms. Orthopnea, coughing, and difficulty breathing all could be results of left-sided heart failure.

The nurse is assessing a client admitted to the cardiac unit. What statement made by the client alerts the nurse to the possibility of right-sided heart failure? a. "I sleep with four pillows at night." b. "My shoes fit really tight lately." c. "I wake up coughing every night." d. "I have trouble catching my breath."

ANS: B Administration of oxygen for any type of shock is appropriate to help reduce potential damage from tissue hypoxia. The other interventions should be completed after oxygen is administered.

The nurse is assessing a client at risk for shock. The client's systolic blood pressure is 20 mm Hg lower than baseline. Which intervention does the nurse perform first? a. Increase the IV fluid rate. b. Administer oxygen. c. Notify the health care provider. d. Place the client in high Fowler's position.

ANS: B The syndrome of hypovolemic shock results in inadequate tissue perfusion and oxygenation; thus some cells are metabolizing anaerobically. Such metabolism increases the production of lactic acid, resulting in an increase in hydrogen ion production and acidosis. Other laboratory values associated with acidosis include increased creatinine (impaired renal function) and increased partial pressure of arterial carbon dioxide. Urine specific gravity is not associated with acidosis.

The nurse is assessing a client who has hypovolemic shock. Which laboratory value indicates that the client is at risk for acidosis? a. Decreased serum creatinine b. Increased serum lactic acid c. Increased urine specific gravity d. Decreased partial pressure of arterial carbon dioxide

ANS: A An increase in heart and respiratory rates (heart rate first) from the client's baseline and a slight increase in diastolic blood pressure may be the only objective manifestations of early shock. These findings do not correlate with other stages of shock.

The nurse is assessing a client who has septic shock. The following assessment data were collected: Baseline Data Today's Data Heart rate 75 beats/min 98 beats/min Blood pressure 125/65 mm Hg 128/75 mm Hg Respiratory rate 12 breaths/min 18 breaths/min Urinary output 40 mL/hr 40 mL/hr The nurse correlates these findings with which stage of shock? a. Early b. Compensatory c. Intermediate d. Refractory

ANS: A, B, D Heart and respiratory rates increased from the client's baseline level and a slight increase in diastolic blood pressure may be the only objective manifestations of this early stage of shock.

The nurse is assessing a client who is in early stages of hypovolemic shock. Which manifestations does the nurse expect? (Select all that apply.) a. Elevated heart rate b. Elevated diastolic blood pressure c. Decreased body temperature d. Elevated respiratory rate e. Decreased pulse rate

ANS: D The late phase of sepsis-induced distributive shock is characterized by most of the same cardiovascular manifestations as any other type of shock. The distinguishing feature is lack of ability to clot blood, causing the client to bleed from areas of minor trauma and to bleed spontaneously. The other manifestations are associated with all types of shock.

The nurse is assessing a client who was admitted for treatment of shock. Which manifestation indicates that the client's shock is caused by sepsis? a. Hypotension b. Pale clammy skin c. Anxiety and confusion d. Oozing of blood at the IV site

ANS: B Clients with a history of heart failure generally have negative findings, such as shortness of breath. The nurse needs to determine whether the client's activity is the same or worse, or whether the client identifies a decrease in activity level.

The nurse is assessing a client with a history of heart failure. What priority question assists the nurse to assess the client's activity level? a. "Do you have trouble breathing or chest pain?" b. "Are you able to walk upstairs without fatigue?" c. "Do you awake with breathlessness during the night?" d. "Do you have new-onset heaviness in your legs?"

ANS: A, B, E, F Left-sided failure occurs with a decrease in contractility of the heart or an increase in afterload. Most of the signs will be noted in the respiratory system. Right-sided failure occurs with problems from the pulmonary vasculature onward. Signs will be noted before the right atrium or ventricle.

The nurse is assessing a client with left-sided heart failure. What conditions does the nurse assess for? (Select all that apply.) a. Pulmonary crackles b. Confusion, restlessness c. Pulmonary hypertension d. Dependent edema e. S3/S4 summation gallop f. Cough worsens at night

ANS: C Hemoptysis is an abnormal occurrence of emphysema, and further diagnostic studies are needed to determine the cause of blood in the sputum. Clubbing of the fingers, barrel chest, and tachypnea are all normal findings in a patient with emphysema.

The nurse is assessing a patient with emphysema. Which assessment finding requires further follow-up with the physician? a. Clubbing of the fingers b. Increased anterior-posterior diameter of the chest c. Hemoptysis d. Tachypnea

ANS: C Certain conditions or treatments that cause immune suppression, such as having cancer and being treated with chemotherapeutic agents, aspirin, and certain antibiotics, can predispose a person to septic shock. The other client situations do not increase the client's risk for septic shock.

The nurse is assessing clients in the emergency department. Which client is at highest risk for developing septic shock? a. 25-year-old man who has irritable bowel syndrome b. 37-year-old woman who is 20% above ideal body weight c. 68-year-old woman who is being treated with chemotherapy d. 82-year-old man taking beta blockers for hypertension

ANS: A Although most people with heart failure will have failure that progresses from left to right, it is possible to have left-sided failure alone for a short period. It is also possible to have heart failure that progresses from right to left. Causes of left ventricular failure include mitral or aortic valve disease, coronary artery disease (CAD), and hypertension.

The nurse is assessing clients on a cardiac unit. Which client does the nurse assess most carefully for developing left-sided heart failure? a. Middle-aged woman with aortic stenosis b. Middle-aged man with pulmonary hypertension c. Older woman who smokes cigarettes daily d. Older man who has had a myocardial infarction

ANS: A During the hyperdynamic phase of septic shock, because of alterations in the clotting cascade, clients begin to form numerous small clots. Heparin is administered to limit clotting and prevent consumption of clotting factors. The other medications would not be prescribed during the hyperdynamic phase of septic shock.

The nurse is caring for a client in the hyperdynamic phase of septic shock. Which medication does the nurse expect to be prescribed? a. Heparin sodium b. Vitamin K c. Corticosteroids d. Hetastarch (Hespan)

ANS: D Anaphylaxis damages cells and causes release of large amounts of histamine and other inflammatory chemicals. This results in massive blood vessel dilation and increased capillary leak, which manifests as swelling. The other clinical manifestations do not relate to anaphylaxis or distributive shock.

The nurse is caring for a client who has had an anaphylactic event. Which priority question does the nurse ask to determine whether the client is experiencing distributive shock? a. "Is your blood pressure higher than usual?" b. "Are you having pain in your throat?" c. "Have you been vomiting?" d. "Are you usually this swollen?"

ANS: D Analgesics are needed to treat sickle cell pain. Warm soaks or compresses can help reduce pain perception. Cool compresses do not help the client in sickle cell crisis. Birth control is not the priority for this client. Increasing iron in the diet is not pertinent for the client in sickle cell crisis.

The nurse is caring for a client who is in sickle cell crisis. What action does the nurse perform first? a. Apply cool compresses to the client's forehead b. Encourage the client's use of two methods of birth control c. Increase food sources of iron in the client's diet d. Provide pain medications as needed.

ANS: D Long-acting beta2 agonists should be used every day to prevent asthma attacks. This medication should not be taken when an attack starts. Asthma medications can be expensive. Telling the client that he or she is using the inhaler incorrectly does not address the client's financial situation, which is the main issue here. Clients with limited incomes should be provided with community resources. Asking the client about fears related to breathlessness does not address the client's immediate concerns.

The nurse is caring for a client who is prescribed a long-acting beta2 agonist. The client states, "The medication is too expensive to use every day. I only use my inhaler when I have an attack." How should the nurse respond? a. "You are using the inhaler incorrectly. This medication should be taken daily." b. "If you decrease environmental stimuli, it will be okay for you to use the inhaler only for asthma attacks." c. "Tell me more about your fears related to feelings of breathlessness." d. "It is important to use this type of inhaler every day. Let's identify potential community services to help you."

ANS: D Positioning the client to alleviate dyspnea will help ease air hunger and anxiety. Administering oxygen therapy is also an important priority action. Determining the client's physical limitations is not a priority in this situation. Encouraging alternate rest and activity periods is not the immediate priority. Monitoring of heart rate, rhythm, and pulses is important, but is not the priority.

The nurse is caring for a client with heart failure in the coronary care unit. The client is now exhibiting signs of air hunger and anxiety. Which nursing intervention does the nurse perform first for this client? a. Determines the client's physical limitations b. Encourages alternate rest and activity periods c. Monitors and documents heart rate, rhythm, and pulses d. Positions the client to alleviate dyspnea

ANS: A, B, E Decreased tissue perfusion with heart failure may cause chest pain or angina. Tachycardia may occur as compensation for or as a result of decreased cardiac output. Fatigue is a symptom of poor tissue perfusion in clients with heart failure. Presence of a cough or dyspnea results as pulmonary venous congestion ensues. Clients with acute heart failure have dry cough and, when severe, pink, frothy sputum. Thick, yellow sputum is indicative of infection. Position for sleeping isn't a symptom; Clients usually find it difficult to lie flat because of dyspnea symptoms.

The nurse is caring for a client with heart failure. For which symptoms does the nurse assess? (Select all that apply.) a. Chest discomfort or pain b. Tachycardia c. Expectorating thick, yellow sputum d. Sleeping on back without a pillow e. Fatigue

ANS: B Prevention and early detection strategies are used to protect the client in sickle cell crisis from infection. Frequent and thorough handwashing is of the utmost importance. Drug therapy is a major defense against infections that develop in the client with sickle cell disease, but is not the most effective way that the nurse can reduce the potential for sepsis. Continually assessing the client for infection and monitoring the daily complete blood count with differential white blood cell count is early detection, not prevention. Taking vital signs every 4 hours will help with early detection of infection, but is not prevention.

The nurse is caring for a client with sickle cell disease. Which action is most effective in reducing the potential for sepsis in this client? a. Administering prophylactic drug therapy b. Frequent and thorough handwashing c. Monitoring laboratory values to look for abnormalities d. Taking vital signs every 4 hours, day and night

ANS: B Movement not only mobilizes secretions but helps strengthen respiratory muscles by impacting the effectiveness of gas exchange processes.

The nurse is caring for a patient who has decreased mobility. Which intervention is a simple and cost-effective method for reducing the risks of stasis of pulmonary secretions and decreased chest wall expansion? a. Antibiotics b. Frequent change of position c. Oxygen humidification d. Chest physiotherapy

ANS: D The normal respiratory drive is a person's level of carbon dioxide (CO2) in the arterial blood. The COPD patient had compensated for his chronic high levels of CO2, and his respiratory drive is dependent on his oxygen levels, not his CO2 levels. If the COPD patient's oxygen level is rapidly increased to what would be considered a normal level, it would compensate for his respiratory drive. The patient with COPD who has difficulty breathing should be given low levels of oxygen and closely observed for the quality and rate of ventilation. A dose of glucocorticoids will not address his immediate needs, but it may provide decreased inflammation and better ventilation over an extended period of time. Encouraging coughing and deep breathing in a patient with COPD does not meet his needs as effectively as administration of low-level oxygen does.

The nurse is caring for a patient with chronic obstructive pulmonary disease (COPD). The patient tells the nurse he is having a "hard time breathing." His respiratory rate is 32 breaths per minute, his pulse is 120 beats per minute, and the oxygen saturation is 90%. What would be the best nursing intervention for this patient? a. Begin oxygen via a face mask at 60% FiO2 (fraction of inspired oxygen). b. Administer a PRN (as necessary) dose of an intranasal glucocorticoid. c. Encourage coughing and deep breathing to clear the airway. d. Initiate oxygen via a nasal cannula, and begin at a flow rate of 3 L/min.

ANS: A Long-term use of nonsteroidal anti-inflammatory drugs such as ibuprofen (Motrin) causes fluid and sodium retention, which can worsen a client's HF. A diuretic may be used in the treatment of HF and hypertension. Although diabetes may be a risk factor for cardiovascular disease, it does not directly cause HF. In proper doses, Synthroid replaces thyroid hormone for those with hypothyroidism; it does not cause HF.

The nurse is caring for an 82-year-old client admitted for exacerbation of heart failure (HF). The nurse questions the client about the use of which medication because it raises an index of suspicion as to the worsening of the client's HF? a. Ibuprofen (Motrin) b. Hydrochlorothiazide (HydroDIURIL) c. NPH insulin d. Levothyroxine (Synthroid)

ANS: A Distributive shock is the type of shock that occurs when blood volume is not lost from the body but is distributed to the interstitial tissues, where it cannot circulate and deliver oxygen. Neurally-induced distributive shock may be caused by pain, anesthesia, stress, spinal cord injury, or head trauma. The other clients are at risk for hypovolemic and cardiogenic shock.

The nurse is caring for multiple clients in the emergency department. The client with which condition is at highest risk for distributive shock? a. Severe head injury from a motor vehicle accident b. Diabetes insipidus from polycystic kidney disease c. Ischemic cardiomyopathy from severe coronary artery disease d. Vomiting of blood from a gastrointestinal ulcer

ANS: D Heart and respiratory rates increased from the client's baseline level or a slight increase in diastolic blood pressure may be the only objective manifestation of this early stage of shock. Catecholamine release occurs early in shock as a compensation for fluid loss; blood pressure will be normal. Early in shock, the client displays rapid, not slow, respirations. Dysrhythmias are a late sign of shock; they are related to lack of oxygen to the heart.

The nurse is caring for postoperative clients at risk for hypovolemic shock. Which condition represents an early symptom of shock? a. Hypotension b. Bradypnea c. Heart blocks d. Tachycardia

ANS: A Impending pulmonary edema is characterized by a change in mental status, disorientation, and confusion, along with dyspnea and increasing fluid levels in the lungs. Dysphagia, sacral edema, and an irregular heart rate are not related to pulmonary edema.

The nurse is concerned that an older adult client with heart failure is developing pulmonary edema. What manifestation alerts the nurse to further assess the client for this complication? a. Confusion b. Dysphagia c. Sacral edema d. Irregular heart rate

ANS: B The client with SCD should receive annual influenza and pneumonia vaccinations; this helps prevent the development of these infections, which could cause a sickle cell crisis. Handwashing is a very important habit for the client with SCD to develop because it reduces the risk for infection. Prophylactic penicillin is given to clients with SCD orally twice a day to prevent the development of infection.

The nurse is educating a group of young women who have sickle cell disease (SCD). Which comment from a class member requires correction? a. "Frequent handwashing is an important habit for me to develop." b. "Getting an annual 'flu shot' would be dangerous for me." c. "I must take my penicillin pills as prescribed, all the time." d. "The pneumonia vaccine is protection that I need."

ANS: A, B, E, F The hematocrit is low (should be 42.6%), indicating a dilutional ratio of red blood cells (RBCs) to fluid. The serum sodium is low because of hemodilution. Microalbuminuria and proteinuria are present, indicating a decrease in renal filtration. This is an early warning sign of decreased compliance of the heart.

The nurse is evaluating the laboratory results for a client with heart failure. What results does the nurse expect? (Select all that apply.) a. Hematocrit (Hct), 32.8% b. Serum sodium, 130 mEq/L c. Serum potassium, 4.0 mEq/L d. Serum creatinine, 1.0 mg/dL e. Proteinuria f. Microalbuminuria

ANS: B Gathering all supplies needed for a chore at one time decreases the amount of energy needed.

The nurse is instructing a client with heart failure about energy conservation. Which is the best instruction? a. "Walk until you become short of breath and then walk back home." b. "Gather everything you need for a chore before you begin." c. "Pull rather than push or carry items heavier than 5 pounds." d. "Take a walk after dinner every day to build up your strength."

ANS: C Dopamine hydrochloride causes vasoconstriction that in turn increases cardiac output and mean arterial pressure, thereby improving tissue perfusion and oxygenation. Tachycardia is not a desired response but often occurs as a side effect.

The nurse is monitoring a client in hypovolemic shock who has been placed on a dopamine hydrochloride (Intropin) drip. Which manifestation is a desired response to this medication? a. Decrease in blood pressure b. Increase in heart rate c. Increase in cardiac output d. Decrease in mean arterial pressure

ANS: A Therapy during the second (late) phase of septic shock is aimed at enhancing the blood's ability to clot. Enoxaparin would increase the client's risk of bleeding and therefore should not be administered during the last phase of septic shock. Administering clotting factors, plasma, platelets, and other blood products will assist the client's blood to clot. Intravenous insulin to control hyperglycemia and antibiotic therapy would continue in the late phases of septic shock.

The nurse is planning care for a client with late-phase septic shock. All of the following treatments have been prescribed. Which prescription does the nurse question? a. Enoxaparin (Lovenox) 40 mg subcutaneous twice daily b. Transfusion of 2 units of fresh frozen plasma c. Regular insulin intravenous drip per protocol d. Cefazolin (Ancef) 1 g IV every 6 hours

ANS: A A decrease in urine output is a sensitive indicator of early shock. In severe shock, urine output is decreased (compared with fluid intake) or even absent. Alterations in temperature, irregular rhythms, and changes in bowel movements are not early signs of shock.

The nurse is planning discharge education for a client who had an exploratory laparotomy. Which nursing statement is appropriate when teaching the client to monitor for early signs of shock? a. "Monitor how much urine you void and report a decrease in the amount." b. "Take your temperature daily and report any below-normal body temperatures." c. "Assess your radial pulse every day and report an irregular rhythm." d. "Monitor your bowel movements and report ongoing constipation or diarrhea."

ANS: C Sodium nitroprusside (Nipride) must be protected from light to prevent degradation of the drug. It should be delivered via pump. This medication does not have any effect on respiratory rate. Hypertension is a sign of milrinone (Primacor) overdose.

The nurse is preparing to administer sodium nitroprusside (Nipride) to a client. Which important action related to the administration of this drug does the nurse implement? a. Assess the client's respiratory rate. b. Administer the medication with gravity tubing. c. Protect the medication from light with an opaque bag. d. Monitor for hypertensive crisis.

ANS: B To prevent a sternal wound infection, the nurse washes hands or performs hand hygiene as a priority. Vital signs do not necessarily need to be assessed beforehand. A mask and gown are not needed. The nurse should gather needed supplies, but this is not the priority.

The nurse is preparing to change a client's sternal dressing. What action by the nurse is most important? a. Assess vital signs b. Perform hand hygiene c. Don (put on) a mask and gown d. Gather needed supplies

ANS: C Teach everyone to prevent dehydration by having adequate fluid intake during exercise or when in a hot and dry environment. Insensitive water loss increases in this type of environment. Heat causes vasodilation as well, also contributing to water loss. The other statements are not accurate.

The nurse is providing community education for clients at risk for dehydration. One client states, "We are not at risk because we live in a hot and dry climate." What is the nurse's best response? a. "You are still at risk but not as high a risk as those who live in hot and humid climates." b. "Any type of heat can cause peripheral vasoconstriction, which causes the body to lose water." c. "In a hot and dry environment, the body can lose an increased amount of water without your knowledge." d. "Even though you are not at risk, you should drink adequate fluids when you exercise."

ANS: A Cough, a symptom of heart failure, is indicative of intra-alveolar edema; the provider should be notified. The client should call the provider for weight gain of 3 pounds in a week. The client should begin by walking 200 to 400 feet per day. Chest pain is indicative of myocardial ischemia and worsening of heart failure; the provider should be notified.

The nurse is providing discharge teaching to a client with heart failure, focusing on when to seek medical attention. Which statement by the client indicates a correct understanding of the teaching? a. "I will call the provider if I have a cough lasting 3 or more days." b. "I will report to the provider weight loss of 2 to 3 pounds in a day." c. "I will try walking for 1 hour each day." d. "I should expect occasional chest pain."

ANS: B, E, F Daily temperatures, washing dishes in hot sudsy water or a dishwasher, and rinsing toothbrushes in liquid bleach or in the dishwasher are infection precautions for the immune compromised client. Clients at increased risk because of immune suppression need to wear a facemask when in large crowds or around ill people. Water need not be bottled but should not be used if it has been standing for longer than 15 minutes. This population is not restricted from pets but is only advised not to change pet litter boxes.

The nurse is providing health education to a client on immunosuppressant therapy. Which instructions does the nurse include in this client's teaching? (Select all that apply.) a. "Wear a facemask at all times." b. "Take your temperature once a day." c. "Drink only bottled water." d. "Avoid any contact with pets." e. "Wash dishes with hot sudsy water." f. "Rinse your toothbrush in liquid laundry bleach."

ANS: B The children of the client with sickle cell disease will inherit the sickle cell trait, but may not inherit the disease. If both parents have the sickle cell trait, their children could get the disease.

The nurse is reinforcing information about genetic counseling to a client with sickle cell disease who has a healthy spouse. What information does the nurse include? a. "Sickle cell disease will be inherited by your children." b. "The sickle cell trait will be inherited by your children." c. "Your children will have the disease, but your grandchildren will not." d. "Your children will not have the disease, but your grandchildren could."

ANS: D Gastrointestinal absorption of digoxin is erratic. Many medications, especially antacids, interfere with its absorption. Clients are taught to hold their digoxin for bradycardia; a heart rate of 80 is too high for this cutoff.

The nurse is starting a client on digoxin (Lanoxin) therapy. What intervention is essential to teach this client? a. "Avoid taking aspirin or aspirin-containing products." b. "Increase your intake of foods high in potassium." c. "Hold this medication if your pulse rate is below 80 beats/min." d. "Do not take this medication within 1 hour of taking an antacid."

ANS: C Cotton balls can decrease pressure ulcers from the oxygen tubing. Continuous oxygen orders mean the client should wear the oxygen at all times. Oxygen fuels a fire. Wearing oxygen while grilling and smoking increases the risk for fire.

The nurse is teaching a client with chronic obstructive pulmonary disease who has been prescribed continuous oxygen therapy at home. Which statement indicates the client correctly understands the teaching? a. "I plan to wear my oxygen when I exercise and feel short of breath." b. "I will use my portable oxygen when grilling burgers in the backyard." c. "I plan to use cotton balls to cushion the oxygen tubing on my ears." d. "I will only smoke while I am wearing my oxygen via nasal cannula."

ANS: A, B, D, F It is critical to have others help the anemic client who is extremely tired. Although it may be difficult for him or her to ask for help, this practice should be stressed to the client. Drinking small protein or nutritional supplements will help rebuild the client's nutritional status. Having four to six small meals daily is preferred over three large meals; this practice conserves the body's expenditure of energy used in digestion and assimilation of nutrients. Stopping activities when strain on the cardiac or respiratory system is noted is critical. A complete bath should be performed only every other day; on days in between, the client can be taught to take a "mini" sponge bath, which will conserve energy and still be safe in preventing the risks for infection. Care activities should be spaced every hour or so rather than in groups to conserve energy; the time just before and after meals should be avoided.

The nurse is teaching a client with newly diagnosed anemia about conserving energy. What does the nurse tell the client? (Select all that apply.) a. "Allow others to perform your car during periods of extreme fatigue." b. "Drink small quantities of protein shakes and nutritional supplements daily." c. "Perform a complete bath daily to reduce your chance of getting an infection." d. "Provide yourself with four to six small, easy-to-eat meals daily." e. "Perform your care activities in groups to conserve your energy." f. "Stop activity when shortness of breath or palpitations are present."

ANS: A Dairy products such as milk, cheese, and eggs will provide the vitamin B12 that the client needs. Grains, leafy vegetables, and starchy vegetables are not a source of vitamin B12.

The nurse is teaching a client with vitamin B12 deficiency anemia about dietary intake. Which type of food does the nurse encourage the client to eat? a. Dairy products b. Grains c. Leafy vegetables d. Starchy vegetables

ANS: B Tobacco use is the most preventable cause of death and disease and is the most important risk factor in the development of impaired gas exchange. Age is not a modifiable risk factor. Drug overdose and immobility both contribute to impaired gas exchange but are not as significant as tobacco use.

What is the most significant modifiable risk factor for the development of impaired gas exchange? a. Age. b. Tobacco use. c. Drug overdose. d. Prolonged immobility.

ANS: A The age-related change that would affect airway clearance is decreased defense mechanisms, whereby the patient will have difficulty excreting anesthesia gas. The nurse needs to monitor the patient's oxygen status carefully to make sure the patient does not retain too much of the drug. Heart muscle thickening and mental status do not affect oxygenation in patients undergoing anesthesia. Lung capacity is not related to anesthesia induction.

The nurse needs to closely monitor the oxygen status of an elderly patient undergoing anesthesia because of which age-related change? a. Decreased lung defense mechanisms may cause ineffective airway clearance. b. Thickening of the heart muscle wall decreases cardiac output. c. Decreased lung capacity makes proper anesthesia induction more difficult. d. Alterations in mental status prevent patients' awareness of ineffective breathing.

ANS: B Jaundice is caused by the elevation of bilirubin level associated with red blood cell (RBC) hemolysis. The other tests would not be helpful in monitoring or treating a hemolytic anemia.

The nurse notes scleral jaundice in a patient being admitted with hemolytic anemia. The nurse will plan to check the laboratory results for the a. Schilling test. b. bilirubin level. c. stool occult blood test. d. gastric analysis testing.

ANS: B The client with heart failure typically has an enlarged heart that displaces the apical pulse to the left.

The nurse notes that the client's apical pulse is displaced to the left. What conclusion can be drawn from this assessment? a. This is a normal finding. b. The heart is hypertrophied. c. The left ventricle is contracted. d. The client has pulsus alternans.

ANS: D Digoxin causes bradycardia; hypokalemia potentiates digoxin. Because digoxin causes bradycardia, the medication should be held. Furosemide decreases circulating blood volume and depletes potassium; no indication suggests that the client has fluid volume excess at this time.

The nurse prepares to administer digoxin to a client with heart failure and notes the following information: Temperature: 99.8° F Pulse: 48 beats/min and irregular Respirations: 20 breaths/min Potassium level: 3.2 mEq/L What action does the nurse take? a. Give the digoxin; reassess the heart rate in 30 minutes. b. Give the digoxin; document assessment findings in the medical record. c. Hold the digoxin, and obtain a prescription for an additional dose of furosemide. d. Hold the digoxin, and obtain a prescription for a potassium supplement.

ANS: A Establishing an airway is the priority in all emergency situations. Although administering Plasmanate and normal saline, and typing and crossmatching for 4 units of PRBCs are important actions, airway always takes priority.

The nurse reviews the medical record of a client with hemorrhagic shock, which contains the following information: -Pulse 140 beats/min and thready -Blood pressure 60/40 mm Hg -Respirations 40/min and shallow -ABG respiratory acidosis -Lactate level 7 mmol/L All of these provider prescriptions are given for the client. Which does the nurse carry out first? a. Notify anesthesia for endotracheal intubation. b. Give Plasmanate 1 unit now c. Give normal saline solution 250 mL/hr d. Type and crossmatch for 4 units of packed red blood cells (PRBCs)

ANS: D When air is allowed to enter the pleural space, the lung will collapse and a chest tube will be inserted to remove the air and reestablish negative pressure in the pleural space. Patients with asthma do not require a chest tube. A bronchoscopy is done to evaluate the bronchi and lungs and to obtain a biopsy. A thoracentesis may be done to remove fluid from the pleural space. A chest tube may be inserted if there are complications from the thoracentesis or for the bronchoscopy.

The nurse would anticipate that which of the following patients will need to be treated with insertion of a chest tube? a. A patient with asthma and severe shortness of breath. b A patient undergoing a bronchoscopy for a biopsy. c. A patient with a pleural effusion requiring fluid removal. d. A patient experiencing a problem with a pneumothorax.

ANS: C Retained CO2 creates H+ byproducts that lower pH. This sends a chemical signal to increase respiratory rate and would result in increased ventilation. All other options would cause the ventilation rate to normalize or decrease to increase carbon dioxide retention or as the result of delivery of higher levels of oxygen to tissues.

The nurse would expect to see increased ventilations if a patient exhibits a. Increased oxygen saturation. b. Decreased carbon dioxide levels. c. Decreased pH. d. Increased hemoglobin levels.

ANS: D The neurologic system controls respiratory drive; the respiratory system controls delivery of oxygen to the lung capillaries; and the cardiac system is responsible for the perfusion of vital organs. These systems are primarily responsible for the adequacy of gas exchange in the body. The endocrine and hepatic systems are not directly involved with gas exchange. The immune system primarily protects the body against infection.

The nurse would identify which body systems as directly involved in the process of normal gas exchange? a. Hepatic system b. Endocrine system, Cardiovascular system c. Immune system, Hepatic system, Cardiovascular system d. Pulmonary system, Cardiovascular system, Neurological system

ANS: A Vital sign trends must be taken into consideration; a BP of 90/60 mm Hg may be normal for this client. Calling the surgeon is not necessary at this point, and increasing IV fluids is not indicated. The same equipment should be used when vital signs are taken postoperatively.

The nursing assistant is concerned about a postoperative client with blood pressure (BP) of 90/60 mm Hg, heart rate of 80 beats/min, and respirations of 22 breaths/min. What does the supervising nurse do? a. Compare these vital signs with the last several readings b. Request that the surgeon see the client c. Increase the rate of intravenous fluids d. Reassess vital signs using different equipment

ANS: A The child is exhibiting signs of increasing respiratory distress suggestive of a pneumothorax. The child needs to be seen as soon as possible.

The parent of a child with cystic fibrosis calls the clinic nurse and describes signs and symptoms of tachypnea, tachycardia, dyspnea, pallor, and cyanosis. What does the nurse suspect the child is experiencing? a. A pneumothorax b. Bronchodilation c. Carbon dioxide retention d. Extremely thick sputum

ANS: B Diffusion is the process of gases exchanging across the alveoli and capillaries of body tissues. Disassociation is not related to oxygenation. Perfusion is the ability of the cardiovascular system to carry oxygenated blood to tissues and return deoxygenated blood to the heart. Ventilation is the process of moving gases into and out of the lungs.

The process of exchanging gases through the alveolar capillary membrane is known as a. Disassociation. b. Diffusion. c. Perfusion. d. Ventilation.

ANS: B Although this may be indicated, it is not the priority action. These are signs of early congestive heart failure, and the physician should be notified. Withholding the infant's feeding is an incomplete response to the problem. Increasing oxygen may alleviate symptoms, but medications such as digoxin and furosemide are necessary to improve heart function and fluid retention. Notifying the physician is the priority nursing action.

What is the appropriate priority nursing action for the infant with a CHD who has an increased respiratory rate, is sweating, and is not feeding well? a. Recheck the infant's blood pressure. b. Alert the physician. c. Withhold oral feeding. d. Increase the oxygen rate.

ANS: A Children and their families need specific instructions on how to minimize crises, including preventing infections; maintaining adequate hydration; and addressing environmental concerns, such as avoidance of extreme cold. Genetic counseling is important, but teaching care for the child is a priority. Sickle cell anemia is a long-term, chronic illness. Multiple blood transfusions are an option for some children with sickle cell disease. The priority is that the child and the parents are properly prepared to manage the chronic disease.

What is the most important nursing consideration when caring for a child with sickle cell anemia? a. Teach the parents and child how to minimize crises. b. Refer the parents and child for genetic counseling. c. Help the child and family to adjust to a short-term disease. d. Observe for complications of multiple blood transfusions.

ANS: A Any effort to organize the right environment, plan, or literature is of no use if the parents are not ready to learn or have high anxiety. Decreasing level of anxiety is often needed before new information can be processed. A baseline assessment of prior knowledge should be taken into consideration before developing any teaching plan. Locating a quiet place for meeting with parents is appropriate; however, an assessment should be done before any teaching is done. Discussing a teaching plan with the nursing team is appropriate after an assessment of the parents' knowledge and readiness.

What is the nurse's first action when planning to teach the parents of an infant with a CHD? a. Assess the parents' anxiety level and readiness to learn. b. Gather literature for the parents. c. Secure a quiet place for teaching. d. Discuss the plan with the nursing team.

ANS: C Signs of systemic inflammatory response syndrome, which precedes sepsis, include rapid respiratory rate, leukocytosis, and tachycardia. In the early stage of septic shock, the client is usually warm and febrile. Hypotension does not develop until later in septic shock due to compensatory mechanisms. Respiratory alkalosis occurs early in shock because of an increased respiratory rate.

What typical sign/symptom indicates the early stage of septic shock? a. Pallor and cool skin b. Blood pressure 84/50 mm Hg c. Tachypnea and tachycardia d. respiratory acidosis

ANS: C Blood pressure measurements for upper and lower extremities are compared during an assessment for CHDs. Discrepancies in blood pressure between the upper and lower extremities cannot be determined if blood pressure is not measured in all four extremities. When a CHD is suspected, the blood pressure should be measured in all four extremities while the child is quiet. Discrepancies between upper and lower extremities may indicate cardiac disease. Blood pressure measurements when the child are crying are likely to be elevated; thus the readings will be inaccurate. Also, all four extremities need to be measured.

When assessing a child for possible congenital heart defects (CHDs), where should the nurse measure blood pressure? a. The right arm b. The left arm c. All four extremities d. Both arms while the child is crying

Answer: B Rationale: The patient with a low hemoglobin level and hematocrit is anemic and is most likely to experience fatigue. Fatigue develops because of the lowered oxygen-carrying capacity that leads to reduced tissue oxygenation with which to carry out cellular functions.

When caring for a patient with metastatic cancer, you note a hemoglobin level of 8.7 g/dL and hematocrit of 26%. You place highest priority on initiating interventions that can reduce A. thirst. B. fatigue. C. headache. D. abdominal pain.

ANS: D When caring for any client, determining airway and respiratory status is the priority. The airway takes priority over obtaining IV access, applying the blood pressure cuff, and assessing for changes in the client's mental status.

When caring for an obtunded client admitted with shock of unknown origin, which action does the nurse take first? a. Obtain IV access and hang prescribed fluid infusions b. Apply the automatic blood pressure cuff c. Assess level of consciousness and pupil reaction to light d. Check the airway and respiratory status

ANS: B Oxygen and carbon dioxide move across the alveolar membrane based on the partial pressure of each gas. Molecules of oxygen are not exchanged for molecules of carbon dioxide. The pressure gradient of each gas (carbon dioxide and oxygen) in the alveoli is responsible for the movement of each gas.

When evaluating the concept of gas exchange, how would the nurse best describe the movement of oxygen and carbon dioxide? a. Oxygen and carbon dioxide are exchanged across the capillary membrane to provide oxygen to hemoglobin. b. Gas moves from an area of high pressure to an area of low pressure across the alveolar membrane. c. The level of inspired oxygen must be sufficient to displace the carbon dioxide molecules in the alveoli. d. Gases are exchanged between the atmosphere and the blood based on the oxygen-carrying capacity of the hemoglobin.

ANS: C Weight loss in this client indicates effective fluid restriction and diuretic drug therapy. Lung crackles indicate intra-alveolar edema and fluid excess. Pedal edema indicates fluid excess. Sucking on ice chips indicates noncompliance with fluid restrictions; alternative methods of treating dry mouth should be explored.

When following up in the clinic with a client with heart failure, how does the nurse recognize that the client has been compliant with fluid restrictions? a. Auscultation of crackles b. Pedal edema c. Weight loss of 6 pounds since the last visit d. Reports sucking on ice chips all day for dry mouth

ANS: B Iron deficiency anemia is a type of microcytic, hypochromic anemia.

When obtaining assessment data from a patient with a microcytic, hypochromic anemia, you question the patient about a. folic acid intake. b. dietary intake of iron. c. a history of gastric surgery. d. a history of sickle cell anemia.

ANS: B Good nutritional sources of iron include boiled egg yolk, liver, green leafy vegetables, cream of Wheat, dried fruits, beans, nuts, and whole-grain breads.

When teaching the parents of a young child about iron deficiency anemia, the nurse would tell them that a rich source of iron is: a. an egg white. b. cream of Wheat. c. a banana. d. a carrot.

ANS: A Ventricular septal defect causes a left-to-right shunting of blood, thus increasing pulmonary blood flow. Coarctation of the aorta is a stenotic lesion that causes increased resistance to blood flow from the proximal to distal aorta. THe defects associated with tetralogy of Fallot result in a right-to-left shunting of blood, thus decreasing pulmonary blood flow. Pulmonary stenosis causes obstruction of blood flow from the right ventricle to the pulmonary artery. Pulmonary blood flow is decreased.

Which CHD results in increased pulmonary blood flow? a. Ventricular septal defect b. Coarctation of the aorta c. Tetralogy of Fallot d. Pulmonary stenosis

ANS: C The frequent transfusions used to treat thalassemia major lead to iron toxicity in patients unless iron chelation therapy is consistently used. Iron supplementation is avoided in patients with thalassemia. There is no need to avoid intramuscular injections. The goal for patients with thalassemia major is to maintain a hemoglobin of 10 g/dL or greater.

Which action will the nurse include in the plan of care for a patient who has thalassemia major? a. Teach the patient to use iron supplements. b. Avoid the use of intramuscular injections. c. Administer iron chelation therapy as needed. d. Notify health care provider of hemoglobin 11g/dL.

ANS: B A compensatory response to shock is vasoconstriction. Initially, the diastolic pressure increases but systolic pressure remains the same. As a result, the difference between the systolic and diastolic pressures (pulse pressure), is smaller or "narrower." When interventions are inadequate and shock worsens, systolic pressure decreases as cardiac output decreases. This causes the pulse pressure to narrow even further, indicating that shock is progressing. Although an increase in urine output usually signals improvement, a change of 1 mL/hr is within the margin of measurement error and is meaningless in this situation.

Which change in laboratory value or clinical manifestations in a client with hypovolemic shock indicates to the nurse that current therapy may need to be changed? a. Urine output increases from 5 mL/hour to 6 mL/hour b. Pulse pressure decreases from 28 mm Hg to 22 mm Hg c. Serum potassium level increases from 3.6 mEq/L to 3.9 mEq/L d. Core body temperature increases from 98.2° F (36.8° C) to 98.8° F (37.1° C)

ANS: B Because clients with aplastic anemia usually have low white blood cell counts that place them at high risk for infection, roommates such as the client with G6PD deficiency anemia should be free from infection or infection risk. The client with sickle cell disease has two draining leg ulcer infections that would threaten the diminished immune system of the client with aplastic anemia. The client with leukemia who is receiving induction chemotherapy and the client with idiopathic thrombocytopenia who is taking steroids are at risk for development of infection, which places the client with aplastic anemia at risk, too.

Which client does the nurse assign as a roommate for the client with aplastic anemia? a. a 23-year-old with sickle cell disease who has two draining leg ulcers b. A 28-year-old with glucose-6-phosphate dehydrogenase (G6PD) deficiency anemia who is receiving mannitol (Osmitrol) c. A 30-year-old with leukemia who is receiving induction chemotherapy d. A 34-year-old with idiopathic thrombocytopenia who is taking steroids

ANS: B Stem cell transplantation in the home setting requires support, assistance, and coordination from others. The client cannot manage this type of care on his own. The client must be emotionally stable to be a candidate for this type of care. It is acceptable for the client's spouse to support the client undergoing this procedure. It is not unexpected for the client to be taking several prescriptions. Five miles is an acceptable distance from the hospital, in case of emergency.

Which client statement indicates that stem cell transplantation that is scheduled to take place in his home is not a viable option? a. "I don't feel strong enough, but my wife said she would help." b. "I was a nurse, so I can take care of myself." c. "I will have lots of medicine to take." d. "We live 5 miles from the hospital."

ANS: A, D, E The client who is unrestrained in a motor vehicle accident is prone to multiple trauma and bleeding. Surgical clients are at high risk for hypovolemic shock owing to fluid loss and hemorrhage. Older adult clients are prone to shock; a gastrointestinal virus results in fluid losses. Unless injured or working in excessive heat, the construction worker and the athlete are not at risk for hypovolemic shock; they may be at risk for dehydration.

Which clients are at immediate risk for hypovolemic shock? (Select all that apply.) a. Unrestrained client in motor vehicle accident b. Construction worker c. Athlete d. Surgical intensive care client e. 85-year-old with gastrointestinal virus

ANS: D The manifestations of hypotension, pale and clammy skin, and decreased urine output are associated with any type of shock, including hypovolemic shock and septic shock. Sepsis and septic shock, however, are associated with disseminated intravascular coagulation, which consumes clotting factors and leaves the client at high risk for hemorrhage. One of the earliest manifestations of septic shock is bleeding from any area of nonintact skin, including IV insertion sites.

Which clinical manifestation in a client alerts the nurse to the probability of septic shock instead of hypovolemic shock? a. Hypotension b. Pale, clammy skin c. Decreased urine output d. Oozing of blood at the IV site

ANS: B Low-grade fever and mild hypotension indicate very early sepsis, but with treatment, the probability of recovery is high. Localized erythema and edema indicate local infection. A low oxygen saturation rate and decreased cognition indicate active (not early) sepsis. Reduced urinary output and increased respiratory rate indicate severe sepsis.

Which clinical symptoms in a postoperative client indicate early sepsis with an excellent recovery rate if treated? a. Localized erythema and edema b. Low-grade fever and mild hypotension c. Low oxygen saturation and decreased cognition d. Reduced urinary output and increased respiratory rate

ANS: B Because the patient with aplastic anemia has pancytopenia, the patient is at risk for infection and bleeding. There is no increased risk for seizures, neurogenic shock, or pulmonary edema.

Which collaborative problem will the nurse include in a care plan for a patient admitted to the hospital with idiopathic aplastic anemia? a. Potential complication: seizures b. Potential complication: infection c. Potential complication: neurogenic shock d. Potential complication: pulmonary edema

ANS: C A 7-year-old is at an age when medication administration responsibility ought to be initiated. The spacer whistle is significant, although its significance varies with each type of spacer. Children may use dry powder inhalers when they are old enough to have a rapid inhalation.

Which comments by the parents of a 7-year-old child with asthma indicate comprehension of instructions regarding medication use for control of the illness? a. The medications are too complicated for a 7-year-old to understand. b. If a spacer is used, a whistling sound indicates that the medication is being inhaled correctly. c. A spacer used on an inhaler helps trap the medication so it is inhaled more readily. d. Dry powder inhalers are for adult use only.

ANS: C BNP is produced and released by the ventricles when the client has fluid overload as a result of HF; a normal value is less than 100 pg/mL. Hypokalemia may occur in response to diuretic therapy for HF, but may also occur with other conditions; it is not specific to HF. Ejection fraction of 60% represents a normal value of 50% to 70%. Consolidation on chest x-ray may indicate pneumonia.

Which diagnostic test result is consistent with a diagnosis of heart failure (HF)? a. Serum potassium level of 3.2 mEq/L b. Ejection fraction of 60% c. B-type natriuretic peptide (BNP) of 760 ng/dL d. Chest x-ray report showing right middle lobe consolidation

ANS: A Capoten is a drug in an ACE inhibitor. Lasix is a loop diuretic. Aldactone blocks the action of aldosterone. Diuril works on the distal tubules.

Which drug is an angiotensin-converting enzyme (ACE) inhibitor? a. Captopril (Capoten) b. Furosemide (Lasix) c. Spironolactone (Aldactone) d. Chlorothiazide (Diuril)

ANS: C The calf swelling and pain suggest that the patient may have developed a deep vein thrombosis, which will require diagnosis and treatment to avoid complications such as pulmonary embolus. The other findings will also be reported to the health care provider but are expected in a patient with this diagnosis.

Which finding about a patient with polycythemia vera is most important for the nurse to report to the health care provider? a. Hematocrit 55% b. Presence of plethora c. Calf swelling and pain d. Platelet count 450,000/μL

ANS: B The absence of neurologic problems is an important diagnostic finding and differentiates folic acid deficiency from cobalamin deficiency.

Which finding allows you to identify the patient's anemia as folic acid deficiency rather than B12 deficiency? a. Loss of appetite b. Lack of neuromuscular symptoms c. Red tongue d. Change in nail shape

ANS: A, B, & C Whole-grain foods and beans are high in folic acid.

Which foods should you encourage patients with folic acid deficiency to include in their daily food intake (select all that apply)? a. Ready-to-eat cereal b. Wheat tortillas c. Lentils d. Strawberries e. Potatoes

ANS: A There are many causes of cobalamin (B12) deficiency. The most common cause is pernicious anemia, a disease in which the gastric mucosa is not secreting intrinsic factor (IF) because of antibodies being directed against the gastric parietal cells or IF itself. Other causes of cobalamin deficiency include gastrectomy, gastritis, nutritional deficiency, chronic alcoholism, and hereditary enzymatic defects of cobalamin use.

Which individual is at high risk for a cobalamin (vitamin B12) deficiency anemia? a. A 47-year-old man who had a gastrectomy (removal of the stomach) b. A 54-year-old man with a history of irritable bowel disease and ulcerative colitis c. A 26-year-old woman who complains of heavy menstrual periods d. A 15-year-old girl who is a vegetarian

ANS: C Exposure to crowds increases the patient's risk for infection, the most common cause of sickle cell crisis. There is no restriction on caffeine use. Iron supplementation is generally not recommended. A high-fluid intake is recommended.

Which instruction will the nurse plan to include in discharge teaching for the patient admitted with a sickle cell crisis? a. Take a daily multivitamin with iron. b. Limit fluids to 2 to 3 quarts per day. c. Avoid exposure to crowds when possible. d. Drink only two caffeinated beverages daily.

ANS: C High-Fowler's position and placing the legs in a dependent position will decrease venous return to the heart, thus decreasing pulmonary venous congestion. Monitoring of vital signs will detect abnormalities, but will not prevent them. Reassuring the client and a family member's presence may help to alleviate anxiety, but dyspnea and anxiety result from hypoxemia secondary to intra-alveolar edema, which must be relieved.

Which intervention best assists the client with acute pulmonary edema in reducing anxiety and dyspnea? a. Monitor pulse oximetry and cardiac rate and rhythm. b. Reassure the client that his distress can be relieved with proper interventions c. Place the client in high-Fowler's position with the legs down. d. Ask a family member to remain with the client

ANS: A The other interventions focus on preventing venous stasis, clot formation, and myocardial infarction. Using a soft-bristled toothbrush minimizes trauma to the gums and prevents bleeding.

Which intervention is most important for the nurse to teach the client with polycythemia vera to prevent injury as a result of the increased bleeding tendency? a. Use a soft-bristled toothbrush. b. Drink at least 3 liters of liquids per day. c. Wear gloves and socks outdoors in cool weather. d. Exercise slowly and only on the advice of your physician.

ANS: B A decreased segmented neutrophil count is indicative of late sepsis. Serum lactate is increased in late sepsis. Monocytosis is usually seen in diseases such as tuberculosis and Rocky Mountain spotted fever. An increased platelet count does not indicate sepsis; late in sepsis, platelets may decrease due to consumptive coagulopathy.

Which laboratory result is seen in late sepsis? a. Decreased serum lactate b. Decreased segmented neutrophil count c. Increased numbers of monocytes d. Increased platelet count

ANS: C Rationale: Both reduced urine output and thirst are stimulated by a decreasing circulating blood volume. When people can respond to thirst by drinking, the action compensates temporarily by increasing circulating fluid volume. Decreased or absent urine output compensates by preventing a greater fluid loss. The fluid that would have been lost from the body as urine is retained. This is why hourly urine output measurements are such a sensitive indicator for whether shock is improving or progressing. Edema and weight gain are not compensations for circulating blood volume. Confusion and lethargy are responses to circulating blood volume, not compensation to improve it. Increasing pulse and respiratory rates compensate for hypoxia, not for reduced volume.

Which manifestations of shock are a result of compensatory mechanisms to maintain circulating blood volume? a. Edema and weight gain b. Confusion and lethargy c. Decreased urine output and thirst d. Increased pulse and respiratory rates

ANS: B Beta-adrenergic blocking agents such as carvedilol reverse consequences of sympathetic stimulation and catecholamine release that worsen heart failure; this category of pharmacologic agents improves morbidity, mortality, and quality of life. Dobutamine and digoxin are inotropic agents used to improve myocardial contractility but have not been directly associated with improving morbidity and mortality. Bumetanide is a high-ceiling diuretic that promotes fluid excretion; it does not improve morbidity and mortality.

Which medication, when given in heart failure, may improve morbidity and mortality? a. Dobutamine (Dobutrex) b. Carvedilol (Coreg) c. Digoxin (Lanoxin) d. Bumetanide (Bumex)

ANS: D The RN with current intensive care experience who is not caring for a postoperative client would be an appropriate assignment. Care of the unstable client with intubation and mechanical ventilation is not within the scope of practice for the LPN/LVN. A client who is experiencing septic shock is too complex for the new RN. Although the RN who is also caring for the post-CABG client is experienced, this assignment will put the post-CABG client at risk for MRSA infection.

Which nurse should be assigned to care for an intubated client who has septic shock as the result of a methicillin-resistant Staphylococcus aureus (MRSA) infection? a. The LPN/LVN who has 20 years of experience b. The new RN who recently finished orienting and is working independently with moderately complex clients c. The RN who will also be caring for a client who had coronary artery bypass graft (CABG) surgery 12 hours ago d. The RN with 2 years of experience in intensive care

ANS: C Daily weight assessment is included in the role of the nursing assistant, who will report the weights to the RN. The role of the professional nurse is to perform assessments; determining alcohol intake, monitoring pain level, and assessing for peripheral edema should not be delegated.

Which nursing action may be delegated to a nursing assistant working on the medical unit? a. Determine the usual alcohol intake for a client with cardiomyopathy b. Monitor the pain level for a client with acute pericarditis c. Obtain daily weights for several clients with class IV heart failure d. Check for peripheral edema in a client with endocarditis.

ANS: C Limiting the need for ambulation on the first day of admission to sitting in a chair or performing basic leg exercises promotes physical rest and reduced oxygen demand. Monitoring of vital signs will alert the nurse to increased energy expenditures but will not prevent them. Waiting until tachycardia occurs permits increased oxygen demand; the nurse should prevent this situation.

Which nursing intervention for a client admitted today with heart failure will assist the client to conserve energy? a. The client ambulates around the nursing unit with a walker. b. The nurse monitors the client's pulse and blood pressure frequently. c. The nurse obtains a bedside commode before administering furosemide. d. The nurse returns the client to bed when he becomes tachycardic.

ANS: A The goal of the nursing action should be the prevention of pneumonia; the action that best addresses this is to cough, turn, and deep breathe to keep secretions from pooling at the base of the lungs. Drinking through a straw increases the risk of aspiration. Humidification thins respiratory secretions, making them easier to expel. Monitoring oxygen status is important but is not a method of prevention

Which nursing intervention is most effective in preventing hospital-acquired pneumonia in an elderly patient? a. Assist patient to cough, turn, and deep breathe every 2 hours. b. Encourage patient to drink through a straw to prevent aspiration. c. Discontinue humidification delivery device to keep excess fluid from lungs. d. Monitor oxygen saturation, and frequently assess lung bases.

ANS: C For all causes of hemolysis, a major focus of treatment is to maintain renal function. When RBCs are hemolyzed, the hemoglobin molecule is released and filtered by the kidneys. The accumulation of hemoglobin molecules can obstruct the renal tubules and lead to acute tubular necrosis

Which organ is at greatest risk due to the effects of hemolytic anemia? a. Heart b. Spleen c. Kidney d. Liver

ANS: A The cause of sickle cell anemia involves increased hemolysis. Thalassemias and folic acid deficiencies decrease erythropoiesis, whereas the anemia related to menstruation is a direct result of blood loss.

Which patient is most likely to experience anemia caused by increased destruction of RBCs? a. An African American man who has a diagnosis of sickle cell disease b. A 59-year-old man whose alcoholism has precipitated folic acid deficiency c. A 30-year-old woman with a history of "heavy periods" accompanied by anemia d. A 3-year-old child whose impaired growth and development is attributable to thalassemia

ANS: A Patients with aplastic anemia are at risk for infection because of the low white blood cell production associated with this type of anemia, so the nurse should avoid assigning a roommate with any possible infectious process.

Which patient should the nurse assign as the roommate for a patient who has aplastic anemia? a. A patient with chronic heart failure b. A patient who has viral pneumonia c. A patient who has right leg cellulitis d. A patient with multiple abdominal drains

ANS: A It is normal for the stools to appear black when a patient is taking iron, and the patient should not call the doctor about this. The other patient statements are correct.

Which patient statement to the nurse indicates a need for additional instruction about taking oral ferrous sulfate? a. "I will call my health care provider if my stools turn black." b. "I will take a stool softener if I feel constipated occasionally." c. "I should take the iron with orange juice about an hour before eating." d. "I should increase my fluid and fiber intake while I am taking iron tablets."

ANS: A & B Avoiding dehydration and high altitudes helps to prevent crises. Vitamins, dairy products, and grapefruit juice cannot help the patient to prevent attacks of sickle cell disease.

Which points should be included in teaching the patient with sickle cell disease (select all that apply)? a. Avoid dehydration. b. Avoid high altitudes. c. Take cobalamin (vitamin B12) regularly. d. Consume dairy products frequently. e. Increase consumption of grapefruit juice.

ANS: B Fluid shifts from vascular to intra-abdominal may cause decreased circulating blood volume and poor tissue perfusion. Volume depletion is only one reason why a person may require a blood transfusion; anemia is another. The client receiving a blood transfusion does not have as high a risk as the client with severe ascites. Myocardial infarction results in tissue necrosis in the heart muscle; no blood or fluid losses occur. Owing to excess antidiuretic hormone secretion, the client with SIADH will retain fluid and therefore is not at risk for hypovolemic shock.

Which problem in the clients below best demonstrates the highest risk for hypovolemic shock? a. Client receiving blood transfusion b. Client with severe ascites c. Client with myocardial infarction d. Client with syndrome of inappropriate antidiuretic hormone (SIADH) secretion

ANS: C The post-kidney transplant client will need to take lifelong immune suppressant therapy and is at risk for infection from internal and external organisms. Pernicious anemia is related to lack of vitamin B12, not to bone marrow failure (aplastic anemia), which would place the client at risk for infection. Inflammation of the pericardial sac is an inflammatory condition that does not pose a risk for septic shock. Although owning pets, especially cats and reptiles, poses a risk for infection, the immune-suppressed kidney transplant client has a very high risk for infection, sepsis, and death.

Which problem places a client at highest risk for sepsis? a. Pernicious anemia b. Pericarditis c. Post kidney transplant d. Client owns an iguana

ANS: D The skin forms the first barrier to prevent entry of organisms into the body; this client is at very high risk for sepsis and death. Although the client with kidney failure has an increased risk for infection, his skin is intact, unlike the client with burn injury. Although the liver acts as a filter for pathogens, the client with cirrhosis has intact skin, unlike the burned client. The client with lung cancer may be at risk for increased secretions and infection, but risk is not as high as for a client with open skin.

Which problem places a person at highest risk for septic shock? a. Kidney failure b. Cirrhosis c. Lung cancer d. 40% burn injury

ANS: D Jaundice is likely because the increased destruction of RBCs causes an elevation in bilirubin levels. The spleen and liver may enlarge because of their hyperactivity, which is related to macrophage phagocytosis of the defective erythrocytes. The other symptoms are common to all types of anemia.

Which sign or symptom would you recognize as a unique characteristic specific to hemolytic anemia? a. Tachycardia b. Weakness c. Decreased RBCs d. Jaundice

ANS: D Because infection is the most common cause of a sickle cell crisis, influenza, Haemophilus influenzae, pneumococcal pneumonia, and hepatitis immunizations should be administered. Although continuous dose opioids and oxygen may be administered during a crisis, patients do not receive these therapies to prevent crisis. Hydroxyurea (Hydrea) is a medication used to decrease the number of sickle cell crises.

Which statement by a patient indicates good understanding of the nurse's teaching about prevention of sickle cell crisis? a. "Home oxygen therapy is frequently used to decrease sickling." b. "There are no effective medications that can help prevent sickling." c. "Routine continuous dosage narcotics are prescribed to prevent a crisis." d. "Risk for a crisis is decreased by having an annual influenza vaccination."

ANS: D To ensure the correct dosage, the medication should be measured with a syringe. The medication should not be mixed with formula or food. It is difficult to judge whether the child received the proper dose if the medication is placed in food or formula. To prevent toxicity, the parent should not repeat the dose without contacting the child's physician. For maximum effectiveness, the medication should be given at the same time everyday.

Which statement suggests that a parent understands how to correctly administer digoxin? a. "I measure the amount I am supposed to give with a teaspoon." b. "I put the medicine in the baby's bottle." c. "When she spits up right after I give the medicine, I give her another dose." d. "I give the medicine at 8 in the morning and evening every day."

ANS: B The infant with congestive heart failure may tire easily. If the infant does not consume an adequate amount of formula in 30 minutes, gavage feedings should be considered. The infant with congestive heart failure may tire easily, so the feeding should not continue beyond 30 minutes. Infants with congestive heart failure may be breastfed. Feedings every 3 hours is a frequently used interval. If the infant were fed less frequently than every 3 hours, more formula would need to be consumed and would tire the infant. The infant is fed smaller amounts of concentrated formula every 3 hours.

Which strategy is appropriate when feeding the infant with congestive heart failure? a. Continue the feeding until a sufficient amount of formula is taken. b. Limit feeding time to no more than 30 minutes. c. Always bottle feed every 4 hours. d. Feed larger volumes of concentrated formula less frequently.

ANS: A, D, E Respirations of 68 for an 8-month-old infant are high. The nurse needs to assess for retractions and wheezing. A 2-year-old who becomes quiet following respiratory distress could be experiencing decompensation and requires an evaluation. Suctioning is a sterile procedure that only the nurse should perform.

Which tasks should the nurse perform rather than delegate to an assistant? (Select all that apply.) a. Suctioning a 2-year-old with a tracheostomy. b. Changing the diaper of the 3-month-old infant recovering from RSV. c. Walking with a 2-year-old who has an IV receiving antibiotics for pneumonia. d. Relieving the nurse who is watching a 2-year-old with croup, because he now sounds quiet. e. Taking the temperature of an 8-month-old infant with bronchiolitis whose respirations are 68 and who is irritable.

ANS: D When the patient is lying flat, the jugular veins are at the level of the right atrium, so JVD is a common (but not a clinically significant) finding. Obtaining vital signs and oxygen saturation is not warranted at this point. JVD is an expected finding when a patient performs the Valsalva maneuver because right atrial pressure increases. JVD that persists when the patient is sitting at a 30- to 45-degree angle or greater is significant. The nurse will document the JVD in the medical record if it persists when the head is elevated.

While assessing a patient who was admitted with heart failure, the nurse notes that the patient has jugular venous distention (JVD) when lying flat in bed. Which action should the nurse take next? a. Document this finding in the patient's record. b. Obtain vital signs, including oxygen saturation. c. Have the patient perform the Valsalva maneuver. d. Observe for JVD with the patient upright at 45 degrees.

ANS: A Left-sided heart failure results in pulmonary congestion, the signs and symptoms of which include shortness of breath, crackles, and discomfort when lying supine. Right-sided heart failure is systemic and results in peripheral edema and hepatojugular distention. Atrial fibrillation results in an irregular heart rate. Myocardial ischemia most often results in chest pain, along with shortness of breath, nausea, and fatigue.

While performing an assessment, the nurse hears crackles in the patient's lung fields. The nurse also learns that the patient is sleeping on three pillows. What do these symptoms most likely indicate? a. Left-sided heart failure b. Right-sided heart failure c. Atrial fibrillation d. Myocardial ischemia

ANS: B Specific clinical manifestations may be related to iron-deficiency anemia. Pallor is the most common finding, and glossitis (inflammation of the tongue) is the second most common; another finding is cheilitis (inflammation of the lips). The patient may report headache, paresthesias, and a burning sensation of the tongue, all of which are caused by lack of iron in the tissues. A sore tongue is a sign of cobalamin (B12) deficiency. Tenting skin is a sign of dehydration that often accompanies diarrhea. Blue mucous membranes are associated with cyanosis.

You are caring for a patient with a diagnosis of iron-deficiency anemia. Which clinical manifestations are you most likely to observe when assessing this patient? a. Convex nails, bright red gums, and alopecia b. Brittle nails; smooth, shiny tongue; and cheilosis c. Tenting of the skin, sunken eyes, and complaints of diarrhea d. Pale pink tongue; dull, brittle hair; and blue mucous membranes

ANS: A & B Anemia of chronic disease, also called anemia of inflammation, is associated with an underproduction of RBCs and mild shortening of RBC survival. The RBCs are usually normocytic, normochromic, and hypoproliferative. The anemia is usually mild, but it can be more severe.

You correctly identify which descriptions as characteristic of anemia of chronic disease (select all that apply) a. Normocytic. b. Normochromic. c. Microcytic. d. Hypochromic. e. Proliferative.

A nurse is monitoring a patient who is receiving an IV infusion of normal saline. The patient is apprehensive and presents with a pounding headache, rapid pulse rate, chills, and dyspnea. What would be the nurse's priority intervention related to these symptoms? a) Discontinue the infusion immediately, monitor vital signs, and report findings to primary care provider immediately. b) Slow the rate of infusion, notify the primary care provider immediately and monitor vital signs. c) Pinch off the catheter or secure the system to prevent entry of air, place the patient in the Trendelenburg position, and call for assistance. d) Discontinue the infusion immediately, apply warm, moist compresses to the site, and restart the IV at another site.

a) Discontinue the infusion immediately, monitor vital signs, and report findings to primary care provider immediately. The nurse is observing the signs and symptoms of speed shock: the body's reaction to a substance that is injected into the circulatory system too rapidly. The nursing interventions for this condition are: discontinue the infusion immediately, report symptoms of speed shock to primary care provider immediately, and monitor vital signs once signs develop. Answer (b) is interventions for fluid overload, answer (c) is interventions for air embolus, and answer (d) is interventions for phlebitis.

A nurse is administering a blood transfusion for a patient following surgery. During the transfusion, the patient displays signs of dyspnea, dry cough, and pulmonary edema. What would be the nurse's priority actions related to these symptoms? a) Slow or stop the infusion; monitor vital signs, notify the physician, place the patient in upright position with feet dependent. b) Stop the transfusion immediately and keep the vein open with normal saline, notify the physician stat, administer antihistamine parenterally as needed. c) Stop the transfusion immediately and keep the vein open with normal saline, notify the physician, and treat symptoms. d) Stop the infusion immediately, obtain a culture of the patient's blood, monitor vital signs, notify the physician, administer antibiotics stat.

a) Slow or stop the infusion; monitor vital signs, notify the physician, place the patient in upright position with feet dependent. The patient is displaying signs and symptoms of circulatory overload: too much blood administered. In answer (b) the nurse is providing interventions for an allergic reaction. In answer (c) the nurse is responding to a febrile reaction, and in answer (d) the nurse is providing interventions for a bacterial reaction.

The oncoming nurse is assigned to the following clients. Which client should the nurse assess first? a) a 47-year-old who had a colon resection yesterday and is reporting pain b) a 20-year-old, 2 days postoperative open appendectomy who refuses to ambulate today c) a 60-year-old who is 3 days post-myocardial infarction and has been stable. d) a newly admitted 88-year-old with a 2-day history of vomiting and loose stools

d) a newly admitted 88-year-old with a 2-day history of vomiting and loose stools Young children, older adults, and people who are ill are especially at risk for hypovolemia. Fluid volume deficit can rapidly result in a weight loss of 5% in adults and 10% in infants. A 5% weight loss is considered a pronounced fluid deficit; an 8% loss or more is considered severe. A 15% weight loss caused by fluid deficiency usually is life threatening. It is important to ambulate after surgery, but this can be addressed after assessment of the 88-year-old. The stable MI client presents no emergent needs at the present. The pain is important to address and should be addressed next or simultaneously (asking a colleague to give pain med).


Conjuntos de estudio relacionados

Globalization and Sustainability

View Set

Psych Statistics - Module 5 Test

View Set

Chapter 39-Assessment of Musculoskeletal Function

View Set

Module 1 - What is AI? - Defining AI & Its impacts

View Set

Data Structures - Asymptotic Analysis

View Set